0% found this document useful (0 votes)
67 views79 pages

CH 05

This document provides an overview of integral calculus and the problem of finding areas. It discusses how integral calculus builds upon differential calculus by relating the problems of finding tangent lines and rates of change to finding areas. The document outlines two approaches to defining and calculating areas and introduces the fundamental theorem of calculus, which relates these problems. It then focuses on finding the area between the graph of a function and the x-axis over an interval.
Copyright
© © All Rights Reserved
We take content rights seriously. If you suspect this is your content, claim it here.
Available Formats
Download as PDF, TXT or read online on Scribd
0% found this document useful (0 votes)
67 views79 pages

CH 05

This document provides an overview of integral calculus and the problem of finding areas. It discusses how integral calculus builds upon differential calculus by relating the problems of finding tangent lines and rates of change to finding areas. The document outlines two approaches to defining and calculating areas and introduces the fundamental theorem of calculus, which relates these problems. It then focuses on finding the area between the graph of a function and the x-axis over an interval.
Copyright
© © All Rights Reserved
We take content rights seriously. If you suspect this is your content, claim it here.
Available Formats
Download as PDF, TXT or read online on Scribd

January 18, 2001 14:01 g65-ch5 Sheet number 1 Page number 319 cyan magenta yellow black

I NTEGRATION

raditionally, that portion of calculus concerned with


finding tangent lines and rates of change is called differ-
ential calculus and that portion concerned with finding
areas is called integral calculus. However, we will see
in this chapter that the two problems are so closely re-
lated that the distinction between differential and integral
calculus is often hard to discern.
In this chapter we will begin with an overview of the
problem of finding areas—we will discuss what the term
“area” means, and we will outline two approaches to defin-
ing and calculating areas. Following this overview, we will
discuss the “Fundamental Theorem of Calculus,” which
is the theorem that relates the problems of finding tangent
lines and areas, and we will discuss techniques for calcu-
lating areas. Finally, we will use the ideas in this chapter
to continue our study of rectilinear motion and to examine
some consequences of the chain rule in integral calculus.
January 18, 2001 14:01 g65-ch5 Sheet number 2 Page number 320 cyan magenta yellow black

320 Integration

5.1 AN OVERVIEW OF THE AREA PROBLEM

In this introductory section we will consider the problem of calculating areas of plane
regions with curvilinear boundaries. All of the results in this section will be reexam-
ined in more detail later in this chapter, so our purpose here is simply to introduce the
fundamental concepts.
••••••••••••••••••••••••••••••••••••••
The main goal of this chapter is to study the following major problem of calculus:
FINDING AREA

5.1.1 THE AREA PROBLEM. Given a function f that is continuous and nonnegative
y on an interval [a, b], find the area between the graph of f and the interval [a, b] on the
x-axis (Figure 5.1.1).
y = f (x)

Of course, from a strictly logical point of view, we should first provide a precise definition
of the term area before discussing methods for calculating areas. However, in this section
we will treat the concept of area intuitively, postponing a more formal definition until
x Section 5.4.
a b Formulas for the areas of plane regions with straight-line boundaries (squares, rectangles,
triangles, trapezoids, etc.) were well known in many early civilizations. On the other hand,
Figure 5.1.1 obtaining formulas for regions with curvilinear boundaries (a circle being the simplest
case) caused problems for early mathematicians. The first real progress on such problems

was made by the Greek mathematician, Archimedes, who obtained the areas of regions
bounded by arcs of circles, parabolas, spirals, and various other curves by ingenious use
of a procedure later known as the method of exhaustion. That method, when applied to a
circle of radius r, consists of inscribing a succession of regular polygons in the circle and
allowing the number of sides n to increase indefinitely (Figure 5.1.2). As n increases, the
polygons tend to “exhaust” the region inside the circle, and the areas of those polygons
become better and better approximations to the exact area of the circle.


ARCHIMEDES (287 B.C.–212 B.C.). Greek mathematician and scientist. Born in Syracuse, Sicily, Archimedes was
the son of the astronomer Pheidias and possibly related to Heiron II, king of Syracuse. Most of the facts about his
life come from the Roman biographer, Plutarch, who inserted a few tantalizing pages about him in the massive
biography of the Roman soldier, Marcellus. In the words of one writer, “the account of Archimedes is slipped like
a tissue-thin shaving of ham in a bull-choking sandwich.”
Archimedes ranks with Newton and Gauss as one of the three greatest mathematicians who ever lived, and he
is certainly the greatest mathematician of antiquity. His mathematical work is so modern in spirit and technique
that it is barely distinguishable from that of a seventeenth-century mathematician, yet it was all done without
benefit of algebra or a convenient number system. Among his mathematical achievements, Archimedes developed
a general method (exhaustion) for finding areas and volumes, and he used the method to find areas bounded by
parabolas and spirals and to find volumes of cylinders, paraboloids, and segments of spheres. He gave a procedure
for approximating π and bounded its value between 3 10 1
71 and 3 7 . In spite of the limitations of the Greek numbering
system, he devised methods for finding square roots and invented a method based on the Greek myriad (10,000)
for representing numbers as large as 1 followed by 80 million billion zeros.
Of all his mathematical work, Archimedes was most proud of his discovery of the method for finding the
volume of a sphere—he showed that the volume of a sphere is two-thirds the volume of the smallest cylinder that
can contain it. At his request, the figure of a sphere and cylinder was engraved on his tombstone.
In addition to mathematics, Archimedes worked extensively in mechanics and hydrostatics. Nearly every
schoolchild knows Archimedes as the absent-minded scientist who, on realizing that a floating object displaces
its weight of liquid, leaped from his bath and ran naked through the streets of Syracuse shouting, “Eureka,
Eureka!”—(meaning, “I have found it!”). Archimedes actually created the discipline of hydrostatics and used it
to find equilibrium positions for various floating bodies. He laid down the fundamental postulates of mechanics,
discovered the laws of levers, and calculated centers of gravity for various flat surfaces and solids. In the excitement
of discovering the mathematical laws of the lever, he is said to have declared, “Give me a place to stand and I will
move the earth.”
Although Archimedes was apparently more interested in pure mathematics than its applications, he was an
engineering genius. During the second Punic war, when Syracuse was attacked by the Roman fleet under the
command of Marcellus, it was reported by Plutarch that Archimedes’ military inventions held the fleet at bay for
three years. He invented super catapults that showered the Romans with rocks weighing a quarter ton or more,
January 18, 2001 14:01 g65-ch5 Sheet number 3 Page number 321 cyan magenta yellow black

5.1 An Overview of the Area Problem 321

Table 5.1.1

n A(n)

100 3.13952597647
200 3.14107590781
300 3.14136298250
400 3.14146346236
500 3.14150997084 Figure 5.1.2
600 3.14153523487
700 3.14155046835 To see how this works numerically, let A(n) denote the area of a regular n-sided polygon
800 3.14156035548 inscribed in a circle of radius 1. Table 5.1.1 shows the values of A(n) for various choices
900 3.14156713408 of n. Note that for large values of n the area A(n) appears to be close to π (square units),
1000 3.14157198278 as one would expect. This suggests that for a circle of radius 1, the method of exhaustion is
2000 3.14158748588 equivalent to an equation of the form
3000 3.14159035683
lim A(n) = π
4000 3.14159136166 n→⬁
5000 3.14159182676 However, Greek mathematicians were very suspicious of the concept of “infinity” and
6000 3.14159207940 intentionally avoided explanations that referred to the “limiting behavior” of some quantity.
7000 3.14159223174 As a consequence, obtaining exact answers by the classical method of exhaustion was a
8000 3.14159233061 cumbersome procedure. In our discussion of the area problem, we will consider a more
9000 3.14159239839 modern version of the method of exhaustion that explicitly incorporates the notion of a
10000 3.14159244688 limit. Because our approach uses a collection of rectangles to “exhaust” an area, we will
refer to it as the rectangle method.
••••••••••••••••••••••••••••••••••••••
There are two basic methods for finding the area of the region having the form shown in
THE RECTANGLE METHOD FOR Figure 5.1.1—the rectangle method and the antiderivative method. The idea behind the
FINDING AREAS
rectangle method is as follows:
y = f (x)
• Divide the interval [a, b] into n equal subintervals, and over each subinterval construct
a rectangle that extends from the x-axis to any point on the curve y = f(x) that is above
the subinterval; the particular point does not matter—it can be above the center, above
an endpoint, or above any other point in the subinterval. In Figure 5.1.3 it is above the
center.
• For each n, the total area of the rectangles can be viewed as an approximation to the
exact area under the curve over the interval [a, b]. Moreover, it is evident intuitively
that as n increases these approximations will get better and better and will approach the
x exact area as a limit (Figure 5.1.4).
Figure 5.1.3
Later, this procedure will serve both as a mathematical definition and a method of compu-
tation—we will define the area under y = f(x) over the interval [a, b] as the limit of the
areas of the approximating rectangles, and we will use the method itself to approximate this
area.

and fearsome mechanical devices with iron “beaks and claws” that reached over the city walls, grasped the ships,
and spun them against the rocks. After the first repulse, Marcellus called Archimedes a “geometrical Briareus (a
hundred-armed mythological monster) who uses our ships like cups to ladle water from the sea.”
Eventually the Roman army was victorious and contrary to Marcellus’ specific orders the 75-year-old
Archimedes was killed by a Roman soldier. According to one report of the incident, the soldier cast a shadow
across the sand in which Archimedes was working on a mathematical problem. When the annoyed Archimedes
yelled, “Don’t disturb my circles,” the soldier flew into a rage and cut the old man down.
With his death the Greek gift of mathematics passed into oblivion, not to be fully resurrected again until the
sixteenth century. Unfortunately, there is no known accurate likeness or statue of this great man.
January 18, 2001 14:01 g65-ch5 Sheet number 4 Page number 322 cyan magenta yellow black

322 Integration

y y y
y = f (x) y = f (x) y = f (x)

x x x
a b a b a b

As n increases, the area of the rectangles


approaches the exact area under the curve.

Figure 5.1.4

y To illustrate this idea, we will use the rectangle method to approximate the area under the
y= x2
curve y = x 2 over the interval [0, 1] (Figure 5.1.5). We will begin by dividing the interval
[0, 1] into n equal subintervals, from which it follows that each subinterval has length 1/n;
the endpoints of the subintervals occur at
1 2 3 n−1
0, , , , . . . , , 1
n n n n
(Figure 5.1.6). We want to construct a rectangle over each of these subintervals whose height
x
is the value of the function f(x) = x 2 at some number in the subinterval. To be specific, let
1
us use the right endpoints, in which case the heights of our rectangles will be
Figure 5.1.5
 2  2   2
1 2 3
, , , . . . , 12
n n n
1 and since each rectangle has a base of width 1/n, the total area An of the n rectangles will
Width = n
be
    2  2  
1 2 3 n–1 1 2 2 3 1
0 n n n
...
n 1 An = + + + ··· + 1 2
(1)
n n n n
Subdivision of [0, 1] into n
subintervals of equal length For example, if n = 4, then the total area of the four approximating rectangles would be
 2  2  2  
Figure 5.1.6 A4 = 14 + 24 + 34 + 12 14 = 15 32
= 0.46875
Table 5.1.2 shows the result of evaluating (1) on a computer for some increasingly large
values of n. These computations suggest that the exact area is close to 13 . In Section 5.4 we
will prove that this area is exactly 13 by showing that
lim An = 1
3
n→⬁

Table 5.1.2

n 4 10 100 1000 10,000 100,000


An 0.468750 0.385000 0.338350 0.333834 0.333383 0.333338

Equation (1) may be written more concisely by using sigma notation, which is discussed
in Section 5.4 in detail. [Sigma () is an uppercase letter in the Greek alphabet used to
denote sums.] With sigma notation, the sum
 2  2   2
1 2 3
+ + + · · · + 12
n n n
may be expressed simply as
n  2
k
k=1
n
January 18, 2001 14:01 g65-ch5 Sheet number 5 Page number 323 cyan magenta yellow black

5.1 An Overview of the Area Problem 323

y
This notation tells us to form the sum of the terms that result when we substitute successive
y = f (x) integers for k in the expression (k /n)2 , starting with k = 1 and ending with k = n. Each
value of a positive integer n then determines a value of the sum. For example, if n = 4, then
4  2  2  2  2  2
k 1 2 3 4 30 15
= + + + = =
A(x)
k=1
4 4 4 4 4 16 8
x
In general, using sigma notation we write
a x
 
1 n k 2
Figure 5.1.7 An =
n k=1 n

• FOR THE READER.


• Many calculating utilities perform automatic summations for expres-








sions that involve some version of the sigma notation. If your calculating utility performs





such summations, use it to verify the value of A100 given in Table 5.1.2. (Otherwise, use it



• to confirm A10 .)

••••••••••••••••••••••••••••••••••••••
Despite the intuitive appeal of the rectangle method, the limits involved can be evaluated
THE ANTIDERIVATIVE METHOD FOR directly only in certain special cases. For this reason, work on the area problem remained at
FINDING AREAS
a rudimentary level until the latter half of the seventeenth century. Two results that were to
prove to be a major breakthrough in the area problem were discovered by mathematicians
Isaac Barrow and Isaac Newton in Great Britain, and Gottfried Leibniz in Germany. These
results appeared, without fanfare, as a proposition in Issac Barrow’s Lectiones geometricae.
Each of the two results can be used to solve the area problem.
The solution based on Proposition 11 was preferred by Isaac Newton and provides us
y with a paradoxically effective indirect approach to the area problem. According to this line
of argument, to find the area under the curve in Figure 5.1.1, one should first consider the
3
y=2 seemingly harder problem of finding the area A(x) between the graph of f and the interval
2
[a, x], where x denotes an arbitrary number in [a, b] (Figure 5.1.7). If one can discover a
1 2 formula for the area function A(x), then the area under the curve from a to b can be obtained
x
-4 -3 -2 -1 1 x 3 4 simply by substituting x = b into this formula.
x+1 This may seem to be a surprising approach to the area problem. After all, why should the
-2 problem of determining the area A(x) for every x in the interval [a, b] be more tractable than
the problem of computing a single value A(b)? However, the basis for this approach is the
(a) observation that although the area function A(x) may be difficult to compute, its derivative
y A (x) is easy to find. To illustrate, let us consider some examples of area functions A(x)
y=x+1 that can be computed from simple geometry.
3
2
x+1 Example 1 For each of the functions f , find the area A(x) between the graph of f and
1
x the interval [a, x] = [−1, x], and find the derivative A (x) of this area function.
-4 -3 -2 -1 1 x 3 4 (a) f(x) = 2 (b) f(x) = x + 1 (c) f(x) = 2x + 3
x+1
-2
Solution (a). From Figure 5.1.8a we see that
(b) A(x) = 2(x − (−1)) = 2(x + 1) = 2x + 2
y is the area of a rectangle of height 2 and base x + 1. For this area function,
3 A (x) = 2 = f(x)
2 2x + 3
1 Solution (b). From Figure 5.1.8b we see that
x
-4 -3 -2 -1 x 1 2 3 4 1 x2 1
A(x) = (x + 1)(x + 1) = +x+
2 2 2
y = 2x + 3 x+1 is the area of an isosceles right triangle with base and height equal to x + 1. For this area
(c) function,
Figure 5.1.8 A (x) = x + 1 = f(x)
January 18, 2001 14:01 g65-ch5 Sheet number 6 Page number 324 cyan magenta yellow black

324 Integration

Solution (c). Recall that the formula for the area of a trapezoid is A = 12 (b + b )h, where
b and b denote the lengths of the parallel sides of the trapezoid, and the altitude h denotes
the distance between the parallel sides. From Figure 5.1.8c we see that
A(x) = 12 ((2x + 3) + 1)(x − (−1)) = x 2 + 3x + 2
is the area of a trapezoid with parallel sides of lengths 1 and 2x + 3 and with altitude
x − (−1) = x + 1. For this area function,
A (x) = 2x + 3 = f(x) 
Note that in every case in Example 1,
A (x) = f (x) (2)
That is, the derivative of the area function A(x) is the function whose graph forms the upper
boundary of the region. We will show in Section 5.6 that Equation (2) is valid not simply for
linear functions such as those in Example 1, but for any continuous function. Thus, to find
the area function A (x), we can look instead for a (particular) function whose derivative is
f(x). This is called an antidifferentiation problem because we are trying to find A(x) by
“undoing” a differentiation. Whereas earlier in the text we were concerned with the process
of differentiation, we will now also be concerned with the process of antidifferentiation.
To see how this antiderivative method applies to a specific example, let us return to the
problem of finding the area between the graph of f (x) = x 2 and the interval [0, 1]. If we
let A(x) denote the area between the graph of f and the interval [0, x], then (2) tells us
that A (x) = f (x) = x 2 . By simple guesswork, we see that one function whose derivative
is f (x) = x 2 is 13 x 3 . It then follows from Theorem 4.8.3 that A(x) = 13 x 3 + C for some
constant C. This is where the decision to solve the area problem for a general right-hand
endpoint helps. If we consider the case x = 0, then the interval [0, x] reduces to a single
point. If we agree that the area above a single point should be taken as zero, then it follows
that
0 = A(0) = 13 03 + C = 0 + C = C or C=0
Therefore, A(x) = 1 3
3
x and the area between the graph of f and the interval [0, 1] is
A(1) = 3 . Note that this conclusion agrees with our numerical estimates in Table 5.1.2.
1

Although the antiderivative method provides us with a convenient solution to the area
problem, it appears to have little to do with the rectangle method. It would be nice to have
a solution that more clearly elucidates the connection between the operation of summing
areas of rectangles on the one hand and the operation of antidifferentiation on the other.
Fortunately, the solution to the area problem based on Barrow’s Proposition 19 reveals just
this connection. In addition, it allows us to formulate in modern language the approach to the
area problem preferred by Leibniz. We will provide this solution in Section 5.6 (Theorem
5.6.1), as well as develop a modern version of Barrow’s Proposition 11 (Theorem 5.6.3).
Together, these two approaches to the area problem comprise what is now known as the
Fundamental Theorem of Calculus.
••••••••••••••••••••••••••••••••••••••
We see that the rectangle method and the use of antidifferentiation provide us with quite
INTEGRAL CALCULUS different approaches to the area problem. The rectangle method is a frontal assault on the
problem, whereas antidifferentiation is more in the form of a sneak attack. In this chapter
we will carefully study both approaches to the problem.
In Sections 5.2 and 5.3 we will begin to develop some techniques for the process of
antidifferentiation, a process that is also known as integration. Later, in Section 5.5 we will
discuss a more general version of the rectangle method known as the Riemann sum. In much
the same way that area can be interpreted as a “limit” using the rectangle method, we will
define the definite integral as a “limit” of Riemann sums.
The definite integral and antidifferentiation are the twin pillars on which integral calculus
rests. Both are important. The definite integral is generally the means by which problems
January 18, 2001 14:01 g65-ch5 Sheet number 7 Page number 325 cyan magenta yellow black

5.2 The Indefinite Integral; Integral Curves and Direction Fields 325

in integral calculus are recognized and formulated. For example, in addition to the area
problem, the problems of computing the volume of a solid, finding the arc length of a curve,
and determining the work done in pumping water out of a tank are all examples of problems
that may be solved by means of a definite integral. On the other hand, it can be difficult
to obtain exact solutions to such problems by direct computation of a definite integral.
Fortunately, in many cases of interest, the Fundamental Theorem of Calculus will allow us
to evaluate a definite integral by means of antidifferentiation. Much of the power of integral
calculus lies in the two-pronged approach of the definite integral and antidifferentiation.

EXERCISE SET 5.1


••••••••••••••••••••••••••••••••••••••••••••••••••••••••••••••••••••••••••••••••••••••••••••••••••••••••••••••

9. f (x) = 3; [a, x] = [1, x]


In Exercises 1–8, estimate the area between the graph of
the function f and the interval [a, b]. Use an approxima- 10. f (x) = 5; [a, x] = [2, x]
tion scheme with n rectangles similar to our treatment of 11. f (x) = 2x + 2; [a, x] = [0, x]
f (x) = x 2 in this section. If your calculating utility will 12. f (x) = 3x − 3; [a, x] = [1, x]
perform automatic summations, estimate the specified area 13. f (x) = 2x + 2; [a, x] = [1, x]
using n = 10, 50, and 100 rectangles. Otherwise, estimate
14. f (x) = 3x − 3; [a, x] = [2, x]
this area using n = 2, 5, and 10 rectangles.
15. How do the area functions in Exercises 11 and 13 compare?
√ Explain.
1. f (x) = x; [a, b] = [0, 1]
1 16. Let f (x) denote a linear function that is nonnegative on the
2. f (x) = ; [a, b] = [0, 1] interval [a, b]. For each value of x in [a, b], define A(x) to
x+1
3. f (x) = sin x; [a, b] = [0, π] be the area between the graph of f and the interval [a, x].
(a) Prove that A(x) = 12 [f (a) + f (x)](x − a).
4. f (x) = cos x; [a, b] = [0, π/2]
(b) Use part (a) to verify that A (x) = f (x).
1 √
5. f (x) = ; [a, b] = [1, 2] 17. Let A denote the area between the graph of f (x) = x
x
and the interval [0, 1], and let B denote the area between
6. f (x) = cos x; [a, b] = [−π/2, π/2]
√ the graph of f (x) = x 2 and the interval [0, 1]. Explain
7. f (x) = 1 − x 2 ; [a, b] = [0, 1] geometrically why A + B = 1.

8. f (x) = 1 − x 2 ; [a, b] = [−1, 1] 18. Let A denote the area between the graph of f (x) = 1/x
and the interval [1, 2], and let B denote the area between
In Exercises 9–14, use simple area formulas from geometry
the graph of f and the interval [ 21 , 1]. Explain geometrically
to find the area function A(x) that gives the area between
why A = B.
the graph of the specified function f and the interval [a, x].
Confirm that A (x) = f (x) in every case.

5.2 THE INDEFINITE INTEGRAL; INTEGRAL CURVES


AND DIRECTION FIELDS

In the last section we saw the potential for antidifferentiation to play an important role
in finding exact areas. In this section we will develop some fundamental results about
antidifferentiation that will ultimately lead us to systematic procedures for solving
many antiderivative problems.

5.2.1 DEFINITION. A function F is called an antiderivative of a function f on a given


interval I if F  (x) = f(x) for all x in the interval.
January 18, 2001 14:01 g65-ch5 Sheet number 8 Page number 326 cyan magenta yellow black

326 Integration

For example, the function F (x) = 13 x 3 is an antiderivative of f(x) = x 2 on the interval


(−⬁, +⬁) because for each x in this interval
d 1 3
F  (x) = x = x 2 = f(x)
dx 3
However, F (x) = 13 x 3 is not the only antiderivative of f on this interval. If we add any
constant C to 13 x 3 , then the function G(x) = 13 x 3 + C is also an antiderivative of f on
(−⬁, +⬁), since
d 1 3
G (x) = x + C = x 2 + 0 = f(x)
dx 3
In general, once any single antiderivative is known, other antiderivatives can be obtained
by adding constants to the known antiderivative. Thus,

x , 13 x 3 + 2, 13 x 3 − 5, 13 x 3 + 2
1 3
3
are all antiderivatives of f(x) = x 2 .
It is reasonable to ask if there are antiderivatives of a function f that cannot be obtained
by adding some constant to a known antiderivative F . The answer is no—once a single
antiderivative of f on an interval I is known, all other antiderivatives on that interval are
obtainable by adding constants to the known antiderivative. This is so because Theorem
4.8.3 tells us that if two functions are differentiable on an open interval I such that their
derivatives are equal on I , then the functions differ by a constant on I . The following
theorem summarizes these observations.

5.2.2 THEOREM. If F (x) is any antiderivative of f(x) on an interval I, then for any
constant C the function F (x) + C is also an antiderivative on that interval. Moreover,
each antiderivative of f(x) on the interval I can be expressed in the form F (x) + C by
choosing the constant C appropriately.

••••••••••••••••••••••••••••••••••••••
The process of finding antiderivatives is called antidifferentiation or integration. Thus, if
THE INDEFINITE INTEGRAL
d
[F (x)] = f(x) (1)
dx
then integrating (or antidifferentiating) the function f(x) produces an antiderivative of the
form F (x) + C. To emphasize this process, Equation (1) is recast using integral notation,

f(x) dx = F (x) + C (2)


where C is understood to represent an arbitrary constant. It is important to note that (1) and
(2) are just different notations to express the same fact. For example,
d 1 3
x 2 dx = 13 x 3 + C is equivalent to x = x2
dx 3
Note that if we differentiate an antiderivative of f(x), we obtain f(x) back again. Thus,

d
f(x) dx = f(x) (3)
dx

The expression f(x) dx is called an indefinite integral. The adjective “indefinite”
emphasizes that the result of antidifferentiation is a “generic” function, descibed only up
to a constant summand. The “elongated s” that appears on the left side of (2) is called an

integral sign, the function f(x) is called the integrand, and the constant C is called the
Extract from the manuscript of Leibniz
constant of integration. Equation (2) should be read as:
dated October 29, 1675 in which the

integral sign first appeared. This notation was devised by Leibniz. In his early papers Leibniz used the notation “omn.” (an abbreviation for

the Latin word “omnes”) to denote integration. Then on October 29, 1675 he wrote, “It will be useful to write
for omn., thus
  for omn.  . . . .” Two or three weeks later he refined the notation further and wrote [ ] dx
rather than alone. This notation is so useful and so powerful that its development by Leibniz must be regarded
as a major milestone in the history of mathematics and science.
January 18, 2001 14:01 g65-ch5 Sheet number 9 Page number 327 cyan magenta yellow black

5.2 The Indefinite Integral; Integral Curves and Direction Fields 327

The integral of f(x) with respect to x is equal to F (x) plus a constant.

The differential symbol, dx, in the differentiation and antidifferentiation operations


d
[ ] and [ ] dx
dx
serves to identify the independent variable. If an independent variable other than x is used,
say t, then the notation must be adjusted appropriately. Thus,
d
[F (t)] = f(t) and f(t) dt = F (t) + C
dt
are equivalent statements.

Example 1

derivative equivalent
formula integration formula
d 3
[x ] = 3x 2 3x 2 dx = x 3 + C
dx
d 1 1
[√ x ] = dx = √ x + C
dx 2√ x 2√ x
d
[tan t] = sec 2 t sec 2 t dt = tan t + C
dt
d 3/2
[u ] = 32 u1/2 3 1/2
u du = u 3/2 + C
du 2

For simplicity, the dx is sometimes absorbed into the integrand. For example,
1 dx can be written as dx
1 dx
dx can be written as
x2 x2
The integral sign and differential serve as delimiters,
 flanking the integrand on the left and
right, respectively. In particular, we do not write dxf(x) when we intend f(x) dx.
••••••••••••••••••••••••••••••••••••••
Integration is essentially educated guesswork—given the derivative f of a function F ,
INTEGRATION FORMULAS one tries to guess what the function F is. However, many basic integration formulas can be
obtained directly from their companion differentiation formulas. Some of the most important
are given in Table 5.2.1.

Example 2 The second integration formula in Table 5.2.1 will be easier to remember if
you express it in words:

To integrate a power of x (other than −1), add 1 to the exponent and divide by the new
exponent.

Here are some examples:


x3
x 2 dx = +C r=2
3
x4
x 3 dx = +C r=3
4
1 x −5+1 1
dx = x −5 dx = +C =− 4 +C r = −5
x 5 −5 + 1 4x
√ x 2 +1
1
1 3 √
x dx = x 2 dx = 1 + C = 23 x 2 + C = 23 ( x )3 + C r= 1
2
2
+ 1
January 18, 2001 14:01 g65-ch5 Sheet number 10 Page number 328 cyan magenta yellow black

328 Integration

Table 5.2.1

differentiation formula integration formula


d
1. [x] = 1 dx = x + C
dx
r +1 x r +1
2.
d
dx [ rx + 1 ] = x r (r ≠ –1) x r dx =
r+1
+ C (r ≠ –1)

d
3. [sin x] = cos x cos x dx = sin x + C
dx
d
4. [–cos x] = sin x sin x dx = –cos x + C
dx

5. d [tan x] = sec 2 x sec 2 x dx = tan x + C


dx

6. d [–cot x] = csc 2 x csc 2 x dx = –cot x + C


dx

7. d [sec x] = sec x tan x sec x tan x dx = sec x + C


dx

8. d [–csc x] = csc x cot x csc x cot x dx = –csc x + C


dx

It is clear that this pattern does not fit the case of


1
dx = x −1 dx
x
since blind adherence to the pattern formula with r = −1 would lead to division by zero.
We will resolve this missing case in Chapter 7. 
••••••••••••••••••••••••••••••••••••••
Our first properties of antiderivatives follow directly from the simple constant factor, sum,
PROPERTIES OF THE INDEFINITE and difference rules for derivatives.
INTEGRAL

5.2.3 THEOREM. Suppose that F (x) and G(x) are antiderivatives of f(x) and g(x),
respectively, and that c is a constant. Then:
(a) A constant factor can be moved through an integral sign; that is,

cf(x) dx = cF (x) + C

(b) An antiderivative of a sum is the sum of the antiderivatives; that is,

[f(x) + g(x)] dx = F (x) + G(x) + C

(c) An antiderivative of a difference is the difference of the antiderivatives; that is,

[f(x) − g(x)] dx = F (x) − G(x) + C

Proof. In general, to establish the validity of an equation of the form


h(x) dx = H (x) + C
one must show that
d
[H (x)] = h(x)
dx
January 18, 2001 14:01 g65-ch5 Sheet number 11 Page number 329 cyan magenta yellow black

5.2 The Indefinite Integral; Integral Curves and Direction Fields 329

We are given that F (x) and G(x) are antiderivatives of f(x) and g(x), respectively, so we
know that
d d
[F (x)] = f(x) and [G(x)] = g(x)
dx dx
Thus,
d d
[cF (x)] = c [F (x)] = cf(x)
dx dx
d d d
[F (x) + G(x)] = [F (x)] + [G(x)] = f(x) + g(x)
dx dx dx
d d d
[F (x) − G(x)] = [F (x)] − [G(x)] = f(x) − g(x)
dx dx dx
which proves the three statements of the theorem.

In practice, the results of Theorem 5.2.3 are summarized by the following formulas:

cf(x) dx = c f(x) dx (4)

[f(x) + g(x)] dx = f(x) dx + g(x) dx (5)

[f(x) − g(x)] dx = f(x) dx − g(x) dx (6)

However, these equations must be applied carefully to avoid errors and unnecessary
complexities arising from the constants of integration. For example, if you were to use (4)
to integrate 0x by writing
 2 
x
0x dx = 0 x dx = 0 +C =0
2
then you will have erroneously lost the constant of integration, and if you use (4) to integrate
2x by writing
 2 
x
2x dx = 2 x dx = 2 + C = x 2 + 2C
2
then you will have an unnecessarily complicated form of the arbitrary constant. Similarly,
if you use (5) to integrate 1 + x by writing
 2 
x x2
(1 + x) dx = 1 dx + x dx = (x + C1 ) + + C2 = x + + C 1 + C2
2 2
then you will have two arbitrary constants when one will suffice. These three kinds of prob-
lems are caused by introducing constants of integration too soon and can be avoided by in-
serting the constant of integration in the final result, rather than in intermediate compuations.

Example 3 Evaluate

(a) 4 cos x dx (b) (x + x 2 ) dx

Solution (a). Since F (x) = sin x is an antiderivative for f(x) = cos x (Table 5.2.1), we
obtain
4 cos x dx = 4 cos x dx = 4 sin x + C

(4)
January 18, 2001 14:01 g65-ch5 Sheet number 12 Page number 330 cyan magenta yellow black

330 Integration

Solution (b). From Table 5.2.1 we obtain


x2 x3
(x + x 2 ) dx = x dx + x 2 dx = + +C 
2 3
(5)

Parts (b) and (c) of Theorem 5.2.3 can be extended to more than two functions, which
in combination with part (a) results in the following general formula:

[c1 f1 (x) + c2 f2 (x) + · · · + cn fn (x)] dx


(7)
= c1 f1 (x) dx + c2 f2 (x) dx + · · · + cn fn (x) dx

Example 4

(3x 6 − 2x 2 + 7x + 1) dx = 3 x 6 dx − 2 x 2 dx + 7 x dx + 1 dx

3x 7 2x 3 7x 2
=− + +x+C 
7 3 2
Sometimes it is useful to rewrite an integrand in a different form before performing the
integration.

Example 5 Evaluate
cos x t 2 − 2t 4
(a) dx (b) dt
sin2 x t4

Solution (a).
cos x 1 cos x
2
dx = dx = csc x cot x dx = − csc x + C
sin x sin x sin x
Formula 8 in Table 5.2.1

Solution (b).
 
t 2 − 2t 4 1
dt = −2 dt = (t −2 − 2) dt
t4 t2
t −1 1
= − 2t + C = − − 2t + C 
−1 t
••••••••••••••••••••••••••••••••••••••
Graphs of antiderivatives of a function f are called integral curves of f . We know from
INTEGRAL CURVES Theorem 5.2.2 that if y = F (x) is any integral curve of f(x), then all other integral curves
are vertical translations of this curve, since they have equations of the form y = F (x) + C.
For example, y = 13 x 3 is one integral curve for f(x) = x 2 , so all the other integral curves
have equations of the form y = 13 x 3 + C; conversely, the graph of any equation of this form
is an integral curve (Figure 5.2.1).
In many problems one is interested in finding a function whose derivative satisfies spec-
ified conditions. The following example illustrates a geometric problem of this type.

Example 6 Suppose that a point moves along some unknown curve y = f(x) in the
xy-plane in such a way that at each point (x, y) on the curve, the tangent line has slope x 2 .
Find an equation for the curve given that it passes through the point (2, 1).

Solution. We know that dy/dx = x 2 , so


y= x 2 dx = 13 x 3 + C
January 18, 2001 14:01 g65-ch5 Sheet number 13 Page number 331 cyan magenta yellow black

5.2 The Indefinite Integral; Integral Curves and Direction Fields 331

Since the curve passes through (2, 1), a specific value for C can be found by using the fact
that y = 1 if x = 2. Substituting these values in the above equation yields
1 = 13 (23 ) + C or C = − 53
so the curve is y = 13 x 3 − 53 . 

Observe that in this example the requirement that the unknown curve pass through the
point (2, 1) enabled us to determine a specific value for the constant of integration, thereby
isolating the single integral curve y = 13 x 3 − 53 from the family y = 13 x 3 + C (Figure 5.2.2).

y y

4 4

3
C=2
C=1 (2, 1)
C=0 x x
-3 3 -3 3
C = –1

C = –2

-3

-4 -4

1 1 5
y= 3
x3 + C y= 3
x3 – 3

Figure 5.2.1 Figure 5.2.2

••••••••••••••••••••••••••••••••••••••
We will now consider another way of looking at integration that will be useful in our later
INTEGRATION FROM THE work. Suppose that f(x) is a known function and we are interested in finding a function
VIEWPOINT OF DIFFERENTIAL
EQUATIONS F (x) such that y = F (x) satisfies the equation
dy
= f(x) (8)
dx
The solutions of this equation are the antiderivatives of f(x), and we know that these can
be obtained by integrating f(x). For example, the solutions of the equation
dy
= x2 (9)
dx
are
x3
y= x 2 dx = +C
3
Equation (8) is called a differential equation because it involves a derivative of an
unknown function. Differential equations are different from the kinds of equations we have
encountered so far in that the unknown is a function and not a number as in an equation
such as x 2 + 5x − 6 = 0.
Sometimes we will not be interested in finding all of the solutions of (8), but rather we
will want only the solution whose integral curve passes through a specified point (x0 , y0 ).
For example, in Example 6 we solved (9) for the integral curve that passed through the point
(2, 1).
For simplicity, it is common in the study of differential equations to denote a solution
of dy/dx = f(x) as y(x) rather than F (x), as earlier. With this notation, the problem of
finding a function y(x) whose derivative is f(x) and whose integral curve passes through
January 18, 2001 14:01 g65-ch5 Sheet number 14 Page number 332 cyan magenta yellow black

332 Integration

the point (x0 , y0 ) is expressed as

dy
= f(x), y(x0 ) = y0 (10)
dx
This is called an initial-value problem, and the requirement that y(x0 ) = y0 is called the
initial condition for the problem.

Example 7 Solve the initial-value problem


dy
= cos x, y(0) = 1
dx
Solution. The solution of the differential equation is
y= cos x dx = sin x + C (11)

The initial condition y(0) = 1 implies that y = 1 if x = 0; substituting these values in (11)
yields
1 = sin(0) + C or C=1
Thus, the solution of the initial-value problem is y = sin x + 1. 
••••••••••••••••••••••••••••••••••••••
If we interpret dy/dx as the slope of a tangent line, then at a point (x, y) on an integral curve
DIRECTION FIELDS of the equation dy/dx = f(x), the slope of the tangent line is f(x). What is interesting
about this is that the slopes of the tangent lines to the integral curves can be obtained without
actually solving the differential equation. For example, if
dy 
= x2 + 1
dx
then we know without solving√the equation √ that at the point where x = 1 the tangent line
to an integral curve has slope 12 + 1 = 2; √ and more generally, at a point where x = a,
the tangent line to an integral curve has slope a 2 + 1.
A geometric description of the integral curves of a differential equation dy/dx = f(x)
can be obtained by choosing a rectangular grid of points in the xy-plane, calculating the
slopes of the tangent lines to the integral curves at the gridpoints, and drawing small por-
tions of the tangent lines at those points. The resulting picture, which is called a direc-
tion field or slope field for the equation, shows the “direction” of the integral curves at the
gridpoints. With sufficiently many gridpoints it is often possible to visualize the integral
curves themselves; for example, Figure 5.2.3a shows a direction field for the differential
equation dy/dx = x 2 , and Figure 5.2.3b shows that same field with the integral curves

y y
5 5
4 4
3 3
2 2
1 1
x x
-5 -4 -3 -2 -1 1 2 3 4 5 -5 -4 -3 -2 -1 1 2 3 4 5
-1 -1
-2 -2
-3 -3
-4 -4
-5 -5

Direction field for dy/dx = x2 Direction field with integral curves

(a) (b)
Figure 5.2.3
January 18, 2001 14:01 g65-ch5 Sheet number 15 Page number 333 cyan magenta yellow black

5.2 The Indefinite Integral; Integral Curves and Direction Fields 333

imposed on it—the more gridpoints that are used, the more completely the direction field
reveals the shape of the integral curves. However, the amount of computation can be consid-
erable, so computers are usually used when direction fields with many gridpoints are needed.

EXERCISE SET 5.2 Graphing Calculator


••••••••••••••••••••••••••••••••••••••••••••••••••••••••••••••••••••••••••••••••••••••••••••••••••••••••••••••
 2
1. In each part, confirm that the formula is correct, and state a 13. x(1 + x 3 ) dx 14. 2 + y2 dy
corresponding integration formula.
d  x 15. x 1/3 (2 − x)2 dx 16. (1 + x 2 )(2 − x) dx
(a) [ 1 + x2 ] = 
dx 1 + x2
d 1 x 5 + 2x 2 − 1 1 − 2t 3
(b) sin(1 + x 3 ) = x 2 cos(1 + x 3 ) 17. dx 18. dt
dx 3 x4 t3
2. In each part, confirm that the stated formula is correct by
19. [4 sin x + 2 cos x] dx
differentiating.
(a) x sin x dx = sin x − x cos x + C 20. [4 sec2 x + csc x cot x] dx
dx x
(b)   =  +C 21. sec x(sec x + tan x) dx 22. sec x(tan x + cos x) dx
2 3/2 1 − x2
1−x
sec θ dy
In Exercises 3–6, find the derivative and state a corresponding 23. dθ 24.
cos θ csc y
integration formula. 
sin x 2
 25. dx 26. φ+ dφ
d  3 d x cos2 x sin2 φ
3. [ x + 5] 4.
dx dx x 2 + 3 sin 2x
27. [1 + sin2 θ csc θ] dθ 28. dx
d √ d cos x
5. [sin(2 x )] 6. [sin x − x cos x]
dx dx 29. Evaluate the integral
In Exercises 7 and 8, evaluate the integral by rewriting the 1
dx
integrand appropriately, if required, and then apply Formula 1 + sin x
2 in Table 5.2.1.
by multiplying the numerator and denominator by an ap-
8 5/7
√ 3 propriate expression.
7. (a) x dx (b) x dx (c) x x dx
30. Use the double-angle formula cos 2x = 2 cos2 x −1 to eval-

3 2 1 uate the integral
8. (a) x dx (b) dx (c) x −7/8 dx
x6 1
dx
1 + cos 2x
In Exercises 9–12, evaluate each integral by applying Theo-
31. (a) Use a graphing utility to generate a slope field for the
rem 5.2.3 and Formula 2 in Table 5.2.1 appropriately.
differential equation dy/dx = x in the region
1 −5 ≤ x ≤ 5 and −5 ≤ y ≤ 5.
9. (a) dx (b) (u3 − 2u + 7) du (b) Graph some representative integral curves of the func-
2x 3
tion f(x) = x.
10. (x 2/3 − 4x −1/5 + 4) dx (c) Find an equation for the integral curve that passes
through the point (4, 7).

11. (x −3 + x − 3x 1/4 + x 2 ) dx 32. (a) Use a graphing utility to generate
√ a slope field for the
  differential equation dy/dx = x in the region
7 √ √ 0 ≤ x ≤ 10 and −5 ≤ y ≤ 5.
12. − y + 4 y dy
3
y 3/4 (b) Graph some √ representative integral curves of the func-
tion f(x) = x for x > 0.
In Exercises 13–28, evaluate the integral, and check your an- (c) Find an equation for the
swer by differentiating.  integral curve that passes
through the point 4, 103
.
January 18, 2001 14:01 g65-ch5 Sheet number 16 Page number 334 cyan magenta yellow black

334 Integration

33. Use a graphing utility to generate some representative inte- tained at 25 ◦ C and 85 ◦ C, respectively. Assume that an x-
gral curves of the function f(x) = 5x 4 − sec2 x over the axis is chosen as in the accompanying figure and that the
interval (−π/2, π/2). temperature T (x) satisfies the equation
34. Use a graphing utility to generate some representative in- d 2T
=0
tegral curves of the function f(x) = (x 3 − 1)/x 2 over the dx 2
interval (0, 5). Find T (x) for 0 ≤ x ≤ 50.
35. Suppose that a point moves along a curve y = f(x) in the
xy-plane in such a way that at each point (x, y) on the curve
25°C 85°C x
the tangent line has slope − sin x. Find an equation for the
curve, given that it passes through the point (0, 2). 0 50 Figure Ex-44
36. Suppose that a point moves along a curve y = f(x) in the
xy-plane in such a way that at each point (x, y) on the curve 45. (a) Show that
the tangent line has slope (x + 1)2 . Find an equation for the
curve, given that it passes through the point (−2, 8). F (x) = 16 (3x + 4)2 and G(x) = 32 x 2 + 4x
differ by a constant by showing that they are antideriva-
In Exercises 37 and 38, solve the initial-value problems. tives of the same function.
(b) Find the constant C such that F (x) − G(x) = C by
dy √ evaluating F (x) and G(x) at some point x0 .
37. (a) = 3 x, y(1) = 2
dx (c) Check your answer in part (b) by simplifying the ex-
π 1
(b)
dy
= sin t + 1, y = pression F (x) − G(x) algebraically.
dt 3 2 46. Follow the directions of Exercise 45 with
dy x+1
(c) = √ , y(1) = 0 x2 5
dx x F (x) = 2 and G(x) = − 2
x +5 x +5
dy 1
38. (a) = , y(1) = 0
dx (2x)3 In Exercises 47 and 48, use a trigonometric identity to help
dy π evaluate the integral.
(b) = sec2 t − sin t, y =1
dt 4
dy √
(c) = x 2 x 3 , y(0) = 0 47. tan2 x dx 48. cot2 x dx
dx
39. Find
√ the general form of a function whose second
√ derivative 49. Use the identities cos 2θ = 1 − 2 sin2 θ = 2 cos2 θ − 1 to
is x. [Hint: Solve the equation f  (x) = x for f(x) by help evaluate the integrals
integrating both sides twice.]
(a) sin2 (x /2) dx (b) cos2 (x /2) dx
40. Find a function f such that f  (x) = x + cos x and such
that f(0) = 1 and f (0) = 2. [Hint: Integrate both sides of 50. Let F and G be the functions defined piecewise by
the equation twice.]  
x, x>0 x + 2, x>0
F (x) = and G(x) =
In Exercises 41–43, find an equation of the curve that satisfies −x, x<0 −x + 3, x<0
the given conditions. (a) Show that F and G have the same derivative.
(b) Show that G(x) = F (x) + C for any constant C.
41. At each point (x, y) on the curve the slope is 2x + 1; the (c) Do parts (a) and (b) violate Theorem 5.2.2? Explain.
curve passes through the point (−3, 0).
51. The speed of sound in air at 0 ◦ C (or 273 K on the Kelvin
42. At each point (x, y) on the curve the slope equals the square scale) is 1087 ft/s, but the speed v increases as the temper-
of the distance between the point and the y-axis; the point ature T rises. Experimentation has shown that the rate of
(−1, 2) is on the curve. change of v with respect to T is
43. At each point (x, y) on the curve, y satisfies the condition
dv 1087 −1/2
d 2 y /dx 2 = 6x; the line y = 5 − 3x is tangent to the curve = √ T
at the point where x = 1. dT 2 273
44. Suppose that a uniform metal rod 50 cm long is insulated where v is in feet per second and T is in kelvins (K). Find
laterally, and the temperatures at the exposed ends are main- a formula that expresses v as a function of T .
January 18, 2001 14:01 g65-ch5 Sheet number 17 Page number 335 cyan magenta yellow black

5.3 Integration by Substitution 335

5.3 INTEGRATION BY SUBSTITUTION

In this section we will study a technique, called substitution, that can often be used to
transform complicated integration problems into simpler ones.

••••••••••••••••••••••••••••••••••••••
The method of substitution can be motivated by examining the chain rule from the viewpoint
u-SUBSTITUTION of antidifferentiation. For this purpose, suppose that F is an antiderivative of f and that
g is a differentiable function. The chain rule implies that the derivative of F (g(x)) can be
expressed as
d
[F (g(x))] = F  (g(x))g  (x)
dx
which we can write in integral form as

F  (g(x))g  (x) dx = F (g(x)) + C (1)


or since F is an antiderivative of f ,

f(g(x))g  (x) dx = F (g(x)) + C (2)

For our purposes it will be useful to let u = g(x) and to write du/dx = g  (x) in the differ-
ential form du = g  (x) dx. With this notation (1) can be expressed as

f(u) du = F (u) + C (3)

The process of evaluating an integral of form (2) by converting it into form (3) with the
substitution
u = g(x) and du = g  (x) dx
is called the method of u-substitution. Here our emphasis is not on the interpretation of
the expression du = g  (x) dx as a function of dx as was done in Section 3.8. Instead, the
differential notation serves primarily as a useful “bookkeeping” device for the method of
u-substitution. The following example illustrates how the method works.

Example 1 Evaluate (x 2 + 1)50 · 2x dx.

Solution. If we let u = x 2 + 1, then du/dx = 2x, which implies that du = 2x dx. Thus,
the given integral can be written as
u51 (x 2 + 1)51
(x 2 + 1)50 · 2x dx = u50 du = +C = +C 
51 51
It is important to realize that in the method of u-substitution you have control over the
choice of u, but once you make that choice you have no control over the resulting expres-
sion for du. Thus, in the last example we chose u = x 2 + 1 but du = 2x dx was computed.
Fortunately, our choice of u, combined with the computed du, worked out perfectly to pro-
duce an integral involving u that was easy to evaluate. However, in general, the method of
u-substitution will fail if the chosen u and the computed du cannot be used to produce an inte-
grand in which no expressions involving x remain, or if you cannot evaluate the resulting in-
tegral. Thus, for example, the substitution u = x 2 , du = 2x dx will not work for the integral

2x sin x 4 dx
because this substitution results in the integral

sin u2 du
which still cannot be evaluated in terms of familiar functions.
January 18, 2001 14:01 g65-ch5 Sheet number 18 Page number 336 cyan magenta yellow black

336 Integration

In general, there are no hard and fast rules for choosing u, and in some problems no choice
of u will work. In such cases other methods need to be used, some of which will be discussed
later. Making appropriate choices for u will come with experience, but you may find the
following guidelines, combined with a mastery of the basic integrals in Table 5.2.1, helpful.

Step 1. Look for some composition f(g(x)) within the integrand for which
the substitution
u = g(x), du = g  (x)
produces an integral that is expressed entirely in terms of u and du.
This may or may not be possible.
Step 2. If you are successful in Step 1, then try to evaluate the resulting inte-
gral in terms of u. Again, this may or may not be possible.
Step 3. If you are successful in Step 2, then replace u by g(x) to express your
final answer in terms of x.
Step 4. If you are unsuccessful in reaching or completing Step 2, then it may
either be that the integrand in the original integral has no antideriva-
tive that can be expressed in terms of familiar functions, or that the
integral is tricky and you have not yet hit on the right substitution.
This state of affairs makes integration challenging, since one often
does not know whether a failure of the substitution method is due to
the nature of the integral or to a lack of cleverness on the part of the
problem solver.

••••••••••••••••••••••••••••••••••••••
In the simplest cases, it is unnecessary to consider Step 1(b) or 1(c). The easiest substitutions
EASY TO RECOGNIZE occur when the integrand is the derivative of a known function, except for a constant added
SUBSTITUTIONS
to or subtracted from the independent variable.

Example 2

sin(x + 9) dx = sin u du = − cos u + C = − cos(x + 9) + C

u=x+9
du = 1 · dx = dx

u24 (x − 8)24
(x − 8)23 dx = u23 du = +C = +C 
24 24
u=x−8
du = 1 · dx = dx

Another easy u-substitution occurs when the integrand is the derivative of a known
function, except for a constant that multiplies or divides the independent variable. The
following example illustrates two ways to evaluate such integrals.

Example 3 Evaluate cos 5x dx.

Solution.
1 1 1 1
cos 5x dx = (cos u) · du = cos u du = sin u + C = sin 5x + C
5 5 5 5
u = 5x
du = 5 dx or dx = 1
5 du
January 18, 2001 14:01 g65-ch5 Sheet number 19 Page number 337 cyan magenta yellow black

5.3 Integration by Substitution 337

Alternative Solution. There is a variation of the preceding method that some people
prefer. The substitution u = 5x requires du = 5 dx. If there were a factor of 5 in the inte-
grand, then we could group the 5 and dx together to form the du required by the substitution.
Since there is no factor of 5, we will insert one and compensate by putting a factor of 15 in
front of the integral. The computations are as follows:
1 1 1 1
cos 5x dx = cos 5x · 5 dx = cos u du = sin u + C = sin 5x + C 
5 5 5 5
u = 5x
du = 5 dx

More generally, if the integrand is a composition of the form f(ax + b), where f(x) is
an easy to integrate function, then the substitution u = ax + b, du = a dx will work.

Example 4
 −4
dx 3 du 3 3 1
1 5 = =3 u−5 du = − u−4 + C = − x−8 +C 
x−8 u5 4 4 3
3

u = 31 x − 8
du = 3 dx or dx = 3 du
1

With the help of Theorem 5.2.3, a complicated integral can sometimes be computed by
expressing it as a sum of simpler integrals.

Example 5
 
1 dx 1
+ sec πx dx =
2
+ sec2 πx dx = − + sec2 πx dx
x2 x2 x
1 1
=− + sec2 u du
x π
u = πx
du = π dx or dx = 1
π du

1 1 1 1
=− + tan u + C = − + tan πx + C 
x π x π

The next three examples illustrate Step 1(a) when the composition involves nonlinear
functions.

Example 6 Evaluate sin2 x cos x dx.

Solution. If we let u = sin x, then


du
= cos x, so du = cos x dx
dx
Thus,
u3 sin3 x
sin2 x cos x dx = u2 du = +C = +C 
3 3

cos x
Example 7 Evaluate √ dx.
x

Solution. If we let u = x, then
du 1 1 1
= √ , so du = √ dx or 2 du = √ dx
dx 2 x 2 x x
January 18, 2001 14:01 g65-ch5 Sheet number 20 Page number 338 cyan magenta yellow black

338 Integration

Thus,

cos x √
√ dx = 2 cos u du = 2 cos u du = 2 sin u + C = 2 sin x+C 
x


3
Example 8 Evaluate t 4 3 − 5t 5 dt.

Solution.

3 1 √ 1
t 4 3 − 5t 5 dt = − 3
u du = − u1/3 du
25 25
u = 3 − 5t 5
du = −25t 4 dt or − 25
1
du = t 4 dt

1 u4/3 3  4/3
=− +C =− 3 − 5t 5 +C 
25 4/3 100
••••••••••••••••••••••••••••••••••••••
The next two examples illustrate Steps 1(b) and 1(c), respectively.
LESS APPARENT SUBSTITUTIONS

Example 9 Evaluate x 2 x − 1 dx.

Solution. Let
u=x−1 so that du = dx (4)
From the first equality in (4)
x 2 = (u + 1)2 = u2 + 2u + 1
so that
√ √
x 2 x − 1 dx = (u2 + 2u + 1) u du = (u5/2 + 2u3/2 + u1/2 ) du

= 27 u7/2 + 54 u5/2 + 23 u3/2 + C

= 27 (x − 1)7/2 + 45 (x − 1)5/2 + 23 (x − 1)3/2 + C 

Example 10 Evaluate cos3 x dx.

Solution. The only compositions in the integrand that suggest themselves are
cos3 x = (cos x)3 and cos2 x = (cos x)2
However, neither the substitution u = cos x nor the substitution u = cos2 x work (verify).
Following the suggestion in Step 1(c), we write

cos3 x dx = cos2 x cos x dx

and solve the equation du = cos x dx for u = sin x. Since sin2 x + cos2 x = 1, we then
have
cos3 x dx = cos2 x cos x dx = (1 − sin2 x) cos x dx = (1 − u2 ) du

u3 1
=u− + C = sin x − sin3 x + C 
3 3
January 18, 2001 14:01 g65-ch5 Sheet number 21 Page number 339 cyan magenta yellow black

5.3 Integration by Substitution 339

••••••••••••••••••••••••••••••••••••••
The advent of computer algebra systems has made it possible to evaluate many kinds of
INTEGRATION USING COMPUTER integrals that would be laborious to evaluate by hand. For example, Derive, running on a
ALGEBRA SYSTEMS
handheld calculator, evaluated the integral
5x 2 3(x + 1)2/3 (5x 2 − 6x + 9)
dx = +C
(1 + x)1/3 8
in about a second. The computer algebra system Mathematica, running on a personal com-
puter, required even less time to evaluate this same integral. However, just as one would
not want to rely on a calculator to compute 2 + 2, so one would not want to use a CAS to
integrate a simple function such as f(x) = x 2 . Thus, even if you have a CAS, you will want
to develop a reasonable level of competence in evaluating basic integrals. Moreover, the
mathematical techniques that we will introduce for evaluating basic integrals are precisely
the techniques that computer algebra systems use to evaluate more complicated integrals.

• FOR THE READER.


• If you have a CAS, use it to calculate the integrals in the examples








of this section. If your CAS produces a form of the answer that is different from the one





in the text, then confirm algebraically that the two answers agree. Your CAS has various





commands for simplifying answers. Explore the effect of using the CAS to simplify the



• expressions it produces for the integrals.

EXERCISE SET 5.3 Graphing Calculator C CAS


••••••••••••••••••••••••••••••••••••••••••••••••••••••••••••••••••••••••••••••••••••••••••••••••••••••••••••••

In Exercises 1–4, evaluate the integrals by making the indi- 4. (a) x 2 1 + x dx; u = 1 + x
cated substitutions.
(b) [csc(sin x)]2 cos x dx; u = sin x
 23
1. (a) 2x x 2 + 1 dx; u = x 2 + 1
(c) sin(x − π) dx; u = x − π
(b) cos3 x sin x dx; u = cos x 5x 4
(d) dx; u = x 5 + 1
√ (x 5 + 1)2
1 √
(c) √ sin x dx; u = x
x In Exercises 5–30, evaluate the integrals by making appro-
3x dx priate substitutions.
(d)  ; u = 4x 2 + 5
4x 2 + 5  3
5. x 2 − x 2 dx 6. (3x − 1)5 dx
2. (a) sec (4x + 1) dx; u = 4x + 1
2

 7. cos 8x dx 8. sin 3x dx
(b) y 1 + 2y 2 dy; u = 1 + 2y 2
√ 9. sec 4x tan 4x dx 10. sec2 5x dx
(c) sin πθ cos πθ dθ; u = sin πθ
 x
11. t 7t 2 + 12 dt 12.  dx
(d) (2x + 7)(x + 7x + 3)
2 4/5
dx; u = x + 7x + 3
2 4 − 5x 2
x2 1
13.  dx 14. dx
3. (a) cot x csc x dx; u = cot x
2
x3 + 1 (1 − 3x)2
x
(b) (1 + sin t)9 cos t dt; u = 1 + sin t 15. dx 16. x cos(3x 2 ) dx
(4x + 1)3
2

sin(5/x) sec2 ( x)
(c) cos 2x dx; u = 2x 17. dx 18. √ dx
x2 x

(d) x sec2 x 2 dx; u = x 2 19. x 2 sec2 (x 3 ) dx 20. cos3 2t sin 2t dt


January 18, 2001 14:01 g65-ch5 Sheet number 22 Page number 340 cyan magenta yellow black

340 Integration

sin 2θ 
21. sin5 3t cos 3t dt 22. dθ 35. (a) Evaluate the integral sin x cos x dx by two methods:
(5 + cos 2θ)3 first by letting u = sin x, then by letting u = cos x.
√ (b) Explain why the two apparently different answers ob-
23. cos 4θ 2 − sin 4θ dθ 24. tan3 5x sec2 5x dx
tained in part (a) are really equivalent.

25. sec3 2x tan 2x dx 26. [sin(sin θ)] cos θ dθ 36. (a) Evaluate (5x − 1)2 dx by two methods: first square
and integrate, then let u = 5x − 1.
√ y dy (b) Explain why the two apparently different answers ob-
27. x x − 3 dx 28. 
y+1 tained in part (a) are really equivalent.

29. sin3 2θ dθ In Exercises 37 and 38, solve the initial-value problems.

30. sec4 3θ dθ dy √
37. = 3x + 1; y(1) = 5
[Hint: Apply Step 1(c) and a trigonometric identity.] dx
dy
38. = 6 − 5 sin 2x; y(0) = 3
In Exercises 31–33, evaluate the integrals assuming that n is dx
a positive integer and b = 0. 39. Find a function f such that the slope of the
√ tangent line at
a point (x, y) on the curve y = f(x) is 3x + 1, and the
√ curve passes through the point (0, 1).
(a + bx)n dx a + bx dx
n
31. 32.
40. Use a graphing utility
√ to generate some typical integral
33. sin (a + bx) cos(a + bx) dx
n curves of f(x) = x / x 2 + 1 over the interval (−5, 5).
41. A population of frogs is estimated to be 100,000 at the be-
C 34. Use a CAS to check the answers you obtained in Exercises ginning of the year 2000. Suppose that the rate of growth of
31–33. If the answer produced by the CAS does not match the population p(t) (in thousands) after t years is p  (t) =
yours, show that the two answers are equivalent. [Sugges- (4 + 0.15t)3/2 . Estimate the projected population at the be-
tion: Mathematica users may find it helpful to apply the ginning of the year 2005.
Simplify command to the answer.]

5.4 SIGMA NOTATION; AREA AS A LIMIT

Recall from the informal discussion in Section 5.1 that if a function f is continuous
and nonnegative on an interval [a, b], then the “rectangle method ” provides us with
one approach to computing the area between the graph of f and the interval [a, b].
We begin this section with a discussion of a notation to represent lengthy sums in a
concise form. Then we will discuss the rectangle method in more detail, both as a
means for defining and for computing the area under a curve. In particular, we will
show that such an area may be interpreted as a limit.
••••••••••••••••••••••••••••••••••••••
The notation we will discuss is called sigma notation or summation notation because it
SIGMA NOTATION uses the uppercase Greek letter  (sigma) to denote various kinds of sums. To illustrate
how this notation works, consider the sum
12 + 2 2 + 3 2 + 4 2 + 5 2
in which each term is of the form k 2 , where k is one of the integers from 1 to 5. In sigma
notation this sum can be written as
5
k2
k=1

which is read “the summation of k 2 , where k runs from 1 to 5.” The notation tells us to
form the sum of the terms that result when we substitute successive integers for k in the
expression k 2 , starting with k = 1 and ending with k = 5.
January 18, 2001 14:01 g65-ch5 Sheet number 23 Page number 341 cyan magenta yellow black

5.4 Sigma Notation; Area as a Limit 341

More generally, if f(k) is a function of k, and if m and n are integers such that m ≤ n,
then
n
f(k) (1)
Ending k=m
value of k
n denotes the sum of the terms that result when we substitute successive integers for k, starting
This tells
us to add
f(k) with k = m and ending with k = n (Figure 5.4.1).
Starting
k=m
Example 1
value of k
Figure 5.4.1 8
k 3 = 43 + 53 + 63 + 73 + 83
k=4
5
2k = 2 · 1 + 2 · 2 + 2 · 3 + 2 · 4 + 2 · 5 = 2 + 4 + 6 + 8 + 10
k=1
5
(2k + 1) = 1 + 3 + 5 + 7 + 9 + 11
k=0
5
(−1)k (2k + 1) = 1 − 3 + 5 − 7 + 9 − 11
k=0
1
k 3 = (−3)3 + (−2)3 + (−1)3 + 03 + 13 = −27 − 8 − 1 + 0 + 1
k=−3
3  
kπ π 2π 3π
k sin = sin + 2 sin + 3 sin 
k=1
5 5 5 5
The numbers m and n in (1) are called, respectively, the lower and upper limits of
summation; and the letter k is called the index of summation. It is not essential to use k as
the index of summation; any letter not reserved for another purpose will do. For example,
6 6 6
1 1 1
, , and
i=1
i j =1
j n=1
n

all denote the sum


1 1 1 1 1
1+ + + + +
2 3 4 5 6
If the upper and lower limits of summation are the same, then the “sum” in (1) reduces
to a single term. For example,
2 1
1 1 1
k 3 = 23 and = =
k=2 i=1
i+2 1+2 3
In the sums
5 2
2 and x3
i=1 j =0

the expression to the right of the  sign does not involve the index of summation. In such
cases, we take all the terms in the sum to be the same, with one term for each allowable
value of the summation index. Thus,
5 2
2=2+2+2+2+2 and x3 = x3 + x3 + x3
i=1 j =0
January 18, 2001 14:01 g65-ch5 Sheet number 24 Page number 342 cyan magenta yellow black

342 Integration

••••••••••••••••••••••••••••••••••••••
A sum can be written in more than one way using sigma notation with different limits of
CHANGING THE LIMITS OF summation and correspondingly different summands. For example,
SUMMATION
5 4 7
2i = 2 + 4 + 6 + 8 + 10 = (2j + 2) = (2k − 4)
i=1 j =0 k=3
On occasion we will want to change the sigma notation for a given sum to a sigma notation
with different limits of summation.

Example 2 Express
7
5k−2
k=3
in sigma notation so that the lower limit of summation is 0 rather than 3.

Solution.
7
5k−2 = 51 + 52 + 53 + 54 + 55
k=3
= 50+1 + 51+1 + 52+1 + 53+1 + 54+1
4 4
= = 5k+1 
j =0 k=0

••••••••••••••••••••••••••••••••••••••
When stating general properties of sums it is often convenient to use a subscripted letter
PROPERTIES OF SUMS such as ak in place of the function notation f(k). For example,
5 5 3
ak = a1 + a2 + a3 + a4 + a5 = aj = ak+2
k=1 j =1 k=−1
n n n−2
ak = a1 + a2 + · · · + an = aj = ak+2
k=1 j =1 k=−1
Our first properties provide some basic rules for manipulating sums.

5.4.1 THEOREM.
n n
(a) cak = c ak (if c does not depend on k)
k=1 k=1
n n n
(b) (ak + bk ) = ak + bk
k=1 k=1 k=1
n n n
(c) (ak − bk ) = ak − bk
k=1 k=1 k=1

We will prove parts (a) and (b) and leave part (c) as an exercise.

Proof (a).
n n
cak = ca1 + ca2 + · · · + can = c(a1 + a2 + · · · + an ) = c ak
k=1 k=1

Proof (b).
n
(ak + bk ) = (a1 + b1 ) + (a2 + b2 ) + · · · + (an + bn )
k=1
n n
= (a1 + a2 + · · · + an ) + (b1 + b2 + · · · + bn ) = ak + bk
k=1 k=1
January 18, 2001 14:01 g65-ch5 Sheet number 25 Page number 343 cyan magenta yellow black

5.4 Sigma Notation; Area as a Limit 343

Restating Theorem 5.4.1 in words:

(a) A constant factor can be moved through a sigma sign.


(b) Sigma distributes across sums.
(c) Sigma distributes across differences.

••••••••••••••••••••••••••••••••••••••

SUMMATION FORMULAS 5.4.2 THEOREM.


n
n(n + 1)
(a) k = 1 + 2 + ··· + n =
k=1
2
n
n(n + 1)(2n + 1)
(b) k 2 = 12 + 22 + · · · + n2 =
k=1
6

n
n(n + 1) 2
(c) k 3 = 13 + 23 + · · · + n3 =
k=1
2

We will prove parts (a) and (b) and leave part (c) as an exercise.

Proof (a). Writing


n
k
k=1
two ways, with summands in increasing order and in decreasing order, and then adding, we
obtain
n
k= 1 + 2 + 3 + · · · + (n − 2) + (n − 1) + n
k=1
n
k= n + (n − 1) + (n − 2) + · · · + 3 + 2 + 1
k=1

n
2 k = (n + 1) + (n + 1) + (n + 1) + · · · + (n + 1) + (n + 1) + (n + 1)
k=1

= n(n + 1)

Thus,
n
n(n + 1)
k=
k=1
2

Proof (b). Note that


(k + 1)3 − k 3 = k 3 + 3k 2 + 3k + 1 − k 3 = 3k 2 + 3k + 1
So,
n n
[(k + 1)3 − k 3 ] = (3k 2 + 3k + 1) (2)
k=1 k=1
Writing out the left side of (2) with the index running down from k = n to k = 1, we have
n
[(k + 1)3 − k 3 ] = [(n + 1)3 − n3 ] + · · · + [43 − 33 ] + [33 − 23 ] + [23 − 13 ]
k=1
= (n + 1)3 − 1 (3)
Combining (3) and (2), and expanding the right side of (2) by using Theorem 5.4.1 and part
January 18, 2001 14:01 g65-ch5 Sheet number 26 Page number 344 cyan magenta yellow black

344 Integration

(a) of this theorem yields


n n n
(n + 1)3 − 1 = 3 k2 + 3 k+ 1
k=1 k=1 k=1
n
n(n + 1)
=3 k2 + 3 +n
k=1
2
So,
n
n(n + 1)
3 k 2 = [(n + 1)3 − 1] − 3 −n
2
n
k=1

= (n + 1)3 − 3(n + 1) − (n + 1)
2
n+1
= [2(n + 1)2 − 3n − 2]
2
n+1 2 n(n + 1)(2n + 1)
= [2n + n] =
2 2
Thus,
n
n(n + 1)(2n + 1)
k2 =
k=1
6

• REMARK.
• The sum in (3) is an example of a telescoping sum, since the cancellation of







each of the two parts of an interior summand with parts of its neighboring summands allows



• the entire sum to collapse like a telescope.

30
Example 3 Evaluate k(k + 1).
k=1
Solution.
30 30 30 30
k(k + 1) = (k 2 + k) = k2 + k
k=1 k=1 k=1 k=1

30(31)(61) 30(31)
= + = 9920 Theorem 5.4.2(a), (b) 
6 2
In formulas such as
n
n(n + 1) n(n + 1)
k= or 1 + 2 + ··· + n =
k=1
2 2
the left side of the equality is said to express the sum in open form and the right side is said
to express it in closed form. The open form indicates the summands and the closed form is
an explicit formula for the sum.
n
Example 4 Express (3 + k)2 in closed form.
k=1

Solution.
n
(3 + k)2 = 42 + 52 + · · · + (3 + n)2
k=1
= [12 + 22 + 33 + 42 + 52 + · · · + (3 + n)2 ] − [12 + 22 + 32 ]
 
3+n
= k 2 − 14
k=1

(3 + n)(4 + n)(7 + 2n) 1


= − 14 = (73n + 21n2 + 2n3 ) 
6 6
January 18, 2001 14:01 g65-ch5 Sheet number 27 Page number 345 cyan magenta yellow black

5.4 Sigma Notation; Area as a Limit 345

• FOR THE READER.


• Your numerical calculating utility probably provides some way of







evaluating sums that can be expressed in sigma notation. Check your documentation to find






out how to do this, and then use your utility to confirm that the numerical result obtained





in Example 3 is correct. If you have access to a CAS, it provides some method for finding





closed forms for sums such as those in Theorem 5.4.2. Use your CAS to confirm the formulas





in that theorem, and then find closed forms for





n n
k4 k5





and


• k=1 k=1

••••••••••••••••••••••••••••••••••••••
Suppose that f is a continuous function that is nonnegative on an interval [a, b], and let R
A DEFINITION OF AREA denote the region that is bounded below by the x-axis, bounded on the sides by the vertical
lines x = a and x = b, and bounded above by the curve y = f(x) (Figure 5.4.2). Recall
y from the informal discussion in Section 5.1 that the “rectangle method” provides us with
y = f (x) one approach to computing the area between the graph of f and the interval [a, b]. Our
goal now is to define formally what we mean by the area of R. We will work from the
definition of the area of a rectangle as the product of its length and width. Define the area
of a region decomposed into a finite collection of rectangles to be the sum of the areas of
those rectangles. To define the area of the region R, we will use these definitions and the
x rectangle method of Section 5.1. The basic idea is as follows:
a b
• Divide the interval [a, b] into n equal subintervals.
Figure 5.4.2
• Over each subinterval construct a rectangle whose height is the value of f at any point
in the subinterval.
• The union of these rectangles forms a region Rn whose area can be regarded as an
approximation to the “area” A of the region R.
• Repeat the process using more and more subdivisions.
• Define the area of R to be the “limit” of the areas of the approximating regions Rn , as
n is made larger and larger without bound. We can express this idea symbolically as
A = area(R) = lim [area(Rn )] (4)
n → +⬁

• REMARK.
• There is a difference in interpretation between writing limn → +⬁ and writing







limx → +⬁ , where n represents a positive integer and x has no such restriction. Equation (4)

y •




should be interpreted to mean that by choosing the positive integer n sufficiently large, we





can make area(Rn ) as close to A as desired. Later we will study limits of the type limn → +⬁





in detail, but for now suffice it to say that the computational techniques we have used for



• limits of type limx → +⬁ will also work for limn → +⬁ .

To make all of this more precise, it will be helpful to capture this procedure in mathemat-
x ical notation. For this purpose, suppose that we divide the interval [a, b] into n subintervals
a x1 x2 . . . xn − 1 b
by inserting n − 1 equally spaced points between a and b, say
Figure 5.4.3 x1 , x2 , . . . , xn−1
(Figure 5.4.3). Each of these subintervals has width (b − a)/n, which it is customary to
y denote by
b−a
)x =
n
In each subinterval we need to choose an x-value at which to evaluate the function f to
determine the height of a rectangle over the interval. If we denote those x-values by
x x1∗ , x2∗ , . . . , xn∗
a x1* x2* ... xn* b
(Figure 5.4.4), then the areas of the rectangles constructed over these intervals will be
Figure 5.4.4 f(x1∗ ))x, f(x2∗ ))x, . . . , f(xn∗ ))x
January 18, 2001 14:01 g65-ch5 Sheet number 28 Page number 346 cyan magenta yellow black

346 Integration

y
(Figure 5.4.5), and the total area of the region Rn will be
area(Rn ) = f(x1∗ ))x + f(x2∗ ))x + · · · + f(xn∗ ))x
With this notation (4) can be expressed as
n
A = lim f(xk∗ ))x
n → +⬁
x k=1
a b
a n gle

which suggests the following definition of the area of the region R.


re c t

5.4.3 DEFINITION (Area Under a Curve). If the function f is continuous on [a, b] and
k th

if f(x) ≥ 0 for all x in [a, b], then the area under the curve y = f(x) over the interval
[a, b] is defined by
n
A = lim f(xk∗ ))x (5)
n → +⬁
k=1
f (xk*)

In (5) the values of x1∗ , x2∗ , . . . , xn∗ may be chosen in many different ways, so it is con-
Area = f (x*k )∆x ceivable that different choices of these values might produce different values of A. Were this
to happen, then Definition 5.4.3 would not be an acceptable definition of area. Fortunately,
xk* this does not happen; it is proved in advanced courses that when f is continuous (as we
∆x have assumed), the same value of A results no matter how the xk∗ are chosen. In practice
Figure 5.4.5 they are chosen in some systematic fashion, some common choices being:

• The left endpoint of each subinterval.


• The right endpoint of each subinterval.
• The midpoint of each subinterval.

If, as shown in Figure 5.4.6, the subinterval [a, b] is divided by x1 , x2 , x3 , . . . , xn−1 into n
equal parts each of length )x = (b − a)/n, and if we let x0 = a and xn = b, then
xk = a + k)x for k = 0, 1, 2, . . . , n
Thus,
xk∗ = xk−1 = a + (k − 1))x Left endpoint (6)

xk∗ = xk = a + k)x Right endpoint (7)


 
xk∗ = 12 (xk−1 + xk ) = a + k − 12 )x Midpoint (8)

a a + ∆x a + 2∆ x a + 3∆ x ... a + (n – 1)∆ x b = a + n∆ x

∆x ∆x ∆x ... ∆x
Figure 5.4.6 x0 x1 x2 x3 xn – 1 xn

••••••••••••••••••••••••••••••••••••••
We would expect from Definition 5.4.3 that for each of the choices (6), (7), and (8), the sum
NUMERICAL APPROXIMATIONS OF
n n
AREA
f(xk∗ ))x = )x f(xk∗ ) = )x[f(x1∗ ) + f(x2∗ ) + · · · + f(xn∗ )] (9)
k=1 k=1

would yield a good approximation to the area A, provided n is a large positive integer.
According to which of these three options is used in choosing the xk∗ , we refer to Formula
(9) as the left endpoint approximation, the right endpoint approximation, or the midpoint
approximation of the exact area (Figure 5.4.7).
January 18, 2001 14:01 g65-ch5 Sheet number 29 Page number 347 cyan magenta yellow black

5.4 Sigma Notation; Area as a Limit 347

y y y

x x x
a b a b a b
Left endpoint approximation Right endpoint approximation Midpoint approximation

(a) (b) (c)


Figure 5.4.7

y
Example 5 Find the left endpoint, right endpoint, and midpoint approximations of the
9 area under the curve y = 9 − x 2 over the interval [0, 3] with n = 10, n = 20, and n = 50
(Figure 5.4.8).
y = 9 – x2

Solution. Details of the computations for the case n = 10 are shown to six decimal places
in Table 5.4.1 and the results of all computations are given in Table 5.4.2. 
x
Table 5.4.1
0 3
n = 10, ∆x = (b – a) / n = (3 – 0) /10 = 0.3
Figure 5.4.8
left endpoint right endpoint midpoint
approximation approximation approximation

k x*k 9 – (x*k )2 x*k 9 – (x*k )2 x*k 9 – (x*k )2

1 0.0 9.000000 0.3 8.910000 0.15 8.977500


2 0.3 8.910000 0.6 8.640000 0.45 8.797500
3 0.6 8.640000 0.9 8.190000 0.75 8.437500
4 0.9 8.190000 1.2 7.560000 1.05 7.897500
5 1.2 7.560000 1.5 6.750000 1.35 7.177500
6 1.5 6.750000 1.8 5.760000 1.65 6.277500
7 1.8 5.760000 2.1 4.590000 1.95 5.197500
8 2.1 4.590000 2.4 3.240000 2.25 3.937500
9 2.4 3.240000 2.7 1.710000 2.55 2.497500
10 2.7 1.710000 3.0 0.000000 2.85 0.877500
64.350000 55.350000 60.075000

n (0.3)(64.350000) (0.3)(55.350000) (0.3)(60.075000)


∆x f (x*k ) = 19.305000 = 16.605000 = 18.022500
k=1

Table 5.4.2

left endpoint right endpoint midpoint


n approximation approximation approximation

10 19.305000 16.605000 18.022500


20 18.663750 17.313750 18.005625
50 18.268200 17.728200 18.000900
January 18, 2001 14:01 g65-ch5 Sheet number 30 Page number 348 cyan magenta yellow black

348 Integration

• REMARK.
• We will show below that the exact area under y = 9−x 2 over the interval [0, 3]







is 18 (i.e., 18 square units), so that in the preceding example the midpoint approximation






is more accurate than either of the endpoint approximations. This can also be seen geo-





metrically from the approximating rectangles: Since the graph of y = 9 − x 2 is decreasing





over the interval [0, 3], each left endpoint approximation overestimates the area, each right





endpoint approximation underestimates the area, and each midpoint approximation falls





between the overestimate and the underestimate (Figure 5.4.9). This is consistent with the





values in Table 5.4.2. Later in the text we will investigate the error that results when an area



• is approximated by the midpoint rule.

y y y
9 9 9

x x x
0 3 0 3 0 3

The left endpoint The right endpoint The midpoint approximation


approximation approximation is better than the endpoint
overestimates underestimates approximations.
the area. the area.

Figure 5.4.9

••••••••••••••••••••••••••••••••••••••
Although numerical approximations of area are useful, we will often wish to compute the
COMPUTING THE EXACT VALUE OF exact value of some area. In certain cases this can be done by explicitly evaluating the limit
AN AREA
in Definition 5.4.3.

Example 6 Use Definition 5.4.3 with xk∗ as the right endpoint of each subinterval to find
the area between the graph of f(x) = x 2 and the interval [0, 1].

Solution. We have
b−a 1−0 1
)x = = =
n n n
and from (7)
k
xk∗ = a + k)x =
n
so that
n n  2 n
k 1 1 n 2
f (xk∗ ))x = (xk∗ )2 )x
= = 3 k
k=1 k=1 k=1
n n n k=1

1 n(n + 1)(2n + 1) 1 1 1
= = + +
n3 6 3 2n 6n2
Therefore,
n  
1 1 1 1
A = lim f(xk∗ ))x = lim + + =
n → +⬁
k=1
n → +⬁ 3 2n 6n2 3
(Note that this conclusion agrees with the numerical evidence we collected in Table 5.1.2.)

January 18, 2001 14:01 g65-ch5 Sheet number 31 Page number 349 cyan magenta yellow black

5.4 Sigma Notation; Area as a Limit 349

In the solution to Example 6 we made use of one of the “closed form” summation
formulas from Theorem 5.4.2. The next result collects some consequences of Theorem
5.4.2 that can facilitate computations of area using Definition 5.4.3.

5.4.4 THEOREM.
n n
1 1 1
(a) lim 1=1 (b) lim k=
n → +⬁ n n → + ⬁ n2 2
k=1 k=1
n n
1 1 1 1
(c) lim k3 = (d) lim k3 =
n → + ⬁ n3 3 n → + ⬁ n4 4
k=1 k=1

The proof of Theorem 5.4.4 is left as an exercise for the reader.

Example 7 Use Definition 5.4.3 with xk∗ as the midpoint of each subinterval to find the
area under the parabola y = f(x) = 9 − x 2 and over the interval [0, 3].

Solution. Each subinterval will have length


b−a 3−0 3
)x = = =
n n n
and from (8)
    
∗ 1 1 3
xk = a + k − )x = k −
2 2 n
Thus,
    
 
1 2 3 2 3
f(xk∗ ))x = [9 − (xk∗ )2 ])x
= 9− k−
2 n n
     
1 9 3
= 9− k −k+
2
2
4 n n
27 27 2 27 27
= − 3k + 3k − 3
n n n 4n
and
n n  
27 27 2 27 27
f(xk∗ ))x = − 3k + 3k − 3
k=1 k=1
n n n 4n
    
1 n 1 n 2 1 1 n
1 1 n
= 27 1− 3 k + k − 2 1
n k=1 n k=1 n n2 k=1
4n n k=1

Therefore,
n
A = lim f(xk∗ ))x
n → +⬁
k=1
    
n n n n
1 1
1 1 1 1
= lim 27 1− 3
k2 + k − 2 1
n → +⬁ n k=1
k=1
nn n2 k=1
4n n k=1

1 1
= 27 1 − + 0 · − 0 · 1 = 18
3 2
where we used Theorem 5.4.4 to compute the limits as n → +⬁ of the expressions
n
1
k j −1 for j = 1, 2, 3 
nj k=1
January 18, 2001 14:01 g65-ch5 Sheet number 32 Page number 350 cyan magenta yellow black

350 Integration

••••••••••••••••••••••••••••••••••••••
In Definition 5.4.3 we assumed that f is continuous and nonnegative on the interval [a, b].
NET SIGNED AREA If f is continuous and attains both positive and negative values on [a, b], then the limit
n
lim f(xk∗ ))x (10)
n → +⬁
k=1

no longer represents the area between the curve y = f(x) and the interval [a, b] on the
x-axis; rather, it represents a difference of areas—the area of the region that is above the
interval [a, b] and below the curve y = f(x) minus the area of the region that is below the
interval [a, b] and above the curve y = f(x). We call this the net signed area between the
AI AIII
graph of y = f(x) and the interval [a, b]. For example, in Figure 5.4.10a, the net signed
a b area between the curve y = f(x) and the interval [a, b] is
AII
(AI + AIII ) − AII = area above [a, b] − area below [a, b]
(a) To explain why the limit in (10) represents this net signed area, let us subdivide the interval
[a, b] in Figure 5.4.10a into n equal subintervals and examine the terms in the sum
n
f(xk∗ ))x (11)
a b k=1

If f(xk∗ ) is positive, then the product f(xk∗ ))x represents the area of the rectangle with
height f(xk∗ ) and base )x (the biege rectangles in Figure 5.4.10b). However, if f(xk∗ ) is
(b) negative, then the product f(xk∗ ))x is the negative of the area of the rectangle with height
Figure 5.4.10 |f(xk∗ )| and base )x (the green rectangles in Figure 5.4.10b). Thus, (11) represents the total
area of the beige rectangles minus the total area of the green rectangles. As n increases, the
pink rectangles fill out the regions with areas AI and AIII and the green rectangles fill out
the region with area AII , which explains why the limit in (10) represents the signed area
between y = f(x) and the interval [a, b]. We formalize this in the following definition.

5.4.5 DEFINITION (Net Signed Area). If the function f is continuous on [a, b], then the
net signed area A between y = f(x) and the interval [a, b] is defined by
n
A = lim f(xk∗ ))x
n → +⬁
k=1

As with Definition 5.4.3, it can be shown that for a continuous function this limit always
exists (independently of the choice of the numbers xk∗ ). The net signed area between the
curve y = f(x) and [a, b] can be positive, negative, or zero; it is positive when there is more
area above the interval than below, negative when there is more area below than above, and
zero when the areas above and below are equal.

Example 8 Use Definition 5.4.5 with xk∗ as the left endpoint of each subinterval to find
the net signed area between the graph of y = f(x) = x − 1 and the interval [0, 2].

Solution. Each subinterval will have length


b−a 2−0 2
)x = = =
n n n
and from (6)
 
2
xk∗ = a + (k − 1))x = (k − 1)
n
Thus,
     
2 2 4 4 2
f (xk∗ ))x = (xk∗ − 1))x = (k − 1) −1 = 2
k− 2 −
n n n n n
January 18, 2001 14:01 g65-ch5 Sheet number 33 Page number 351 cyan magenta yellow black

5.4 Sigma Notation; Area as a Limit 351

and
n n   
4 4 2
f(xk∗ ))x = − k−
n2n2 n
     
k=1 k=1

y 1 n 4 1 n 1 n
=4 k − 1 −2 1
n2 k=1 n n k=1 n k=1
y=x–1
Therefore,
A1
      
n n n n
1 4 1 1
x A = lim f(xk∗ ))x = lim 4 k − 1 −2 1
n → +⬁ n → +⬁ n2 n n n
 
1 2 k=1 k=1 k=1 k=1
A2 1
=4 − 0 · 1 − 2(1) = 0
2
Since the net signed area is zero, the area A1 below the graph of f and above the interval
[0, 2] must equal the area A2 above the graph of f and below the interval [0, 2]. This
Figure 5.4.11 conclusion agrees with the graph of f shown in Figure 5.4.11. 

EXERCISE SET 5.4 C CAS


••••••••••••••••••••••••••••••••••••••••••••••••••••••••••••••••••••••••••••••••••••••••••••••••••••••••••••••

1. Evaluate
3 6 1 In Exercises 11–16, use Theorem 5.4.2 to evaluate the sums,
(a) k3 (b) (3j − 1) (c) (i 2 − i) and check your answers using the summation feature of a cal-
k=1 j =2 i=−4 culating utility.
5 4 6
(d) 1 (e) (−2)k (f ) sin nπ. 100 100 20

n=0 k=0 n=1 11. k 12. (7k + 1) 13. k2


k=1 k=1 k=1
2. Evaluate 20 30 6
4 5 20
kπ j 2 14. k2 15. k(k−2)(k+2) 16. (k−k 3 )
(a) k sin (b) (−1) (c) π
k=1
2 j =0 i=7
k=4 k=1 k=1

5 6
√ 10 In Exercises 17–20, express the sums in closed form.
(d) 2m+1 (e) n (f ) cos kπ.
n n−1 n−1
m=3 n=1 k=0 3k k2 k3
17. 18. 19.
k=1
n k=1
n k=1
n2
In Exercises 3–8, write each expression in sigma notation,
n  
but do not evaluate. 5 2k
20. −
n n
3. 1 + 2 + 3 + · · · + 10 k=1
C 21. For each of the sums that you obtained in Exercises 17–20,
4. 3 · 1 + 3 · 2 + 3 · 3 + · · · + 3 · 20
use a CAS to check your answer. If the answer produced
5. 2 + 4 + 6 + 8 + · · · + 20 6. 1+3+5+7+· · ·+15 by the CAS does not match your own, show that the two
7. 1 − 3 + 5 − 7 + 9 − 11 8. 1 − 1
2
+ 1
3
− 1
4
+ 1
5
answers are equivalent.
n
9. (a) Express the sum of the even integers from 2 to 100 in
22. Solve the equation k = 465.
sigma notation. k=1
(b) Express the sum of the odd integers from 1 to 99 in sig-
ma notation. In Exercises 23–26, express the function of n in closed form
and then find the limit.
10. Express in sigma notation.
(a) a1 − a2 + a3 − a4 + a5 1 + 2 + 3 + ··· + n
(b) −b0 + b1 − b2 + b3 − b4 + b5 23. lim
n → +⬁ n2
(c) a0 + a1 x + a2 x 2 + · · · + an x n 1 + 2 + 32 + · · · + n2
2 2

(d) a 5 + a 4 b + a 3 b2 + a 2 b3 + ab4 + b5 24. lim


n → +⬁ n3
January 18, 2001 14:01 g65-ch5 Sheet number 34 Page number 352 cyan magenta yellow black

352 Integration

n n−1
5k 2k 2 44. The function f and interval [a, b] of Exercise 38.
25. lim 26. lim
n → +⬁
k=1
n2 n → +⬁
k=1
n3 45. The function f and interval [a, b] of Exercise 39.
27. Express 1 + 2 + 2 + 2 + 2 + 2 in sigma notation with
2 3 4 5
46. The function f and interval [a, b] of Exercise 40.
(a) j = 0 as the lower limit of summation
(b) j = 1 as the lower limit of summation In Exercises 47 and 48, use Definition 5.4.3 with xk∗ as the
(c) j = 2 as the lower limit of summation. midpoint of each subinterval to find the area under the curve
28. Express y = f(x) over the interval [a, b].
9
k2k+4 47. The function f(x) = x 2 ; a = 0, b = 1
k=5
48. The function f(x) = x 2 ; a = −1, b = 1
in sigma notation with
(a) k = 1 as the lower limit of summation
In Exercises 49–52, use Definition 5.4.5 with xk∗ as the right
(b) k = 13 as the upper limit of summation.
endpoint of each subinterval to find the net signed area be-
In Exercises 29–32, divide the interval [a, b] into n = 4 tween the curve y = f(x) and the interval [a, b].
subintervals of equal length, and then compute
4 49. y = x; a = −1, b = 1. Verify your answer with a simple
f(xk∗ ))x geometric argument.
k=1
with xk∗ as (a) the left endpoint of each subinterval, (b) the 50. y = x; a = −1, b = 2. Verify your answer with a simple
midpoint of each subinterval, and (c) the right endpoint of geometric argument.
each subinterval. 51. y = x 2 − 1; a = 0, b = 2 52. y = x 3 ; a = −1, b = 1
53. Use Definition 5.4.3 with xk∗ as the left endpoint of each
29. f(x) = 3x + 1; a = 2, b = 6 subinterval to find the area under the graph of y = mx and
30. f(x) = 1/x; a = 1, b = 9 over the interval [a, b], where m > 0 and a ≥ 0.
31. f(x) = cos x; a = 0, b = π 54. Use Definition 5.4.5 with xk∗ as the right endpoint of each
32. f(x) = 2x − x ; a = −1, b = 3
2 subinterval to find the net signed area between the graph of
y = mx and the interval [a, b].
In Exercises 33–36, use a calculating utility with summa- 55. (a) Show that the area under the graph of y = x 3 and over
tion capabilities or a CAS to obtain an approximate value the interval [0, b] is b4 /4.
for the area between the curve and the specified interval with
(b) Find a formula for the area under y = x 3 over the in-
n = 10, 20, and 50 subintervals by using the (a) left end-
terval [a, b], where a ≥ 0.
point, (b) right endpoint, and (c) midpoint approximations. √
(If you do not have access to such a utility, then just do the 56. Find the area between the graph of y = x and the interval
case n = 10.) [0, 1]. [Hint: Use the result of Exercise 17 of Section 5.1.]
57. An artist wants to create a rough triangular design using
C 33. y = 1/x; [1, 2] C 34. y = 1/x 2 ; [1, 3] uniform square tiles glued edge to edge. She places n tiles
√ in a row to form the base of the triangle and then makes each
C 35. y = x; [0, 4] C 36. y = sin x; [0, π/2]
successive row two tiles shorter than the preceding row. Find
a formula for the number of tiles used in the design. [Hint:
In Exercises 37–42, use Definition 5.4.3 with xk∗ as the right Your answer will depend on whether n is even or odd.]
endpoint of each subinterval to find the area under the curve
58. An artist wants to create a sculpture by gluing together uni-
y = f(x) over the interval [a, b].
form spheres. She creates a rough rectangular base that has
50 spheres along one edge and 30 spheres along the other.
37. y = 12 x; a = 1, b = 4
She then creates successive layers by gluing spheres in the
38. y = 5 − x; a = 0, b = 5 grooves of the preceding layer. How many spheres will there
39. y = 9 − x 2 ; a = 0, b = 3 be in the sculpture?
40. y = 4 − 14 x 2 ; a = 0, b = 3 59. By writing out the sums, determine whether the following
41. y = x ; a = 2, b = 6
3 are valid identities.
  
42. y = 1 − x 3 ; a = −3, b = −1 n n
(a) fi (x) dx = fi (x) dx
In Exercises 43–46, use Definition 5.4.5 with xk∗ as the left i=1 i=1

endpoint of each subinterval to find the area under the curve   


y = f(x) over the interval [a, b].
n n
d d
(b) fi (x) = [fi (x)]
dx i=1 i=1
dx
43. The function f and interval [a, b] of Exercise 37.
January 18, 2001 14:01 g65-ch5 Sheet number 35 Page number 353 cyan magenta yellow black

5.5 The Definite Integral 353

60. Which of the following are valid identities?  61. Prove part (c) of Theorem 5.4.1.
n n n n n n
ai 62. Prove part (c) of Theorem 5.4.2. [Hint: Begin with the dif-
(a) ai bi = ai bi (b) = ai bi
b ference (k + 1)4 − k 4 and follow the steps used to prove part
i=1 i=1 i=1 i=1 i i=1 i=1
 2 (b) of the theorem.]
n n
(c) ai2 = ai 63. Prove Theorem 5.4.4.
i=1 i=1

5.5 THE DEFINITE INTEGRAL

In this section we will introduce the concept of a “definite integral,” which will link the
concept of area to other important concepts such as length, volume, density, probabil-
ity, and work.

••••••••••••••••••••••••••••••••••••••
In our definition of net signed area (Definition 5.4.5), we assumed that for each positive
RIEMANN SUMS AND THE number n, the interval [a, b] was subdivided into n subintervals of equal length to create
DEFINITE INTEGRAL
bases for the approximating rectangles. For some functions it may be more convenient to
use rectangles with different widths (see Exercise 33); however, if we are to “exhaust” an
y area with rectangles of different widths, then it is important that successive subdivisions be
y = f (x) constructed in such a way that the widths of the rectangles approach zero as n increases
(Figure 5.5.1). Thus, we must preclude the kind of situation that occurs in Figure 5.5.2 in
which the right half of the interval is never subdivided. If this kind of subdivision were
allowed, the error in the approximation would not approach zero as n increased.
A partition of the interval [a, b] is a collection of numbers
x
a b a = x0 < x1 < x2 < · · · < xn−1 < xn = b
Figure 5.5.1 that divides [a, b] into n subintervals of lengths
)x1 = x1 − x0 , )x2 = x2 − x1 , )x3 = x3 − x2 , . . . , )xn = xn − xn−1
The partition is said to be regular provided the subintervals all have the same length
y
y = f (x) b−a
)xk = )x =
n
For a regular partition, the widths of the approximating rectangles approach zero as n is
made large. Since this need not be the case for a general partition, we need some way to
x
measure the “size” of these widths. One approach is to let max )xk denote the largest of
a b the subinterval widths. The magnitude max )xk is called the mesh size of the partition. For
example, Figure 5.5.3 shows a partition of the interval [0, 6] into four subintervals with a
Figure 5.5.2 mesh size of 2.

∆ x1 ∆ x2 ∆ x3 ∆ x4
0 3 5 9 6
2 2 2

max ∆xk = ∆ x3 = 9
– 52 = 2
2

Figure 5.5.3

If we are to generalize Definition 5.4.5 so that it allows for unequal subinterval widths,
we must replace the constant length )x by the variable length )xk . When this is done the
sum
n n
f(xk∗ ))x is replaced by f(xk∗ ))xk
k=1 k=1
January 18, 2001 14:01 g65-ch5 Sheet number 36 Page number 354 cyan magenta yellow black

354 Integration

We also need to replace the expression n → +⬁ by an expression that guarantees us that


the lengths of all subintervals approach zero. We will use the expression max )xk → 0 for
this purpose. (Some writers use the symbol ) rather than max )xk for the mesh size of
the partition, in which case max )xk → 0 would be replaced by ) → 0.) Based on our
inituitive concept of area, we would then expect the net signed area A between the graph
of f and the interval [a, b] to satisfy the equation
n
A= lim f(xk∗ ))xk
max )xk → 0
k=1

(We will see in a moment that this is the case.) The limit that appears in this expression is
one of the fundamental concepts of integral calculus and forms the basis for the following
definition.

5.5.1 DEFINITION. A function f is said to be integrable on a finite closed interval


[a, b] if the limit
n
lim f(xk∗ ))xk
max )xk → 0
k=1

exists and does not depend on the choice of partitions or on the choice of the numbers
xk∗ in the subintervals. When this is the case we denote the limit by the symbol
b n
f(x) dx = lim f(xk∗ ))xk
a max)xk → 0
k=1

which is called the definite integral of f from a to b. The numbers a and b are called
the lower limit of integration and the upper limit of integration, respectively, and f(x)
is called the integrand.

The notation used for the definite integral deserves some comment. Historically, the
expression “f(x) dx” was interpreted to be the “infinitesimal area” of a rectangle with
height f(x) and “infinitesimal” width dx. By “summing” these  infinitesimal areas, the
entire area under the curve was obtained. The integral symbol “ ”is an “elongated s” that
was used to indicate this summation. For us, the integral symbol “ ” and the symbol “dx”
can serve as reminders that the definite integral is actually a limit of a summation as )xk → 0.

The sum that appears in Definition 5.5.1 is called a Riemann sum, and the definite integral


GEORG FRIEDRICH BERNHARD RIEMANN (1826–1866). German mathematician. Bernhard Riemann, as he
is commonly known, was the son of a Protestant minister. He received his elementary education from his father
and showed brilliance in arithmetic at an early age. In 1846 he enrolled at Göttingen University to study theology
and philology, but he soon transferred to mathematics. He studied physics under W. E. Weber and mathematics
under Karl Friedrich Gauss, whom some people consider to be the greatest mathematician who ever lived. In 1851
Riemann received his Ph.D. under Gauss, after which he remained at Göttingen to teach. In 1862, one month after
his marriage, Riemann suffered an attack of pleuritis, and for the remainder of his life was an extremely sick man.
He finally succumbed to tuberculosis in 1866 at age 39.
An interesting story surrounds Riemann’s work in geometry. For his introductory lecture prior to becoming
an associate professor, Riemann submitted three possible topics to Gauss. Gauss surprised Riemann by choosing
the topic Riemann liked the least, the foundations of geometry. The lecture was like a scene from a movie. The
old and failing Gauss, a giant in his day, watching intently as his brilliant and youthful protégé skillfully pieced
together portions of the old man’s own work into a complete and beautiful system. Gauss is said to have gasped
with delight as the lecture neared its end, and on the way home he marveled at his student’s brilliance. Gauss
died shortly thereafter. The results presented by Riemann that day eventually evolved into a fundamental tool that
Einstein used some 50 years later to develop relativity theory.
In addition to his work in geometry, Riemann made major contributions to the theory of complex functions
and mathematical physics. The notion of the definite integral, as it is presented in most basic calculus courses, is
due to him. Riemann’s early death was a great loss to mathematics, for his mathematical work was brilliant and
of fundamental importance.
January 18, 2001 14:01 g65-ch5 Sheet number 37 Page number 355 cyan magenta yellow black

5.5 The Definite Integral 355

is sometimes called the Riemann integral in honor of the German mathematician Bernhard
Riemann who formulated many of the basic concepts of integral calculus. (The reason
for the similarity in notation between the definite integral and the indefinite integral will
become clear in the next section, where we will establish a link between the two types of
“integration.”)
The limit that appears in Definition 5.5.1 is somewhat different from the kinds of limits
discussed in Chapter 2. Loosely phrased, the expression
n
lim f(xk∗ ))xk = L
max )xk → 0
k=1
is intended to convey the idea that we can force the Riemann sums to be as close as we
please to L, regardless of how the xk∗ are chosen, by making the mesh size of the partition
sufficiently small. Although it is possible to give a more formal definition of this limit, we
will simply rely on intuitive arguments when applying Definition 5.5.1.

Example 1 Use Definition 5.5.1 to show that if f(x) = C is a constant function, then
b
f(x) dx = C(b − a)
a

Solution. Since f(x) = C is constant, it follows that no matter how the xk∗ are chosen,
n n n
f(xk∗ ))xk = C)xk = C )xk = C(b − a)
k=1 k=1 k=1
Since every Riemann sum has the same value C(b − a), it follows that
n
lim
max )xk → 0
f(xk∗ ))xk = lim
max )xk → 0
C(b − a) = C(b − a) 
k=1

Note that in Definition 5.5.1, we do not assume that the function f is necessarily con-
tinuous on the interval [a, b].

Example 2 Define a function f on the interval [0, 1] by f(x) = 1 if 0 < x ≤ 1 and


f(0) = 0. Use Definition 5.5.1 to show that
1
f(x) dx = 1
0

Solution. We first note that since


lim f(x) = lim+ 1 = 1 = 0 = f(0)
x → 0+ x →0
f is not continuous on the interval [0, 1]. Consider any partition of [0, 1] and any choice of
the xk∗ corresponding to this partition. Then either x1∗ = 0 or it does not. If not, then
n n
f(xk∗ ))xk = )xk = 1
k=1 k=1
On the other hand, if x1∗ = 0, then f(x1∗ ) = f(0) = 0 and
n n n
f(xk∗ ))xk = )xk = −)x1 + )xk = 1 − )x1
k=1 k=2 k=1
In either case we see that the difference between the Riemann sum
n
f(xk∗ ))xk
k=1
and 1 is at most )x1 . Since )x1 approaches zero as max )xk → 0, it follows that
1
f(x) dx = 1 
0
January 18, 2001 14:01 g65-ch5 Sheet number 38 Page number 356 cyan magenta yellow black

356 Integration

Although Example 2 shows that a function does not have to be continuous on an interval
to be integrable on that interval, we will be interested primarily in the definite integrals of
continuous functions. Our earlier discussion of net signed area suggests that a function that
is continuous on an interval should also be integrable on that interval. This is the content of
the next result, which we state without proof.

5.5.2 THEOREM. If a function f is continuous on an interval [a, b], then f is inte-


grable on [a, b].

We can use Theorem 5.5.2 to clarify the connection between the definite integral and
net signed area. Suppose that f is a continuous function on an interval [a, b]. Recall that in
Section 5.4 we defined the net signed area A between the graph of f and the interval [a, b]
to be given by the limit
n
A = lim f(xk∗ ))x
n → +⬁
k=1
On the other hand, it follows from Theorem 5.5.2 and Definition 5.5.1 that we can use
regular partitions of [a, b] to compute the definite integral of f over [a, b] as the limit
b n
f(x) dx = lim f(xk∗ ))x
a n → +⬁
k=1
Since the two limits are the same, we conclude that
n b n
A = lim f(xk∗ ))x = f(x) dx = lim f(xk∗ ))xk
n → +⬁ a max )xk → 0
k=1 k=1
In other words, the definite integral of a continuous function f from a to b may always
be interpreted as the net signed area between the graph of f and the interval [a, b]. Of
course, if f is nonnegative, this is simply the area beneath the graph of f and above the
interval [a, b]. It follows that our area computations in Section 5.4 may be reformulated as
computations of particular definite integrals. For example, we showed that the area between
the graph of f(x) = 9 − x 2 and the interval [0, 3] is 18 square units. Equivalently, this
computation shows us that
3
(9 − x 2 ) dx = 18
0
Fortunately, there are often effective and efficient methods for evaluating definite integrals
that do not require the explicit evaluation of limits. (We will have more to say about this in
Section 5.6.) In the simplest cases, definite integrals can be calculated using formulas from
plane geometry to compute signed areas.

Example 3 Sketch the region whose area is represented by the definite integral, and
evaluate the integral using an appropriate formula from geometry.
4 2 1
(a) 2 dx (b) (x + 2) dx (c) 1 − x 2 dx
1 −1 0

Solution (a). The graph of the integrand is the horizontal line y = 2, so the region is a
rectangle of height 2 extending over the interval from 1 to 4 (Figure 5.5.4a). Thus,
4
2 dx = (area of rectangle) = 2(3) = 6
1

Solution (b). The graph of the integrand is the line y = x + 2, so the region is a trapezoid
whose base extends from x = −1 to x = 2 (Figure 5.5.4b). Thus,
2
(x + 2) dx = (area of trapezoid) = 12 (1 + 4)(3) = 15
2
−1
January 18, 2001 14:01 g65-ch5 Sheet number 39 Page number 357 cyan magenta yellow black

5.5 The Definite Integral 357



Solution (c). The graph of y = 1 − x 2 is the upper semicircle of radius 1, centered at
the origin, so the region is the right quarter-circle extending from x = 0 to x = 1 (Figure
5.5.4c). Thus,
1
π
1 − x 2 dx = (area of quarter-circle) = 14 π(12 ) = 
0 4

y y y
y = x+2
4 4

y = √1 – x 2
1
3 3
y=2
2 2

1 1
x x x
1 2 3 4 5 -2 -1 1 2 3 1

(a) (b) (c)


Figure 5.5.4

Example 4 Evaluate
y
2 1
(a) (x − 1) dx (b) (x − 1) dx
y=x–1 0 0

A1 Solution. The graph of y = x − 1 is shown in Figure 5.5.5, and we leave it for you to
x verify that the shaded triangular regions both have area 12 . Over the interval [0, 2] the net
1 2 signed area is A1 − A2 = 12 − 12 = 0, and over the interval [0, 1] the net signed area is
A2
−A2 = − 12 . Thus,
2 1
(x − 1) dx = 0 and (x − 1) dx = − 12
0 0
Figure 5.5.5 (Recall that in Example 8 of Section 5.4, we used Definition 5.4.5 to show that the net
signed area between the graph of y = x − 1 and the interval [0, 2] is 0.) 

••••••••••••••••••••••••••••••••••••••
It is assumed in Definition 5.5.1 that [a, b] is a finite closed interval with a < b, and hence
PROPERTIES OF THE DEFINITE the upper limit of integration in the definite integral is greater than the lower limit of integra-
INTEGRAL
tion. However, it will be convenient to extend this definition to allow for cases in which the
upper and lower limits of integration are equal or the lower limit of integration is greater than
the upper limit of integration. For this purpose we make the following special definitions.

5.5.3 DEFINITION.
(a) If a is in the domain of f , we define
a
f(x) dx = 0
a

(b) If f is integrable on [a, b], then we define


a b
f(x) dx = − f(x) dx
b a
January 18, 2001 14:01 g65-ch5 Sheet number 40 Page number 358 cyan magenta yellow black

358 Integration

y • REMARK.
• Part (a) of this definition is consistent with the intuitive idea that the area

y = f (x) •

between a point on the x-axis and a curve y = f(x) should be zero (Figure 5.5.6). Part










(b) of the definition is simply a useful convention; it states that interchanging the limits of



• integration reverses the sign of the integral.
x
Example 5
a
1
The area between (a) x 2 dx = 0
y = f (x) and a is zero. 1
0  1  π
Figure 5.5.6
(b) 1 − x 2 dx = − 1 − x 2 dx = − 
1 0 4
Example 3(c)

Because definite integrals are defined as limits, they inherit many of the properties of
limits. For example, we know that constants can be moved through limit signs and that the
limit of a sum or difference is the sum or difference of the limits. Thus, you should not be
surprised by the following theorem, which we state without formal proof.

5.5.4 THEOREM. If f and g are integrable on [a, b] and if c is a constant, then cf,
f + g, and f − g are integrable on [a, b] and
b b
(a) cf(x) dx = c f(x) dx
a a
b b b
(b) [f(x) + g(x)] dx = f(x) dx + g(x) dx
a a a
b b b
(c) [f(x) − g(x)] dx = f(x) dx − g(x) dx
a a a

Part (b) of this theorem can be extended to more than two functions. More precisely,
b
[f1 (x) + f2 (x) + · · · + fn (x)] dx
a
b b b
= f1 (x) dx + f2 (x) dx + · · · + fn (x) dx
a a a

Some properties of definite integrals can be motivated by interpreting the integral as an


y area. For example, if f is continuous and nonnegative on the interval [a, b], and if c is a
y = f (x) point between a and b, then the area under y = f(x) over the interval [a, b] can be split
into two parts and expressed as the area under the graph from a to c plus the area under the
graph from c to b (Figure 5.5.7), that is,
b c b
x f(x) dx = f(x) dx + f(x) dx
a c b a a c
Figure 5.5.7 This is a special case of the following theorem about definite integrals, which we state with-
out proof.

5.5.5 THEOREM. If f is integrable on a closed interval containing the three numbers


a, b, and c, then
b c b
f(x) dx = f(x) dx + f(x) dx
a a c

no matter how the numbers are ordered.


January 18, 2001 14:01 g65-ch5 Sheet number 41 Page number 359 cyan magenta yellow black

5.5 The Definite Integral 359

The following theorem, which we state without formal proof, can also be motivated by
interpreting definite integrals as areas.

5.5.6 THEOREM.
(a) If f is integrable on [a, b] and f(x) ≥ 0 for all x in [a, b], then
b
f(x) dx ≥ 0
a

(b) If f and g are integrable on [a, b] and f(x) ≥ g(x) for all x in [a, b], then
b b
f(x) dx ≥ g(x) dx
a a

y Geometrically, part (a) of this theorem states the obvious fact that if f is nonnegative on
y = f (x)
[a, b], then the net signed area between the graph of f and the interval [a, b] is also nonneg-
ative (Figure 5.5.8). Part (b) has its simplest interpretation when f and g are nonnegative
on [a, b], in which case the theorem states that if the graph of f does not go below the graph
of g, then the area under the graph of f is at least as large as the area under the graph of g
x
a b
(Figure 5.5.9).
Net signed area ≥ 0 • REMARK.



Part (b) of this theorem states that one can integrate both sides of the inequality
f(x) ≥ g(x) without altering the sense of the inequality. We also note that in the case where



Figure 5.5.8 •




• b > a, both parts of the theorem remain true if ≥ is replaced by ≤, >, or < throughout.

y Example 6 Evaluate
y = f (x) 1 
(5 − 3 1 − x 2 ) dx
0
y = g(x)
x Solution. From parts (a) and (c) of Theorem 5.5.4 we can write
a b 1  1 1  1 1
(5 − 3 1 − x 2 ) dx = 5 dx − 3 1 − x 2 dx = 5 dx − 3 1 − x 2 dx
Area under f ≥ area under g 0 0 0 0 0

Figure 5.5.9 The first integral can be interpreted as the area of a rectangle of height 5 and base 1, so its
value is 5, and from Example 3 the value of the second integral is π/4. Thus,
1  π 3π
(5 − 3 1 − x 2 ) dx = 5 − 3 =5− 
0 4 4
••••••••••••••••••••••••••••••••••••••
The problem of determining when functions with discontinuities are integrable is quite
DISCONTINUITIES AND complex and beyond the scope of this text. However, there are a few basic results about
INTEGRABILITY
integrability that are important to know; we begin with a definition.

5.5.7 DEFINITION. A function f that is defined on an interval I is said to be bounded


on I if there is a positive number M such that
y
y=M −M ≤ f(x) ≤ M
f
for all x in the interval I . Geometrically, this means that the graph of f over the interval
x
a b
I lies between the lines y = −M and y = M.

y = –M For example, a continuous function f is bounded on every finite closed interval because
the Extreme-Value Theorem (4.5.3) implies that f has an absolute maximum and an abso-
lute minimum on the interval; hence, its graph will lie between the line y = −M and
f is bounded on [a, b].
y = M, provided we make M large enough (Figure 5.5.10). In contrast, a function that has
Figure 5.5.10 a vertical asymptote inside of an interval is not bounded on that interval because its graph
January 18, 2001 14:01 g65-ch5 Sheet number 42 Page number 360 cyan magenta yellow black

360 Integration

f over the interval cannot be made to lie between the lines y = −M and y = M, no matter
how large we make the value of M (Figure 5.5.11).
The following theorem, which we state without proof, provides some facts about inte-
y=M
grability for functions with discontinuities.
a x
b
5.5.8 THEOREM. Let f be a function that is defined on the finite closed interval [a, b].
y = –M (a) If f has finitely many discontinuities in [a, b] but is bounded on [a, b], then f is
integrable on [a, b].
(b) If f is not bounded on [a, b], then f is not integrable on [a, b].

f is not bounded on [a, b]. • FOR THE READER.


• Sketch the graph of a function over the interval [0, 1] that has the




Figure 5.5.11 •
• properties stated in part (a) of this theorem.

EXERCISE SET 5.5


••••••••••••••••••••••••••••••••••••••••••••••••••••••••••••••••••••••••••••••••••••••••••••••••••••••••••••••

In Exercises 1–4, find the value of In Exercises 9 and 10, use Definition 5.5.1 to express the in-
n
tegrals as limits of Riemann sums. Do not try to evaluate the
(a) f(xk∗ ))xk (b) max )xk .
integrals.
k=1

2 1
1. f(x) = x + 1; a = 0, b = 4; n = 3; 9. (a) 2x dx (b)
x
dx
)x1 = 1, )x2 = 1, )x3 = 2; 1 0 x+1
x1∗ = 13 , x2∗ = 32 , x3∗ = 3 2 √ π/2
10. (a) x dx (b) (1 + cos x) dx
2. f(x) = cos x; a = 0, b = 2π; n = 4; 1 −π/2
)x1 = π/2, )x2 = 3π/4, )x3 = π/2, )x4 = π/4;
In Exercises 11–14, sketch the region whose signed area is
x1∗ = π/4, x2∗ = π, x3∗ = 3π/2.x4∗ = 7π/4
represented by the definite integral, and evaluate the integral
3. f(x) = 4 − x 2 ; a = −3, b = 4; n = 4; using an appropriate formula from geometry, where needed.
)x1 = 1, )x2 = 2, )x3 = 1, )x4 = 3;
−1
x1∗ = − 52 , x2∗ = −1, x3∗ = 14 , x4∗ = 3 3
11. (a) x dx (b) x dx
4. f(x) = x ; a = −3, b = 3; n = 4;
3
0 −2
)x1 = 2, )x2 = 1, )x3 = 1, )x4 = 2; 4 5
(c) x dx (d) x dx
x1∗ = −2, x2∗ = 0, x3∗ = 0, x4∗ = 2 −1 −5
2   1  
In Exercises 5–8, use the given values of a and b to express 12. (a) 1 − 12 x dx (b) 1 − 12 x dx
the following limits as definite integrals. (Do not evaluate the 0 −1
integrals.) 3   3 
(c) 1 − 21 x dx (d) 1 − 12 x dx
2 0
n
5. lim (xk∗ )2 )xk ; a = −1, b = 2 5 π
max )xk → 0
k=1
13. (a) 2 dx (b) cos x dx
0 0
n
2 1 
6. lim (xk∗ )3 )xk ; a = 1, b = 2 (c) |2x − 3| dx (d) 1 − x 2 dx
max )xk → 0
k=1 −1 −1
n
−5 π/3
7. lim 4xk∗ (1 − 3xk∗ ))xk ; a = −3, b = 3 14. (a) 6 dx (b) sin x dx
max )xk → 0
k=1 −10 −π/3
n 3 2
8. lim (sin2 xk∗ ))xk ; a = 0, b = π/2 (c) |x − 2| dx (d) 4 − x 2 dx
max )xk → 0 0 0
k=1
January 18, 2001 14:01 g65-ch5 Sheet number 43 Page number 361 cyan magenta yellow black

5.5 The Definite Integral 361

3
√ 4
15. Use the areas shown in the accompanying figure to find x x2
23. (a) dx (b) dx
b c
2 1−x 0 3 − cos x
(a) f(x) dx (b) f(x) dx −1
a b x4 2
x3 − 9
24. (a) √ dx (b) dx
c d
−3 3−x −2 |x| + 1
(c) f(x) dx (d) f(x) dx.
a a
In Exercises 25 and 26, evaluate the integrals by completing
y y = f (x) the square and applying appropriate formulas from geometry.
Area = 0.8 Area = 1.5
10  3 
x
25. 10x − x 2 dx 26. 6x − x 2 dx
a b c d 0 0

In Exercises 27 and 28, evaluate the limit over the interval


Area = 2.6
[a, b] by expressing it as a definite integral and applying an
Figure Ex-15
appropriate formula from geometry.

16. In each part, evaluate the integral, given that n


 27. lim (3xk∗ + 1))xk ; a = 0, b = 1
2x, x ≤ 1 max )xk → 0
k=1
f(x) = n 
2, x>1
1 1 28. lim 4 − (xk∗ )2 )xk ; a = −2, b = 2
max )xk → 0
(a) f(x) dx (b) f(x) dx k=1
0 −1 29. In each part, use Theorems 5.5.2 and 5.5.8 to determine
10 5
whether the function f is integrable on the interval [−1, 1].
(c) f(x) dx (d) f(x) dx (a) f(x) = cos x
1 1/2

2 x/|x|, x = 0
17. Find [f(x) + 2g(x)] dx if (b) f(x) =
−1 0, x=0
2 2 
f(x) dx = 5 and g(x) dx = −3 1/x 2 , x = 0
(c) f(x) =
−1 −1 0, x=0
4 
18. Find [3f(x) − g(x)] dx if sin 1/x, x = 0
1 (d) f(x) =
4 4 0, x=0
f(x) dx = 2 and g(x) dx = 10 30. It can be shown that every interval contains both rational and
1 1
5 irrational numbers. Accepting this to be so, do you believe
19. Find f(x) dx if that the function
1

1 if x is rational
1 5 f(x) =
f(x) dx = −2 and f(x) dx = 1 0 if x is irrational
0 0
−2 is integrable on a closed interval [a, b]? Explain your rea-
20. Find f(x) dx if soning.
3
1 3 31. It can be shown that the limit in Definition 5.5.1 has all of
f(x) dx = 2 and f(x) dx = −6 the limit properties stated in Theorem 2.2.2. Accepting this
−2 1
to be so, show that
b b
In Exercises 21 and 22, use Theorem 5.5.4 and appropriate
formulas from geometry to evaluate the integrals. (a) cf(x) dx = c f(x) dx
a a
b b b
1  3 (b) [f(x) + g(x)] dx = f(x) dx + g(x) dx
21. (a) (x + 2 1 − x 2 ) dx (b) (4 − 5x) dx a a a
0 −1
32. Find the smallest and largest values that the Riemann sum
0  2
22. (a) (2 + 9 − x 2 ) dx (b) (1 − 3|x|) dx 3
−3 −2 f(xk∗ ))xk
k=1
In Exercises 23 and 24, use Theorem 5.5.6 to determine can have on the interval [0, 4] if f(x) = x 2 − 3x + 4 and
whether the value of the integral is positive or negative. )x1 = 1, )x2 = 2, )x3 = 1.
January 18, 2001 14:01 g65-ch5 Sheet number 44 Page number 362 cyan magenta yellow black

362 Integration


33. The function f(x) = x is continuous on [0, 4] and there- for a < x ≤ b. Prove that
fore integrable on this interval. Evaluate b b
f(x) dx = g(x) dx
4 √ a a
x dx
0 [Hint: Write
by using Definition 5.5.1. Use subintervals of unequal length b b
given by the partition f(x) dx = [(f(x) − g(x)) + g(x)] dx
a a
0 < 4(1)2 /n2 < 4(2)2 /n2 < · · · < 4(n − 1)2 /n2 < 4 and use the result of Exercise 34 along with Theorem
and let xk∗ be the right endpoint of the kth subinterval. 5.5.4(b).]
34. Suppose that f is defined on the interval [a, b] and that 36. Define the function f by f(x) = 1/x, x = 0 and f(0) = 0.
f(x) = 0 for a < x ≤ b. Use Definition 5.5.1 to prove that It follows from Theorem 5.5.8(b) that f is not integrable
b on the interval [0, 1]. Prove this to be the case by applying
f(x) dx = 0 Definition 5.5.1. [Hint: Argue that no matter how small the
a
mesh size is for a partition of [0, 1], there will always be a
35. Suppose that g is a continuous function on the interval [a, b]
choice of x1∗ that will make the Riemann sum in Definition
and that f is a function defined on [a, b] with f(x) = g(x)
5.5.1 as large as we like.]

5.6 THE FUNDAMENTAL THEOREM OF CALCULUS

In this section we will establish two basic relationships between definite and indefinite
integrals that together constitute a result called the Fundamental Theorem of Calculus.
One part of this theorem will relate the rectangle and antiderivative methods for cal-
culating areas, and the second part will provide a powerful method for evaluating
definite integrals using antiderivatives.
••••••••••••••••••••••••••••••••••••••
As in earlier sections, let us begin by assuming that f is nonnegative and continuous on
THE FUNDAMENTAL THEOREM OF an interval [a, b], in which case the area A under the graph of f over the interval [a, b] is
CALCULUS
represented by the definite integral
y b
y = f (x) A= f(x) dx (1)
a
(Figure 5.6.1).
Recall that our discussion of the antiderivative method in Section 5.1 suggested that if
A A(x) is the area under the graph of f from a to x (Figure 5.6.2), then:
x
a b • A (x) = f(x)
• A(a) = 0 The area under the curve from a to a is the area above the single point a, and hence is zero.
Figure 5.6.1
• A(b) = A The area under the curve from a to b is A.

y
The formula A (x) = f(x) states that A(x) is an antiderivative of f(x), which implies that
y = f (x) every other antiderivative of f(x) on [a, b] can be obtained by adding a constant to A(x).
Accordingly, let
F (x) = A(x) + C
A(x)
be any antiderivative of f(x), and consider what happens when we subtract F (a) from
x
F (b):
a x b
F (b) − F (a) = [A(b) + C] − [A(a) + C] = A(b) − A(a) = A − 0 = A
Figure 5.6.2
Hence (1) can be expressed as
b
f(x) dx = F (b) − F (a)
a
January 18, 2001 14:01 g65-ch5 Sheet number 45 Page number 363 cyan magenta yellow black

5.6 The Fundamental Theorem of Calculus 363

In words, this equation states:

The definite integral can be evaluated by finding any antiderivative of the integrand and
then subtracting the value of this antiderivative at the lower limit of integration from its
value at the upper limit of integration.

Although our evidence for this result assumed that f is nonnegative on [a, b], this assump-
tion is not essential.

5.6.1 THEOREM (The Fundamental Theorem of Calculus, Part 1). If f is continuous on


[a, b] and F is any antiderivative of f on [a, b], then
b
f(x) dx = F (b) − F (a) (2)
a

Proof. Let x1 , x2 , . . . , xn−1 be any numbers in [a, b] such that


a < x1 < x2 < · · · < xn−1 < b
These values divide [a, b] into n subintervals
[a, x1 ], [x1 , x2 ], . . . , [xn−1 , b] (3)
whose lengths, as usual, we denote by
)x1 , )x2 , . . . , )xn
By hypothesis, F  (x) = f(x) for all x in [a, b], so F satisfies the hypotheses of the Mean-
Value Theorem (4.8.2) on each subinterval in (3). Hence, we can find numbers x1∗ , x2∗ , . . . , xn∗
in the respective subintervals in (3) such that
F (x1 ) − F (a) = F  (x1∗ )(x1 − a) = f(x1∗ ))x1
F (x2 ) − F (x1 ) = F  (x2∗ )(x2 − x1 ) = f(x2∗ ))x2
F (x3 ) − F (x2 ) = F  (x3∗ )(x3 − x2 ) = f(x3∗ ))x3
..
.
F (b) − F (xn−1 ) = F  (xn∗ )(b − xn−1 ) = f(xn∗ ))xn
Adding the preceding equations yields
n
F (b) − F (a) = f(xk∗ ))xk (4)
k=1

Let us now increase n in such a way that max )xk → 0. Since f is assumed to be continuous,
b
the right side of (4) approaches a f(x) dx by Theorem 5.5.2 and Definition 5.5.1. However,
the left side of (4) is independent of n; that is, the left side of (4) remains constant as n
increases. Thus,
n b
F (b) − F (a) = lim f(xk∗ ))xk = f(x) dx
max )xk → 0 a
k=1

It is standard to denote the difference F (b) − F (a) as


b b
F (x) a
= F (b) − F (a) or F (x) a
= F (b) − F (a)
For example, using the first of these notations we can express (2) as

b b
f(x) dx = F (x) (5)
a a
January 18, 2001 14:01 g65-ch5 Sheet number 46 Page number 364 cyan magenta yellow black

364 Integration

2
Example 1 Evaluate x dx.
1

Solution. The function F (x) = 12 x 2 is an antiderivative of f(x) = x; thus, from (2)


2 2
1 2 1 2 1 2 1 3
x dx = x = (2) − (1) = 2 − = 
1 2 1 2 2 2 2

Example 2 In Example 5 of Section 5.4 we approximated the area under the graph
of y = 9 − x 2 over the interval [0, 3] using left endpoint, right endpoint, and midpoint
approximations, all of which produced an approximation of roughly 18 (square units). In
Example 7 of that section we used Definition 5.4.3 to prove that the exact area A is indeed 18.
We can now solve this problem more quickly using the Fundamental Theorem of Calculus:
3 3  
x3 27
A= (9 − x ) dx = 9x −
2
= 27 − − 0 = 18 
0 3 0 3
y
1 y = cos x
Example 3
x
(a) Find the area under the curve y = cos x over the interval [0, π/2] (Figure 5.6.3).
6 c i
(b) Make a conjecture about the value of the integral
-1 π
cos x dx
Figure 5.6.3 0
and confirm your conjecture using the Fundamental Theorem of Calculus.

Solution (a). Since cos x ≥ 0 over the interval [0, π/2], the area A under the curve is
π/2
π/2
π
A= cos x dx = sin x = sin − sin 0 = 1
0 2
0

Solution (b). The given integral can be interpreted as the signed area between the graph of
y = cos x and the interval [0, π]. The graph in Figure 5.6.3 suggests that over the interval
[0, π] the portion of area above the x-axis is the same as the portion of area below the x-axis,
so we conjecture that the signed area is zero; this implies that the value of the integral is
zero. This is confirmed by the computations
π π
cos x dx = sin x = sin π − sin 0 = 0 
0 0
••••••••••••••••••••••••••••••••••••••
Observe that in the preceding examples we did not include a constant of integration in the
THE RELATIONSHIP BETWEEN antiderivatives. In general, when applying the Fundamental Theorem of Calculus there is
DEFINITE AND INDEFINITE
INTEGRALS no need to include a constant of integration because it will drop out anyhow. To see that this
is so, let F be any antiderivative of the integrand on [a, b], and let C be any constant; then
b b
f(x) dx = F (x) + C = [F (b) + C] − [F (a) + C] = F (b) − F (a)
a a
Thus, for purposes of evaluating a definite integral we can omit the constant of integration
in
b b
f(x) dx = F (x) + C
a a
and express (5) as
b b
f(x) dx = f(x) dx (6)
a a

which relates the definite and indefinite integrals.


January 18, 2001 14:01 g65-ch5 Sheet number 47 Page number 365 cyan magenta yellow black

5.6 The Fundamental Theorem of Calculus 365

Example 4
9 9 9
9 √ √ 2 3/2 2 52
x dx = x dx = x 1/2 dx = x = (27 − 1) = 
1 1 1 3 1 3 3

• REMARK.
• Usually, we will dispense with the step of displaying the indefinite integral





• explicitly and write the antiderivative immediately, as in our first three examples.

Example 5 Table 5.2.1 will be helpful for the following computations.

Solution.
9
9 √ 9
x 2
x dx = x 5/2
dx = 2 7/2
7
x = 27 (2187 − 128) = 4118
7
= 588 27
4 4 4
π/2
π/2
sin x cos x 1  π  1 1
dx = − =− cos − cos 0 = − [0 − 1] =
0 5 5 5 2 5 5
0
π/3
π/3 π √ √
sec2 x dx = tan x = tan − tan 0 = 3−0= 3
0 3
0
π/4
π/4 π  π √ √
sec x tan x dx = sec x = sec − sec − = 2− 2=0 
−π/4 4 4
−π/4

• WARNING.
• The requirements in the Fundamental Theorem of Calculus that f be contin-







uous on [a, b] and that F be an antiderivative for f over the entire interval [a, b] are important





to keep in mind. Disregarding these assumptions will likely lead to incorrect results. For






example, the function f(x) = 1/x 2 fails on two counts to be continuous at x = 0: f(x) is





not defined at x = 0 and limx → 0 f(x) does not exist. Thus, the Fundamental Theorem of
Calculus should not be used to integrate f on any interval that contains x = 0. However, if










we ignore this and blindly apply Formula (2) over the interval [−1, 1], we might think that




• 1 1

• 1 1
= − = −[1 − (−1)] = −2




2
dx
−1 x x −1






This answer is clearly ridiculous, since f(x) = 1/x 2 is a nonnegative function and hence










cannot possibly produce a negative definite integral. Indeed, even if we were to extend f





to be defined at 0, say by setting


• 
1/x 2 , x = 0



f(x) =



x=0





0,








f would still be unbounded on any interval containing x = 0, so Theorem 5.5.8(b) tells us


• that f is not even integrable across any such interval.

• FOR THE READER.


• If you have a CAS, read the documentation on evaluating definite





• integrals, and then check the results in the preceding examples.

The Fundamental Theorem of Calculus can be applied without modification to definite


integrals in which the lower limit of integration is greater than or equal to the upper limit
of integration.
January 18, 2001 14:01 g65-ch5 Sheet number 48 Page number 366 cyan magenta yellow black

366 Integration

Example 6
1 1
x3 1 1
x 2 dx = = − =0
1 3 1 3 3
0 2 0
x 0 16
x dx = = − = −8
4 2 4 2 2
The latter result is consistent with the result that would be obtained by first reversing the
limits of integration in accordance with Definition 5.5.3(b):
0 4 4 
x2 16 0
x dx = − x dx = − =− − = −8 
4 0 2 0 2 2
To integrate a continuous function that is defined piecewise on an interval [a, b], split
this interval into subintervals at the breakpoints of the function, and integrate separately
over each subinterval in accordance with Theorem 5.5.5.
6
Example 7 Evaluate f(x) dx if
0

x2, x<2
f(x) =
3x − 2, x≥2

Solution. From Theorem 5.5.5


6 2 6 2 6
f(x) dx = f(x) dx + f(x) dx = x 2 dx + (3x − 2) dx
0 0 2 0 2
2 6  
x3 3x 2 8 128
= + − 2x = − 0 + (42 − 2) = 
3 0 2 2 3 3

2
Example 8 Evaluate |x| dx.
−1

Solution. Since |x| = x when x ≥ 0 and |x| = −x when x ≤ 0,


2 0 2
|x| dx = |x| dx + |x| dx
−1 −1 0
0 2
= (−x) dx + x dx
−1 0
0 2
x2 x2 1 5
=− + = +2= 
2 −1 2 0 2 2
••••••••••••••••••••••••••••••••••••••
To evaluate a definite integral using the Fundamental Theorem of Calculus, one needs to be
DUMMY VARIABLES able to find an antiderivative of the integrand; thus, it is important to know what kinds of
functions have antiderivatives. It is our next objective to show that all continuous functions
have antiderivatives, but to do this we will need some preliminary results.
Formula (6) shows that there is a close relationship between the integrals
b
f(x) dx and f(x) dx
a
However, the definite and indefinite integrals differ in some important ways. For one thing,
the two integrals are different kinds of objects—the definite integral is a number (the net
signed area between the graph of y = f(x) and the interval [a, b]), whereas the indefinite
integral is a function, or more accurately a set of functions [the antiderivatives of f(x)].
January 18, 2001 14:01 g65-ch5 Sheet number 49 Page number 367 cyan magenta yellow black

5.6 The Fundamental Theorem of Calculus 367

However, the two types of integrals also differ in the role played by the variable of integration.
In an indefinite integral, the variable of integration is “passed through” to the antiderivative
in the sense that integrating a function of x produces a function of x, integrating a function
of t produces a function of t, and so forth. For example,
x3 t3
x 2 dx = + C and t 2 dt = +C
3 3
In contrast, the variable of integration in a definite integral is not passed through to the end
result, since the end result is a number. Thus, integrating a function of x over an interval and
integrating the same function of t over the same interval of integration produce the same
value for the integral. For example,
3 3 3 3
x3 27 1 26 t3 27 1 26
x 2 dx = = − = and t 2 dt = = − =
1 3 x=1 3 3 3 1 3 t=1 3 3 3
However, this latter result should not be surprising, since the area under the graph of the
curve y = f(x) over an interval [a, b] on the x-axis is the same as the area under the graph
of the curve y = f(t) over the interval [a, b] on the t-axis (Figure 5.6.4).

y y

y = f (x) y = f (t)

A A
x t
a b a b

b b
A= f (x) dx = f (t) dt
a a

Figure 5.6.4

Because the variable of integration in a definite integral plays no role in the end result,
it is often referred to as a dummy variable. In summary:

Whenever you find it convenient to change the letter used for the variable of integration
in a definite integral, you can do so without changing the value of the integral.

••••••••••••••••••••••••••••••••••••••
To reach our goal of showing that continuous functions have antiderivatives, we will need
THE MEAN-VALUE THEOREM FOR to develop a basic property of definite integrals, known as the Mean-Value Theorem for
INTEGRALS
Integrals. In the next section we will use this theorem to extend the familiar idea of “aver-
age value” so that it applies to continuous functions, but here we will need it as a tool for
developing other results.
y = f (x) Let f be a continuous nonnegative function on [a, b], and let m and M be the minimum
M and maximum values of f(x) on this interval. Consider the rectangles of heights m and M
over the interval [a, b] (Figure 5.6.5). It is clear geometrically from this figure that the area
m b

a b A= f(x) dx
a
Figure 5.6.5
under y = f(x) is at least as large as the area of the rectangle of height m and no larger than
the area of the rectangle of height M. It seems reasonable, therefore, that there is a rectangle
over the interval [a, b] of some appropriate height f(x ∗ ) between m and M whose area is
January 18, 2001 14:01 g65-ch5 Sheet number 50 Page number 368 cyan magenta yellow black

368 Integration

y = f (x) precisely A; that is,


b
f(x) dx = f(x ∗ )(b − a)
a

f (x*) (Figure 5.6.6). This is a special case of the following result.

a x* b
5.6.2 THEOREM (The Mean-Value Theorem for Integrals). If f is continuous on a closed
Figure 5.6.6
interval [a, b], then there is at least one number x ∗ in [a, b] such that
b
f(x) dx = f(x ∗ )(b − a) (7)
a

Proof. By the Extreme-Value Theorem (4.5.3), f assumes a maximum value M and a mini-
mum value m on [a, b]. Thus, for all x in [a, b],
m ≤ f(x) ≤ M
and from Theorem 5.5.6(b)
b b b
m dx ≤ f(x) dx ≤ M dx
a a a
or
b
m(b − a) ≤ f(x) dx ≤ M(b − a) (8)
a
or
b
1
m≤ f(x) dx ≤ M
b−a a
This implies that
b
1
f(x) dx (9)
b−a a
is a number between m and M, and since f(x) assumes the values m and M on [a, b], it
follows from the Intermediate-Value Theorem (2.5.8) that f(x) must assume the value (9)
at some x ∗ in [a, b]; that is,
b b
1
f(x) dx = f(x ∗ ) or f(x) dx = f(x ∗ )(b − a)
b−a a a

Example 9 Since f(x) = x 2 is continuous on the interval [1, 4], the Mean-Value Theorem
for Integrals guarantees that there is a number x ∗ in [1, 4] such that
4
x 2 dx = f(x ∗ )(4 − 1) = (x ∗ )2 (4 − 1) = 3(x ∗ )2
1
But
4 4
x3
x 2 dx = = 21
1 3 1
so that

3(x ∗ )2 = 21 or (x ∗ )2 = 7 or x ∗ = ± 7

Thus, x ∗ = 7 ≈ 2.65 is the number in the interval [1, 4] whose existence is guaranteed
by the Mean-Value Theorem for Integrals. 
January 18, 2001 14:01 g65-ch5 Sheet number 51 Page number 369 cyan magenta yellow black

5.6 The Fundamental Theorem of Calculus 369

••••••••••••••••••••••••••••••••••••••
In Section 5.1 we suggested that if f is continuous and nonnegative on [a, b], and if
PART 2 OF THE FUNDAMENTAL A(x) is the area under the graph of y = f(x) over the interval [a, x] (Figure 5.6.2), then
THEOREM OF CALCULUS
A (x) = f(x). But A(x) can be expressed as the definite integral
x
A(x) = f(t) dt
a
(where we have used t rather than x as the variable of integration to avoid confusion with
the x that appears as the upper limit of integration). Thus, the relationship A (x) = f(x)
can be expressed as
x 
d
f(t) dt = f(x)
dx a
This is a special case of the following more general result, which applies even if f has
negative values.

5.6.3 THEOREM (The Fundamental Theorem of Calculus, Part 2). If f is continuous on an


interval I, then f has an antiderivative on I . In particular, if a is any number in I, then
the function F defined by
x
F (x) = f(t) dt
a
is an antiderivative of f on I ; that is, F  (x) = f(x) for each x in I, or in an alternative
notation
x 
d
f(t) dt = f(x) (10)
dx a

Proof. We will show first that F (x) is defined at each x in the interval I . If x > a and x
is in the interval I , then Theorem 5.5.2 applied to the interval [a, x] and the continuity of
f on I ensure that F (x) is defined; and if x is in the interval I and x ≤ a, then Definition
5.5.3 combined with Theorem 5.5.2 ensures that F (x) is defined. Thus, F (x) is defined for
all x in I .
Next we will show that F  (x) = f(x) for each x in the interval I . If x is not an endpoint
of I , then it follows from the definition of a derivative that
F (w) − F (x)
F  (x) = lim
w→x w−x
 w x 
1
= lim f(t) dt − f(t) dt
w→x w − x a a
 w a 
1
= lim f(t) dt + f(t) dt
w→x w − x a x
 w 
1
= lim f(t) dt (11)
w→x w − x x
w
Applying the Mean-Value Theorem for Integrals (5.6.2) to x f(t) dt, we obtain
w
1 1
f(t) dt = [f(t ∗ ) · (w − x)] = f(t ∗ ) (12)
w−x x w−x
where t ∗ is some number between x and w. Because t ∗ is between x and w, it follows that
t ∗ → x as w → x. Thus f(t ∗ ) → f(x) as w → x, since f is assumed continuous at x.
Therefore, it follows from (11) and (12) that
 w 
 1
F (x) = lim f(t) dt = lim f(t ∗ ) = f(x)
w→x w − x x w→x
January 18, 2001 14:01 g65-ch5 Sheet number 52 Page number 370 cyan magenta yellow black

370 Integration

If x is an endpoint of the interval I , then the two-sided limits in the proof must be replaced
by the appropriate one-sided limits, but otherwise the arguments are identical.
In words, Formula (10) states:

If a definite integral has a variable upper limit of integration, a constant lower limit of
integration, and a continuous integrand, then the derivative of the integral with respect
to its upper limit is equal to the integrand evaluated at the upper limit.

Example 10 Find
x 
d
t 3 dt
dx 1
by applying Part 2 of the Fundamental Theorem of Calculus, and then confirm the result by
performing the integration and then differentiating.

Solution. The integrand is a continuous function, so from (10)


x 
d
t dt = x 3
3
dx 1
Alternatively, evaluating the integral and then differentiating yields
x 
x
t4 x4 1 d x4 1
t dt =
3
= − , − = x3
1 4 t=1 4 4 dx 4 4
so the two methods for differentiating the integral agree. 
Example 11 Since
sin x
f(x) =
x
is continuous on any interval that does not contain the origin, it follows from (10) that on
the interval (0, +⬁) we have
x 
d sin t sin x
dt =
dx 1 t x
Unlike the preceding example, there is no way to evaluate the integral in terms of familiar
functions, so Formula (10) provides the only simple method for finding the derivative.

••••••••••••••••••••••••••••••••••••••
The two parts of the Fundamental Theorem of Calculus, when taken together, tell us that
DIFFERENTIATION AND differentiation and integration are inverse processes in the sense that each undoes the effect
INTEGRATION ARE INVERSE
PROCESSES of the other. To see why this is so, note that Part 1 of the Fundamental Theorem of Calculus
(5.6.1) implies that
x
f  (t) dt = f(x) − f(a)
a
which tells us that if the value of f(a) is known, then the function f can be recovered from
its derivative f  by integrating. Conversely, Part 2 of the Fundamental Theorem of Calculus
(5.6.3) states that
x 
d
f(t) dt = f(x)
dx a
which tells us that the function f can be recovered from its integral by differentiating. Thus,
differentiation and integration can be viewed as inverse processes.
It is common to treat parts 1 and 2 of the Fundamental Theorem of Calculus as a single
theorem, and refer to it simply as the Fundamental Theorem of Calculus. This theorem
ranks as one of the greatest discoveries in the history of science, and its formulation by
Newton and Leibniz is generally regarded to be the “discovery of calculus.”
January 18, 2001 14:01 g65-ch5 Sheet number 53 Page number 371 cyan magenta yellow black

5.6 The Fundamental Theorem of Calculus 371

EXERCISE SET 5.6 Graphing Calculator C CAS


••••••••••••••••••••••••••••••••••••••••••••••••••••••••••••••••••••••••••••••••••••••••••••••••••••••••••••••
2 3π/4
1. In each part, use a definite integral to find the area of the 21. (a) |2x − 3| dx (b) | cos x| dx
region, and check your answer using an appropriate formula 0 0
from geometry. 2  π/2 1 
22. (a) 2 + |x| dx (b)  − sin x  dx
2
(a) (b) (c) −1 0
y y y
y= x+1 C 23. (a) CAS programs provide methods for entering functions
y=2 that are defined piecewise. Check your documentation
2
to see how this is done, and then use the CAS to evaluate
y=2–x 
2 x, x ≤ 1
x x x f(x) dx, where f(x) =
0 2 –1 1 0 1 3 0 x2, x > 1
Use Theorem 5.5.5 to check the answer by hand.
2. In each part, use a definite integral to find the area under (b) Find a formula for an antiderivative F of f on the in-
the curve y = f(x) over the stated interval, and check your terval [0, 4] and verify that
answer using an appropriate formula from geometry.
(a) f(x) = x; [0, 5] 2
(b) f(x) = 5; [3, 9] f(x) dx = F (2) − F (0)
(c) f(x) = x + 3; [−1, 2] 0

C 24. (a) Use a CAS to evaluate


In Exercises 3–6, find the area under the curve y = f(x) over √
the stated interval.
4 x, 0≤x<1
f(x) dx, where f(x) =
0 1/x , 2
x≥1
3. f(x) = x 3 ; [2, 3] 4. f(x) = x 4 ; [−1, 1]
√ Use Theorem 5.5.5 to check the answer by hand.
5. f(x) = x; [1, 9] 6. f(x) = x −3/5 ; [1, 4]
(b) Find a formula for an antiderivative F of f on the in-
In Exercises 7–19, evaluate the integrals using Part 1 of the terval [0, 4] and verify that
Fundamental Theorem of Calculus. 4
f(x) dx = F (4) − F (0)
0 2 0
7. (x 2 − 4x + 7) dx 8. x(1 + x 3 ) dx
−3 −1
3 2 In Exercises 25–27, use a calculating utility to find the mid-
1 1
9. 2
dx 10. dx point approximation of the integral using n = 20 subinter-
1 x 1 x6 vals, and then find the exact value of the integral using Part
9 √ 8
1 of the Fundamental Theorem of Calculus.
11. 2x x dx 12. (5x 2/3 − 4x −2 ) dx
4 1
3 π/2 1
π/2 π/4 1
13. sin θ dθ 14. sec2 θ dθ 25. dx 26. sin x dx 27. sec2 x dx
−π/2 0 1 x2 0 −1
π/4 1 C 28. Compare the answers obtained by the midpoint rule in Ex-
15. cos x dx 16. (x − sec x tan x) dx ercises 25–27 to those obtained using the built-in numerical
−π/4 0
4 √
 (approximate) integration command of a calculating utility
3
17. √ − 5 t − t −3/2 dt or a CAS.
1 t 29. Find the area under the curve y = x 2 + 1 over the interval
9
18. (4y −1/2
+ 2y 1/2
+y −5/2
) dy [0, 3]. Make a sketch of the region.
4 30. Find the area that is above the x-axis, but below the curve
π/2  
2 y = (1 − x)(x − 2). Make a sketch of the region.
19. dx x+
π/6 sin2 x 31. Find the area under the curve y = 3 sin x over the interval
C 20. Use a CAS to evaluate the integral [0, 2π/3]. Sketch the region.
4a
32. Find the area below the interval [−2, −1], but above the
(a 1/2 − x 1/2 ) dx
a curve y = x 3 . Make a sketch of the region.
and check the answer by hand. 33. Find the total area between the curve y = x 2 − 3x − 10
and the interval [−3, 8]. Make a sketch of the region. [Hint:
In Exercises 21 and 22, use Theorem 5.5.5 to evaluate the Find the portion of area above the interval and the portion
given integrals. of area below the interval separately.]
January 18, 2001 14:01 g65-ch5 Sheet number 54 Page number 372 cyan magenta yellow black

372 Integration

0
34. (a) Use a graphing utility to generate the graph of d t
41. dt [Hint: Use Definition 5.5.3(b).]
1 dx x cos t
f(x) = (x + 2)(x + 1)(x − 3)(x − 5) u
100 d
42. |x| dx
and use the graph to make a conjecture about the sign du 0
of the integral x 
5 43. Let F (x) = 3t 2 + 1 dt. Find
f(x) dx 2
−2 (a) F (2) (b) F  (2) (c) F  (2).
(b) Check your conjecture by evaluating the integral. x
cos t
35. (a) Let f be an odd function; that is, f(−x) = −f(x). In- 44. Let F (x) = dt. Find
0 t2 + 3
vent a theorem that makes a statement about the value
(a) F (0) (b) F  (0) (c) F  (0).
of an integral of the form
a
x
t −3
45. Let F (x) = dt for −⬁ < x < +⬁.
f(x) dx 0 t2 + 7
−a
(b) Confirm that your theorem works for the integrals (a) Find the value of x where F attains its minimum value.
1 π/2 (b) Find intervals over which F is only increasing or only
x 3 dx and sin x dx decreasing.
−1 −π/2 (c) Find open intervals over which F is only concave up or
(c) Let f be an even function; that is, f(−x) = f(x). In- only concave down.
vent a theorem that makes a statement about the rela- 46. Use the plotting and numerical integration commands of a
C
tionship between the integrals CAS to generate the graph of the function F in Exercise 45
a a
f(x) dx and f(x) dx over the interval −20 ≤ x ≤ 20, and confirm that the graph
−a 0 is consistent with the results obtained in that exercise.
(d) Confirm that your theorem works for the integrals 47. (a) Over what open interval does the formula
1 π/2 x
dt
x 2 dx and cos x dx F (x) =
−1 −π/2 1 t
C 36. Use the theorem you invented in Exercise 35(a) to evaluate represent an antiderivative of f(x) = 1/x?
the integral (b) Find a point where the graph of F crosses the x-axis.
5
x7 − x5 + x 48. (a) Over what open interval does the formula
dx
−5 x4 + x2 + 7 x
1
F (x) = dt
and check your answer with a CAS. 1 t −9
2

37. Define F (x) by represent an antiderivative of


x
1
F (x) = (t 3 + 1) dt f(x) = 2 ?
1 x −9
(a) Use Part 2 of the Fundamental Theorem of Calculus to (b) Find a point where the graph of F crosses the x-axis.
find F  (x).
(b) Check the result in part (a) by first integrating and then In Exercises 49 and 50, find all values of x ∗ in the stated inter-
differentiating. val that satisfy Equation (7) in the Mean-Value Theorem for
38. Define F (x) by Integrals (5.6.2), and explain what these numbers represent.
x
F (x) = cos 2t dt √
49. (a) f(x) = x; [0, 9]
π/4
(b) f(x) = 3x 2 + 2x + 1; [−1, 2]
(a) Use Part 2 of the Fundamental Theorem of Calculus to
find F  (x). 50. (a) f(x) = sin x; [−π, π] (b) f(x) = 1/x 2 ; [1, 3]
(b) Check the result in part (a) by first integrating and then
differentiating. It was shown in the proof of the Mean-Value Theorem for
Integrals (5.6.2) that if f is continuous on [a, b], and if
In Exercises 39–42, use Part 2 of the Fundamental Theorem m ≤ f(x) ≤ M on [a, b], then
of Calculus to find the derivatives. b
m(b − a) ≤ f(x) dx ≤ M(b − a)
d x √ d x  a
39. (a) sin( t ) dt (b) 1 + cos2 t dt
dx 1 dx 1
[see (8)]. These inequalities make it possible to obtain bounds
x x on the size of a definite integral from bounds on the size of
d dt d dt
40. (a) √ (b) dt its integrand. This is illustrated in Exercises 51 and 52.
dx 0 1+ t dx 1 1 + t + t2
January 18, 2001 14:01 g65-ch5 Sheet number 55 Page number 373 cyan magenta yellow black

5.7 Rectilinear Motion Revisited; Average Value 373



51. Find the maximum and minimum values of x 3 + 2 for 53. Prove:
0 ≤ x ≤ 3, and use these values to find bounds on the value (a) [cF (x)]ba = c[F (x)]ba
of the integral
3 (b) [F (x) + G(x)]ba = F (x)]ba + G(x)]ba
x 3 + 2 dx
0 (c) [F (x) − G(x)]ba = F (x)]ba − G(x)]ba .
52. Find values of m and M such that m ≤ x sin x ≤ M for 54. Prove the Mean-Value Theorem for Integrals (Theorem
0 ≤ x ≤ π, and use these values to find bounds on the value 5.6.2) by applying the Mean-Value Theorem (4.8.2) to an
of the integral antiderivative F for f .
π
x sin x dx
0

5.7 RECTILINEAR MOTION REVISITED; AVERAGE VALUE

In Section 4.4 we used the derivative to define the notions of instantaneous velocity
and acceleration for a particle moving along a line. In this section we will resume the
study of such motion using the tools of integration. We will also investigate the general
problem of integrating a rate of change, and we will show how the definite integral
can be used to define the average value of a continuous function. More applications of
integration will be given in Chapter 6.

••••••••••••••••••••••••••••••••••••••
Recall from Definitions 4.4.1 and 4.4.2 that if s(t) is the position function of a particle
FINDING POSITION AND VELOCITY moving on a coordinate line, then the instantaneous velocity and acceleration of the particle
BY INTEGRATION
are given by the formulas
ds dv d 2s
v(t) = s  (t) = and a(t) = v  (t) = = 2
dt dt dt
It follows from these formulas that s(t) is an antiderivative of v(t) and v(t) is an antideriva-
s tive of a(t); that is,

s(t) = v(t) dt and v(t) = a(t) dt (1–2)


s0
Thus, if the velocity of a particle is known, then its position function can be obtained from
(1) by integration, provided there is sufficient additional information to determine the con-
t
stant of integration. In particular, we can determine the constant of integration if we know
t0
the position s0 of the particle at some time t0 , since this information determines a unique
There is a unique position antiderivative s(t) (Figure 5.7.1). Similarly, if the acceleration function of the particle is
function such that s(t0) = s0. known, then its velocity function can be obtained from (2) by integration if we know the
velocity v0 of the particle at some time t0 (Figure 5.7.2).
Figure 5.7.1

Example 1 Find the position function of a particle that moves with velocity v(t) = cos πt
v along a coordinate line, assuming that the particle has coordinate s = 4 at time t = 0.

Solution. The position function is


1
v0 s(t) = v(t) dt = cos πt dt = sin πt + C
π
Since s = 4 when t = 0, it follows that
t
1
t0 4 = s(0) = sin 0 + C = C
π
There is a unique velocity Thus,
function such that v(t0) = v0.
1
Figure 5.7.2
s(t) = sin πt + 4 
π
January 18, 2001 14:01 g65-ch5 Sheet number 56 Page number 374 cyan magenta yellow black

374 Integration

••••••••••••••••••••••••••••••••••••••
One of the most important cases of rectilinear motion occurs when a particle has constant
UNIFORMLY ACCELERATED acceleration. We call this uniformly accelerated motion.
MOTION
We will show that if a particle moves with constant acceleration along an s-axis, and if
the position and velocity of the particle are known at some point in time, say when t = 0,
then it is possible to derive formulas for the position s(t) and the velocity v(t) at any time
t. To see how this can be done, suppose that the particle has constant acceleration
a(t) = a (3)
and
s = s0 when t = 0 (4)
v = v0 when t = 0 (5)
where s0 and v0 are known. We call (4) and (5) the initial conditions for the motion.
With (3) as a starting point, we can integrate a(t) to obtain v(t), and we can integrate v(t)
to obtain s(t), using an initial condition in each case to determine the constant of integration.
The computations are as follows:

v(t) = a(t) dt = a dt = at + C1 (6)

To determine the constant of integration C1 we apply initial condition (5) to this equation
to obtain
v0 = v(0) = a · 0 + C1 = C1
Substituting this in (6) and putting the constant term first yields
v(t) = v0 + at
Since v0 is constant, it follows that

s(t) = v(t) dt = (v0 + at) dt = v0 t + 12 at 2 + C2 (7)

To determine the constant C2 we apply initial condition (4) to this equation to obtain
s0 = s(0) = v0 · 0 + 12 a · 0 + C2 = C2
Substituting this in (7) and putting the constant term first yields
s(t) = s0 + v0 t + 12 at 2
In summary, we have the following result.

5.7.1 UNIFORMLY ACCELERATED MOTION. If a particle moves with constant accel-


eration a along an s-axis, and if the position and velocity at time t = 0 are s0 and v0 ,
respectively, then the position and velocity functions of the particle are

s(t) = s0 + v0 t + 12 at 2 (8)

v(t) = v0 + at (9)

• FOR THE READER.


• How can you tell from the velocity versus time curve whether a particle





• moving along a line has uniformly accelerated motion?

Example 2 Suppose that an intergalactic spacecraft uses a sail and the “solar wind” to
produce a constant acceleration of 0.032 m/s2 . Assuming that the spacecraft has a velocity
of 10,000 m/s when the sail is first raised, how far will the spacecraft travel in 1 hour, and
what will its velocity be at the end of this hour?

Solution. In this problem the choice of a coordinate axis is at our discretion, so we will
choose it to make the computations as simple as possible. Accordingly, let us introduce an
January 18, 2001 14:01 g65-ch5 Sheet number 57 Page number 375 cyan magenta yellow black

5.7 Rectilinear Motion Revisited; Average Value 375

s-axis whose positive direction is in the direction of motion, and let us take the origin to
coincide with the position of the spacecraft at the time t = 0 when the sail is raised. Thus,
the Formulas (8) and (9) for uniformly accelerated motion apply with
s0 = s(0) = 0, v0 = v(0) = 10,000, and a = 0.032
Since 1 hour corresponds to t = 3600 s, it follows from (8) that in 1 hour the spacecraft
travels a distance of
s(3600) = 10,000(3600) + 12 (0.032)(3600)2 ≈ 36,200,000 m
and it follows from (9) that after 1 hour its velocity is
v(3600) = 10,000 + (0.032)(3600) ≈ 10,100 m/s 
Example 3 A bus has stopped to pick up riders, and a woman is running at a constant
velocity of 5 m/s to catch it. When she is 11 m behind the front door the bus pulls away
Bus
with a constant acceleration of 1 m/s2 . From that point in time, how long will it take for
s the woman to reach the front door of the bus if she keeps running with a velocity of 5 m/s?
Woman 0
Solution. As shown in Figure 5.7.3, choose the s-axis so that the bus and the woman are
11 m moving in the positive direction, and the front door of the bus is at the origin at the time
Figure 5.7.3
t = 0 when the bus begins to pull away. To catch the bus at some later time t, the woman
will have to cover a distance sw(t) that is equal to 11 m plus the distance sb (t) traveled by
the bus; that is, the woman will catch the bus when
sw(t) = sb (t) + 11 (10)
Since the woman has a constant velocity of 5 m s, the distance she travels in t seconds is
/
sw(t) = 5t. Thus, (10) can be written as
sb (t) = 5t − 11 (11)
Since the bus has a constant acceleration of a = 1 m s , and since s0 = v0 = 0 at time
/ 2

t = 0 (why?), it follows from (8) that


sb (t) = 12 t 2
Substituting this equation into (11) and reorganizing the terms yields the quadratic equation
1 2
2
t − 5t + 11 = 0 or t 2 − 10t + 22 = 0
Solving this equation for t using the quadratic formula yields two solutions:
√ √
t = 5 − 3 ≈ 3.3 and t = 5 + 3 ≈ 6.7
(verify). Thus, the woman can reach the door at two different times, t = 3.3 s and t = 6.7 s.
The reason that there are two solutions can be explained as follows: When the woman first
reaches the door, she is running faster than the bus and can run past it if the driver does not
see her. However, as the bus speeds up, it eventually catches up to her, and she has another
chance to flag it down. 
••••••••••••••••••••••••••••••••••••••
In Section 4.4 we discussed the free-fall model of motion near the surface of the Earth with
THE FREE-FALL MODEL the promise that we would derive Formula (5) of that section later in the text; we will now
show how to do this. As stated in 4.4.4 and illustrated in Figure 4.4.8, we will assume that
the object moves on an s-axis whose origin is at the surface of the Earth and whose positive
direction is up; and we will assume that the position and velocity of the object at time t = 0
are s0 and v0 , respectively.
It is a fact of physics that a particle moving on a vertical line near the Earth’s surface and
subject only to the force of the Earth’s gravity moves with essentially constant acceleration.
The magnitude of this constant, denoted by the letter g, is approximately 9.8 m/s2 or 32

ft/s2 , depending on whether distance is measured in meters or feet.

Strictly speaking, the constant g varies with the latitude and the distance from the Earth’s center. However, for
motion at a fixed latitude and near the surface of the Earth, the assumption of a constant g is satisfactory for many
applications.
January 18, 2001 14:01 g65-ch5 Sheet number 58 Page number 376 cyan magenta yellow black

376 Integration

Recall that a particle is speeding up when its velocity and acceleration have the same
sign and is slowing down when they have opposite signs. Thus, because we have chosen
the positive direction to be up, it follows that the acceleration a(t) of a particle in free fall
is negative for all values of t. To see that this is so, observe that an upward-moving particle
(positive velocity) is slowing down, so its acceleration must be negative; and a downward-
moving particle (negative velocity) is speeding up, so its acceleration must also be negative.
Thus, we conclude that
a(t) = −g
and hence it follows from (8) and (9) that the position and velocity functions of an object
in free fall are

s(t) = s0 + v0 t − 12 gt 2 (12)

v(t) = v0 − gt (13)

• FOR THE READER.


• Had we chosen the positive direction of the s-axis to be down, then

the acceleration would have been a(t) = g (why?). How would this have affected Formulas








• (12) and (13)?

Example 4 A ball is hit directly upward with an initial velocity of 49 m/s and is struck
at a point that is 1 m above the ground. Assuming that the free-fall model applies, how high
will the ball travel?

Solution. Since distance is in meters, we take g = 9.8 m/s2 . Initially, we have s0 = 8


and v0 = 49, so from (12) and (13)
v(t) = −9.8t + 49
s(t) = −4.9t 2 + 49t + 1

s The ball will rise until v(t) = 0, that is, until −9.8t + 49 = 0 or t = 5. At this instant the
height above the ground will be
1250 s(5) = −4.9(5)2 + 49(5) + 1 = 123.5 m 

Example 5 A penny is released from rest near the top of the Empire State Building at
a point that is 1250 ft above the ground (Figure 5.7.4). Assuming that the free-fall model
applies, how long does it take for the penny to hit the ground, and what is its speed at the
time of impact?

Solution. Since distance is in feet, we take g = 32 ft/s2 . Initially, we have s0 = 1250 and
0
v0 = 0, so from (12)
Figure 5.7.4 s(t) = −16t 2 + 1250 (14)
Impact occurs when s(t) = 0. Solving this equation for t, we obtain
−16t 2 + 1250 = 0
1250 625
t2 = =
16 8
25
t = ± √ ≈ ±8.8 s
8

Since t ≥ 0, we can discard the negative solution and conclude that it takes 25/ 8 ≈ 8.8 s
for the penny to hit the ground. To obtain the velocity at the time of impact, we substitute
January 18, 2001 14:01 g65-ch5 Sheet number 59 Page number 377 cyan magenta yellow black

5.7 Rectilinear Motion Revisited; Average Value 377



t = 25/ 8, v0 = 0, and g = 32 in (13) to obtain
   
25 25 √
v √ = 0 − 32 √ = −200 2 ≈ −282.8 ft/s
8 8
Thus, the speed at the time of impact is
  
 25  √
v √
  = 200 2 ≈ 282.8 ft/s
8
which is more than 192 mi/h. 
••••••••••••••••••••••••••••••••••••••
The Fundamental Theorem of Calculus
INTEGRATING RATES OF CHANGE b
f(x) dx = F (b) − F (a) (15)
a
has a useful interpretation that can be seen by rewriting it in a slightly different form. Since
F is an antiderivative of f on the interval [a, b], we can use the relationship F  (x) = f(x)
to rewrite (15) as
y
b
y = F(x)
Slope = F′(x)
F  (x) dx = F (b) − F (a) (16)
a
F(b) – F(a)
In this formula we can view F  (x) as the rate of change of F (x) with respect to x, and
we can view F (b) − F (a) as the change in the value of F (x) as x increases from a to b
x (Figure 5.7.5). Thus, we have the following useful principle.
a b

Integrating the slope of y = F(x)


5.7.2 INTEGRATING A RATE OF CHANGE. Integrating the rate of change of F (x) with
over the interval [a, b] produces respect to x over an interval [a, b] produces the change in the value of F (x) that occurs
the change F(b) – F(a) in the as x increases from a to b.
value of F(x).

Figure 5.7.5 Here are some examples of this idea:

• If P (t) is a population (e.g., plants, animals, or people) at time t, then P  (t) is the rate
at which the population is changing at time t, and
t2
P  (t) dt = P (t2 ) − P (t1 )
t1

is the change in the population between times t1 and t2 .


• If A(t) is the area of an oil spill at time t, then A (t) is the rate at which the area of the
spill is changing at time t, and
t2
A (t) dt = A(t2 ) − A(t1 )
t1

is the change in the area of the spill between times t1 and t2 .


• If P  (x) is the marginal profit that results from producing and selling x units of a product
(see Section 4.6), then
x2
P  (x) dx = P (x2 ) − P (x1 )
x1

is the change in the profit that results when the production level increases from x1 units
to x2 units.

••••••••••••••••••••••••••••••••••••••
As another application of (16), suppose that s(t) and v(t) are the position and velocity
DISPLACEMENT IN RECTILINEAR functions of a particle moving on a coordinate line. Since v(t) is the rate of change of s(t)
MOTION
with respect to t, it follows from the principle in 5.7.2 that integrating v(t) over an interval
January 18, 2001 14:01 g65-ch5 Sheet number 60 Page number 378 cyan magenta yellow black

378 Integration

[t0 , t1 ] will produce the change in the value of s(t) as t increases from t0 to t1 ; that is,
t1 t1
v(t) dt = s  (t) dt = s(t1 ) − s(t0 ) (17)
t0 t0

The expression s(t1 ) − s(t0 ) in this formula is called the displacement or change in po-
sition of the particle over the time interval [t0 , t1 ]. For a particle moving horizontally, the
displacement is positive if the final position of the particle is to the right of its initial posi-
tion, negative if it is to the left of its initial position, and zero if it coincides with the initial
position (Figure 5.7.6).

Positive displacement Negative displacement


x x

Figure 5.7.6 s(t0) s(t1) s(t1) s(t0)

• REMARK.
• In physical problems it is important to associate the correct units with definite







integrals. In general, the units for the definite integral





b





f(x) dx

• a








will be units of f(x) times units of x. This is because the definite integral is a limit of Rie-
mann sums each of whose terms is a product of the form f(x) · )x. For example, if time










is measured in seconds (s) and velocity is measured in meters per second (m/s), then





integrating velocity over a time interval will produce a result whose units are in meters,
since m/s × s = m. Note that this is consistent with Formula (17), since displacement has








units of length.

••••••••••••••••••••••••••••••••••••••
In general, the displacement of a particle is not the same as the distance traveled by the
DISTANCE TRAVELED IN particle. For example, a particle that travels 100 units in the positive direction and then
RECTILINEAR MOTION
100 units in the negative direction travels a distance of 200 units but has a displacement of
zero, since it returns to its starting position. The only case in which the displacement and
the distance traveled are the same occurs when the particle moves in the positive direction
without reversing the direction of its motion.

• FOR THE READER.


• What is the relationship between the displacement of a particle and







the distance it travels if the particle moves in the negative direction without reversing the



• direction of motion?

From (17), integrating the velocity function of a particle over a time interval yields the
displacement of a particle over that time interval. In contrast, to find the total distance
traveled by the particle over the time interval (the distance traveled in the positive direction
plus the distance traveled in the negative direction), we must integrate the absolute value
of the velocity function; that is, we must integrate the speed:
 
total distance
t1
traveled during 
 time interval  = |v(t)| dt (18)
t0
[t0 , t1 ]

Example 6 Suppose that a particle moves on a coordinate line so that its velocity at time
t is v(t) = t 2 − 2t m/s.

(a) Find the displacement of the particle during the time interval 0 ≤ t ≤ 3.
(b) Find the distance traveled by the particle during the time interval 0 ≤ t ≤ 3.
January 18, 2001 14:01 g65-ch5 Sheet number 61 Page number 379 cyan magenta yellow black

5.7 Rectilinear Motion Revisited; Average Value 379

Solution (a). From (17) the displacement is


3 3 3
t3
v(t) dt = (t 2 − 2t) dt = − t2 =0
0 0 3 0
Thus, the particle is at the same position at time t = 3 as at t = 0.
Solution (b). The velocity can be written as v(t) = t 2 − 2t = t (t − 2), from which we
see that v(t) ≤ 0 for 0 ≤ t ≤ 2 and v(t) ≥ 0 for 2 ≤ t ≤ 3. Thus, it follows from (18) that
the distance traveled is
3 2 3
|v(t)| dt = −v(t) dt + v(t) dt
0 0 2
2 3
= −(t 2 − 2t) dt + (t 2 − 2t) dt
0 2
2 3
t3 t3 4 4 8
=− − t2 + − t2 = + = m 
3 0 3 2 3 3 3
••••••••••••••••••••••••••••••••••••••
In Section 4.4 we showed how to use the position versus time curve to obtain information
ANALYZING THE VELOCITY VERSUS about the behavior of a particle moving on a coordinate line (Table 4.4.1). Similarly, there is
TIME CURVE
valuable information that can be obtained from the velocity versus time curve. For example,
the integral in (17) can be interpreted geometrically as the net signed area between the graph
v
of v(t) and the interval [t0 , t1 ], and it can be interpreted physically as the displacement of
the particle over this interval. Thus, we have the following result.

5.7.3 FINDING DISPLACEMENT FROM THE VELOCITY VERSUS TIME CURVE. For a
t particle in rectilinear motion, the net signed area between the velocity versus time curve
t0 t1 and an interval [t0 , t1 ] on the t-axis represents the displacement of the particle over that
time interval (Figure 5.7.7).

The net signed area is the Example 7 Figure 5.7.8 shows three velocity versus time curves for a particle in rectilinear
displacement of the particle motion along a horizontal line. In each case, find the displacement of the particle over the
during the interval [ t0, t1]. time interval 0 ≤ t ≤ 4, and explain what it tells you about the motion of the particle.
Figure 5.7.7
v v v

1 1
t t t
4 4 1 2 3 4
-1 -1

(a) (b) (c)


Figure 5.7.8

Solution. In part (a) of Figure 5.7.8 the net signed area under the curve is 2, so the particle
is 2 units to the right of its starting point at the end of the time period. In part (b) the net
signed area under the curve is −2, so the particle is 2 units to the left of its starting point at
the end of the time period. In part (c) the net signed area under the curve is 0, so the particle
is back at its starting point at the end of the time period. 
By replacing the concept of net signed area with that of “total area,” we can also inter-
pret geometrically the total distance traveled by a particle in rectilinear motion. If f(x) is a
continuous function on an interval [a, b], we define the total area between the curve y =
f(x) and the interval to be the integral of |f(x)| over the interval [a, b]. Geometrically, the
total area is the area of the region that is between the graph of f and the x-axis.
January 18, 2001 14:01 g65-ch5 Sheet number 62 Page number 380 cyan magenta yellow black

380 Integration

y Example 8 Find the total area between the curve y = 1 − x 2 and the x-axis over the
interval [0, 2] (Figure 5.7.9).
1
y = 1 – x2

x Solution. The area A is given by


-1 1 2 2 1 2
-1 A= |1 − x 2 | dx = (1 − x 2 ) dx + −(1 − x 2 ) dx
0 0 1
1 2
-2 x3 x3
= x− − x−
3 0 3 1
-3
 
2 4
= − − =2 
Figure 5.7.9
3 3

From (18), integrating the speed |v(t)| over a time interval [t0 , t1 ] produces the distance
traveled by the particle during the time interval. However, we can also interpret the integral
in (18) as the total area between the velocity versus time curve and the interval [t0 , t1 ] on
the t-axis. Thus, we have the following result.

5.7.4 FINDING DISTANCE TRAVELED FROM THE VELOCITY VERSUS TIME CURVE. For
a particle in rectilinear motion, the total area between the velocity versus time curve and
an interval [t0 , t1 ] on the t-axis represents the distance traveled by the particle over that
time interval.

Example 9 For each of the velocity versus time curves in Figure 5.7.8 find the total
distance traveled by the particle over the time interval 0 ≤ t ≤ 4.

Solution. In all three parts of Figure 5.7.8 the total area between the curve and the interval
[0, 4] is 2, so the particle travels a distance of 2 units during the time period in all three cases,
even though the displacement is different in each case, as discussed in Example 7. 
••••••••••••••••••••••••••••••••••••••
In scientific work, numerical information is often summarized by computing some sort of
AVERAGE VALUE OF A average or mean value of the observed data. There are various kinds of averages, but the
CONTINUOUS FUNCTION
most common is the arithmetic mean or arithmetic average, which is formed by adding
the data and dividing by the number of data points. Thus, the arithmetic average a of n
numbers a1 , a2 , . . . , an is
n
1 1
a= (a1 + a2 + · · · + an ) = ak
n n k=1
In the case where the ak ’s are values of a function f , say,
a1 = f(x1 ), a2 = f(x2 ), . . . , an = f(xn )
then the arithmetic average a of these function values is
n
1
a= f(xk )
n k=1
We will now show how to extend this concept so that we can compute not only the
arithmetic average of finitely many function values but an average of all values of f(x) as
x varies over a closed interval [a, b]. For this purpose recall the Mean-Value Theorem for
Integrals (5.6.2), which states that if f is continuous on the interval [a, b], then there is at
least one number x ∗ in this interval such that
b
f(x) dx = f(x ∗ )(b − a)
a
January 18, 2001 14:01 g65-ch5 Sheet number 63 Page number 381 cyan magenta yellow black

5.7 Rectilinear Motion Revisited; Average Value 381

The quantity
b
1
f(x ∗ ) = f(x) dx (19)
b−a a

will be our candidate for the average value of f over the interval [a, b]. To explain what
motivates this, divide the interval [a, b] into n subintervals of equal length
b−a
)x = (20)
n
and choose arbitrary numbers x1∗ , x2∗ , . . . , xn∗ in successive subintervals. Then the arithmetic
average of the values f(x1∗ ), f(x2∗ ), . . . , f(xn∗ ) is
1
ave = [f(x1∗ ) + f(x2∗ ) + · · · + f(xn∗ )]
n
or from (20)
n
1 1
ave = [f(x1∗ ))x + f(x2∗ ))x + · · · + f(xn∗ ))x] = f(xk∗ ))x
b−a b−a k=1

Taking the limit as n → +⬁ yields


n b
1 1
lim f(xk∗ ))x = f(x) dx
n → +⬁ b − a b−a a
k=1

Since this equation describes what happens when we compute the average of “more and
more” values of f(x), we are led to the following definition.

5.7.5 DEFINITION. If f is continuous on [a, b], then the average value (or mean
value) of f on [a, b] is defined to be
b
1
fave = f(x) dx (21)
b−a a

• REMARK.
• When f is nonnegative on [a, b], the quantity fave has a simple geometric inter-







pretation, which can be seen by writing (21) as






b
fave · (b − a) =





f(x) dx

y = f (x) •


a






The left side of this equation is the area of a rectangle with a height of fave and base of length
b − a, and the right side is the area under y = f(x) over [a, b]. Thus, fave is the height of











a rectangle constructed over the interval [a, b], whose area is the same as the area under

fave




the graph of f over that interval (Figure 5.7.10). Note also that the Mean-Value Theorem,
when expressed in form (21), ensures that there is always at least one number x ∗ in [a, b]





a b

at which the value of f is equal to the average value of f over the interval.
Figure 5.7.10 √
Example 10 Find the average value of the function f(x) = x over the interval [1, 4],
and find all numbers in the interval at which the value of f is the same as the average.

Solution.
 4
1 b
1 4 √
1 2x 3/2
fave = f(x) dx = x dx =
b−a a 4−1 1 3 3
1

1 16 2 14
= − = ≈ 1.6
3 3 3 9
January 18, 2001 14:01 g65-ch5 Sheet number 64 Page number 382 cyan magenta yellow black

382 Integration

√ √
The x-values at which f(x) = x is the same as the average satisfy x = 14/9, from
which we obtain x = 196/81 ≈ 2.4 (Figure 5.7.11). 
y
y = √x
2
14
fave = 9
1

x
1 2 3 4
196
81

Figure 5.7.11

••••••••••••••••••••••••••••••••••••••
In Section 3.1 we considered the motion of a particle moving along a coordinate line, and
AVERAGE VELOCITY REVISITED we motivated the concept of instantaneous velocity by viewing it as the limit of average
velocities over smaller and smaller time intervals. That discussion led us to conclude that
the average velocity of the particle over a time interval could be interpreted as the slope of
a secant line of the position versus time curve (Figure 3.1.6). We will now show that the
same result is true if Definition 5.7.5 is used to compute the average velocity.
For this purpose, suppose that s(t) and v(t) are the position and velocity functions of
such a particle, and let us use Formula (21) to calculate the average velocity of the particle
over a time interval [t0 , t1 ]. This yields
1 t1
1 t1
s(t1 ) − s(t0 )
vave = v(t) dt = s  (t) dt =
t1 − t 0 t0 t1 − t 0 t0 t1 − t0
Thus, the average velocity over a time interval is the displacement divided by the elapsed
time. Geometrically, this is the slope of the secant line shown in Figure 5.7.12. Thus, the
discussion of average velocity in Section 3.1 is consistent with Definition 5.7.5.

s
s = s(t)
s(t1)

s(t1) – s(t0)
s(t0)
t1 – t0 t
t0 t1

s(t1) – s(t0) displacement


vave = =
t1 – t0 elapsed time

Figure 5.7.12

EXERCISE SET 5.7 Graphing Calculator C CAS


••••••••••••••••••••••••••••••••••••••••••••••••••••••••••••••••••••••••••••••••••••••••••••••••••••••••••••••

1. (a) If h (t) is the rate of change of a child’s height measured (c) If H (t) is the rate of change of the speed of sound with
 10
in inches per year, what does the integral 0 h (t) dt respect to temperature measured in ft/s per ◦ F, what
 100
represent, and what are its units? does the integral 32 H (t) dt represent, and what are
(b) If r  (t) is the rate of change of the radius of a spherical its units?
balloon measured in centimeters per second, what does (d) If v(t) is the velocity of a particle in rectilinear
 t motion,
2
the integral 1 r  (t) dt represent, and what are its units? measured in cm/h, what does the integral t12 v(t) dt
represent, and what are its units?
January 18, 2001 14:01 g65-ch5 Sheet number 65 Page number 383 cyan magenta yellow black

5.7 Rectilinear Motion Revisited; Average Value 383

2. (a) Suppose that sludge is emptied into a river at the rate of 10. (a) v(t) = t√2/3 ; s(8) = 0
V (t) gallons per minute, starting at time t = 0. Write (b) a(t) = t; v(4) = 1; s(4) = −5
an integral that represents the total volume of sludge
that is emptied into the river during the first hour. In Exercises 11–14, a particle moves with a velocity of v(t)
(b) Suppose that the tangent line to a curve y = f(x) has m/s along an s-axis. Find the displacement and the distance
slope traveled by the particle during the given time interval.
 x2 m(x) at the point x. What does the integral
x1 m(x) dx represent?
11. (a) v(t) = sin t; 0 ≤ t ≤ π/2
3. In each part, the velocity versus time curve is given for a
particle moving along a line. Use the curve to find the dis- (b) v(t) = cos t; π/2 ≤ t ≤ 2π
placement and the distance traveled by the particle over the 12. (a) v(t) = 2t − 4; 0 ≤ t ≤ 6
time interval 0 ≤ t ≤ 3. (b) v(t) = |t − 3|; 0 ≤ t ≤ 5
13. (a) v(t) = t 3 − 3t 2 + 2t; 0 ≤ t ≤ 3
(a) (b) √
v v (b) v(t) = t − 2; 0 ≤ t ≤ 3
1 1 14. (a) v(t) = 21 − (1/t 2 ); 1 ≤ t ≤ 3
t t √
(b) v(t) = 3/ t; 4 ≤ t ≤ 9
1 2 3 1 2 3
-1 -1 In Exercises 15–18, a particle moves with acceleration a(t)
m/s2 along an s-axis and has velocity v0 m/s at time t = 0.
Find the displacement and the distance traveled by the parti-
4. Sketch a velocity versus time curve for a particle that travels cle during the given time interval.
a distance of 5 units along a coordinate line during the time
interval 0 ≤ t ≤ 10 and has a displacement of 0 units. 15. a(t) = −2; v0 = 3; 1 ≤ t ≤ 4
5. The accompanying figure shows the acceleration versus 16. a(t) = t − 2; v0 = 0; 1 ≤ t ≤ 5
time curve for a particle moving along a coordinate line. √
17. a(t) = 1/ 5t + 1; v0 = 2; 0 ≤ t ≤ 3
If the initial velocity of the particle is 20 m/s, estimate
(a) the velocity at time t = 4 s 18. a(t) = sin t; v0 = 1; π/4 ≤ t ≤ π/2
(b) the velocity at time t = 6 s. 19. In each part use the given information to find the position,
velocity, speed, and acceleration at time t = 1.
a (m/s2 ) (a) v = sin 12 πt; s = 0 when t = 0
10 (b) a = −3t; s = 1 and v = 0 when t = 0
20. The accompanying figure shows the velocity versus time
curve over the time interval 1 ≤ t ≤ 5 for a particle moving
along a horizontal coordinate line.
5 (a) What can you say about the sign of the acceleration over
the time interval?
(b) When is the particle speeding up? Slowing down?
(c) What can you say about the location of the particle at
t (s)
5 10
time t = 5 relative to its location at time t = 1? Explain
your reasoning.
Figure Ex-5

6. Determine whether the particle in Exercise 5 is speeding up v


or slowing down at times t = 4 s and t = 6 s.
t
In Exercises 7–10, a particle moves along an s-axis. Use the 1 2 3 4 5
given information to find the position function of the particle.

7. (a) v(t) = t 3 − 2t 2 + 1; s(0) = 1 Figure Ex-20


(b) a(t) = 4 cos 2t; v(0) = −1; s(0) = −3
8. (a) v(t) = 1 + sin t; s(0) = −3 In Exercises 21–24, sketch the curve and find the total area
(b) a(t) = t 2 − 3t + 1; v(0) = 0; s(0) = 0 between the curve and the given interval on the x-axis.
9. (a) v(t) = 2t − 3; s(1) = 5
(b) a(t) = cos t; v(π/2) = 2; s(π/2) = 0 21. y = x 2 − 1; [0, 3] 22. y = sin x; [0, 3π/2]
January 18, 2001 14:01 g65-ch5 Sheet number 66 Page number 384 cyan magenta yellow black

384 Integration

√ x2 − 1 1
23. y = x + 1 − 1; [−1, 1] 24. y = ; 2, 2 (b) A bicycle rider traveling on a straight path accelerates
x2 uniformly from rest to 30 km/h in 1 min. Find his ac-
25. Suppose that the velocity function of a particle moving along celeration in km/s2 .
an s-axis is v(t) = 20t 2 − 100t + 50 ft/s and that the par-
32. A car traveling 60 mi/h along a straight road decelerates at
ticle is at the origin at time t = 0. Use a graphing utility
a constant rate of 10 ft/s2 .
to generate the graphs of s(t), v(t), and a(t) for the first
(a) How long will it take until the speed is 45 mi/h?
6 s of motion.
(b) How far will the car travel before coming to a stop?
26. Suppose that the acceleration function of a particle moving
33. Spotting a police car, you hit the brakes on your new Porsche
along an s-axis is a(t) = 4t − 30 m/s and that the position
to reduce your speed from 90 mi/h to 60 mi/h at a constant
and velocity at time t = 0 are s0 = −5 m and v0 = 3 m/s.
rate over a distance of 200 ft.
Use a graphing utility to generate the graphs of s(t), v(t),
(a) Find the acceleration in ft/s2 .
and a(t) for the first 25 s of motion.
(b) How long does it take for you to reduce your speed to
C 27. Let the velocity function for a particle that is at the origin 55 mi/h?
initially and moves along an s-axis be v(t) = 0.5 − t sin t. (c) At the acceleration obtained in part (a), how long would
(a) Generate the velocity versus time curve, and use it to it take for you to bring your Porsche to a complete stop
make a conjecture about the sign of the displacement from 90 mi/h?
over the time interval 0 ≤ t ≤ 5.
34. A particle moving along a straight line is accelerating at
(b) Use a CAS to find the displacement.
a constant rate of 3 m/s2 . Find the initial velocity if the
C 28. Let the velocity function for a particle that is at the origin particle moves 40 m in the first 4 s.
initially and moves along an s-axis be v(t) = 0.5 − t cos πt.
35. A motorcycle, starting from rest, speeds up with a constant
(a) Generate the velocity versus time curve, and use it to
acceleration of 2.6 m/s2 . After it has traveled 120 m, it slows
make a conjecture about the sign of the displacement
down with a constant acceleration of −1.5 m/s2 until it at-
over the time interval 0 ≤ t ≤ 1.
tains a speed of 12 m/s. What is the distance traveled by the
(b) Use a CAS to find the displacement.
motorcycle at that point?
29. Suppose that at time t = 0 a particle is at the origin of an
36. A sprinter in a 100-m race explodes out of the starting block
x-axis and has a velocity of v0 = 25 cm/s. For the first 4 s
with an acceleration of 4.0 m/s2 , which she sustains for 2.0
thereafter it has no acceleration, and then it is acted on by
s. Her acceleration then drops to zero for the rest of race.
a retarding force that produces a constant negative acceler-
(a) What is her time for the race?
ation of a = −10 cm/s2 .
(b) Make a graph of her distance from the starting block
(a) Sketch the acceleration versus time curve over the in-
versus time.
terval 0 ≤ t ≤ 12.
(b) Sketch the velocity versus time curve over the time in- 37. A car that has stopped at a toll booth leaves the booth with
terval 0 ≤ t ≤ 12. a constant acceleration of 2 ft/s2 . At the time the car leaves
(c) Find the x-coordinate of the particle at times t = 8 s the booth it is 5000 ft behind a truck traveling with a con-
and t = 12 s. stant velocity of 50 ft/s. How long will it take for the car
(d) What is the maximum x-coordinate of the particle over to catch the truck, and how far will the car be from the toll
the time interval 0 ≤ t ≤ 12? booth at that time?
30. Formulas (8) and (9) for uniformly accelerated motion can 38. In the final sprint of a rowing race the challenger is rowing
be rearranged in various useful ways. For simplicity, let at a constant speed of 12 m/s. At the point where the leader
s = s(t) and v = v(t), and derive the following variations is 100 m from the finish line and the challenger is 15 m be-
of those formulas. hind, the leader is rowing at 8 m/s but starts accelerating at
v 2 − v02 2(s − s0 ) a constant 0.5 m/s2 . Who wins?
(a) a = (b) t =
2(s − s0 ) v0 + v
In Exercises 39–48, assume that a free-fall model applies.
(c) s = s0 + vt − 12 at 2 [Note how this differs from (8).] Solve these exercises by applying Formulas (12) and (13) or,
if appropriate, use those from Exercise 30 with a = −g. In
Exercises 31–38 involve uniformly accelerated motion. In
these exercises take g = 32 ft/s2 or g = 9.8 m/s2 , depending
these exercises assume that the object is moving in the pos-
on the units.
itive direction of a coordinate line, and apply Formulas (8)
and (9) or those from Exercise 30, as appropriate. In some of
these problems you will need the fact that 88 ft/s = 60 mi/h. 39. A projectile is launched vertically upward from ground level
with an initial velocity of 112 ft/s.
31. (a) An automobile traveling on a straight road decelerates (a) Find the velocity at t = 3 s and t = 5 s.
uniformly from 55 mi/h to 25 mi/h in 30 s. Find its (b) How high will the projectile rise?
acceleration in ft/s2 . (c) Find the speed of the projectile when it hits the ground.
January 18, 2001 14:01 g65-ch5 Sheet number 67 Page number 385 cyan magenta yellow black

5.7 Rectilinear Motion Revisited; Average Value 385

40. A projectile fired downward from a height of 112 ft reaches 48. Given that the Moon’s gravity is 1/6 of the Earth’s, how
the ground in 2 s. What is its initial velocity? much faster would a projectile have to be launched upward
41. A projectile is fired vertically upward from ground level from the surface of the Earth than from the surface of the
with an initial velocity of 16 ft/s. Moon to reach a height of 1000 ft?
(a) How long will it take for the projectile to hit the ground?
In Exercises 49–52, find the average value of the function
(b) How long will the projectile be moving upward?
over the given interval.
42. A rock is dropped from the top of the Washington Monu-
ment, which is 555 ft high. 49. f(x) = 3x; [1, 3] 50. f(x) = x 2 ; [−1, 2]
(a) How long will it take for the rock to hit the ground?
51. f(x) = sin x; [0, π] 52. f(x) = cos x; [0, π]
(b) What is the speed of the rock at impact?
43. A helicopter pilot drops a package when the helicopter is 53. (a) Find fave of f(x) = x over [0, 2].
2

200 ft above the ground and rising at a speed of 20 ft/s. (b) Find a number x ∗ in [0, 2] such that f(x ∗ ) = fave .
(c) Sketch the graph of f(x) = x 2 over [0, 2] and construct
(a) How long will it take for the package to hit the ground?
a rectangle over the interval whose area is the same as
(b) What will be its speed at impact?
the area under the graph of f over the interval.
44. A stone is thrown downward with an initial speed of 96 ft/s
from a height of 112 ft. 54. (a) Find fave of f(x) = 2x over [0, 4].
(b) Find a number x ∗ in [0, 4] such that f(x ∗ ) = fave .
(a) How long will it take for the stone to hit the ground?
(c) Sketch the graph of f(x) = 2x over [0, 4] and construct
(b) What will be its speed at impact?
a rectangle over the interval whose area is the same as
45. A projectile is fired vertically upward with an initial velocity the area under the graph of f over the interval.
of 49 m/s from a tower 150 m high.
55. (a) Suppose that the velocity function of a particle mov-
(a) How long will it take for the projectile to reach its max-
ing along a coordinate line is v(t) = 3t 3 + 2. Find the
imum height?
average velocity of the particle over the time interval
(b) What is the maximum height?
1 ≤ t ≤ 4 by integrating.
(c) How long will it take for the projectile to pass its starting
(b) Suppose that the position function of a particle mov-
point on the way down?
ing along a coordinate line is s(t) = 6t 2 + t. Find the
(d) What is the velocity when it passes the starting point on
average velocity of the particle over the time interval
the way down?
1 ≤ t ≤ 4 algebraically.
(e) How long will it take for the projectile to hit the ground?
(f ) What will be its speed at impact? 56. (a) Suppose that the acceleration function of a particle mov-
ing along a coordinate line is a(t) = t + 1. Find the av-
46. A man drops a stone from a bridge. What is the height of
erage acceleration of the particle over the time interval
the bridge if
0 ≤ t ≤ 5 by integrating.
(a) the stone hits the water 4 s later
(b) Suppose that the velocity function of a particle moving
(b) the sound of the splash reaches the man 4 s later? [Take
along a coordinate line is v(t) = cos t. Find the aver-
1080 ft/s as the speed of sound.]
age acceleration of the particle over the time interval
47. In the final stages of a Moon landing, a lunar module fires its 0 ≤ t ≤ π/4 algebraically.
retrorockets and descends to a height of h = 5 m above the
57. Water is run at a constant rate of 1 ft3 /min to fill a cylindrical
lunar surface (Figure Ex-47). At that point the retrorockets
tank of radius 3 ft and height 5 ft. Assuming that the tank is
are cut off, and the module goes into free fall. Given that
empty initially, make a conjecture about the average weight
the Moon’s gravity is 1/6 of the Earth’s, find the speed of
of the water in the tank over the time period required to fill
the module when it touches the lunar surface.
it, and then check your conjecture by integrating. [Take the
weight density of water to be 62.4 lb/ft3 .]
58. (a) The temperature of a 10-m-long metal bar is 15 ◦ C at
one end and 30 ◦ C at the other end. Assuming that the
temperature increases linearly from the cooler end to
the hotter end, what is the average temperature of the
bar?
(b) Explain why there must be a point on the bar where the
temperature is the same as the average, and find it.
h 59. (a) Suppose that a reservoir supplies water to an industrial
park at a constant rate of r = 4 gallons per minute
(gal/min) between 8:30 A.M. and 9:00 A.M. How much
Figure Ex-47 water does the reservoir supply during that time period?
January 18, 2001 14:01 g65-ch5 Sheet number 68 Page number 386 cyan magenta yellow black

386 Integration

(b) Suppose that one of the industrial plants increases its 60. A traffic engineer monitors the rate at which cars enter the
water consumption between 9:00 A.M. and 10:00 A.M. main highway during the afternoon rush hour. From her data
and that the rate at which the reservoir supplies water she estimates that between 4:30 P.M. and 5:30 P.M. the rate
increases linearly, as shown in the accompanying fig- R(t) at which cars enter the highway is given by the formula
ure. How much water does the reservoir supply during R(t) = 100(1 − 0.0001t 2 ) cars per minute, where t is the
that 1-hour time period? time (in minutes) since 4:30 P.M.
(c) Suppose that from 10:00 A.M. to 12 noon the rate at (a) When does the peak traffic flow into the highway occur?
which the reservoir√supplies water is given by the for-
(b) Estimate the number of cars that enter the highway dur-
mula r(t) = 10 + t gal/min, where t is the time (in
ing the rush hour.
minutes) since 10:00 A.M. How much water does the
reservoir supply during that 2-hour time period? 61. (a) Prove: If f is continuous on [a, b], then
b
Water Consumption [f(x) − fave ] dx = 0
10 a
9
8 (b) Does there exist a constant c = fave such that
r (gal/min)

7
6 b
5
4
[f(x) − c] dx = 0?
3 a
2
1
0 10 20 30 40 50 60
9:00 a.m. Time (min) 10:00 a.m.
Figure Ex-59

5.8 EVALUATING DEFINITE INTEGRALS BY SUBSTITUTION

In this section we will discuss two methods for evaluating definite integrals in which a
substitution is required.
••••••••••••••••••••••••••••••••••••••
Recall from Section 5.3 that indefinite integrals of the form
TWO METHODS FOR MAKING
SUBSTITUTIONS IN DEFINITE f(g(x))g  (x) dx
INTEGRALS
can sometimes be evaluated by making the u-substitution
u = g(x), du = g  (x) dx (1)
which converts the integral to the form

f(u) du

To apply this method to a definite integral of the form


b
f(g(x))g  (x) dx
a
we need to account for the effect that the substitution has on the x-limits of integration.
There are two ways of doing this.
Method 1 First evaluate the indefinite integral

f(g(x))g  (x) dx

by substitution, and then use the relationship


b b
f(g(x))g  (x) dx = f(g(x))g  (x) dx
a a
to evaluate the definite integral. This procedure does not require any modification of the
x-limits of integration.
January 18, 2001 14:01 g65-ch5 Sheet number 69 Page number 387 cyan magenta yellow black

5.8 Evaluating Definite Integrals by Substitution 387

Method 2 Make the substitution (1) directly in the definite integral, and then use the
relationship u = g(x) to replace the x-limits, x = a and x = b, by corresponding u-limits,
u = g(a) and u = g(b). This produces a new definite integral
g(b)
f(u) du
g(a)
that is expressed entirely in terms of u.
2
Example 1 Use the two methods above to evaluate x(x 2 + 1)3 dx.
0

Solution by Method 1. If we let


u = x2 + 1 so that du = 2x dx (2)
then we obtain
1 u4 (x 2 + 1)4
x(x 2 + 1)3 dx = u3 du = +C = +C
2 8 8
Thus,
2 2
2
(x 2 + 1)4
x(x + 1) dx =
2 3
x(x + 1) dx
2 3
=
0 x=0 8 x=0
625 1
= − = 78
8 8

Solution by Method 2. If we make the substitution u = x 2 + 1 in (2), then


if x = 0, u=1
if x = 2, u=5
Thus,
2 5 5
1 u4 625 1
x(x + 1) dx =
2 3
u du =
3
= − = 78
0 2 1 8 u=1 8 8
which agrees with the result obtained by Method 1. 
The following theorem states precise conditions under which Method 2 can be used.

5.8.1 THEOREM. If g  is continuous on [a, b] and f is continuous on an interval


containing the values of g(x) for a ≤ x ≤ b, then
b g(b)
f(g(x))g  (x) dx = f(u) du
a g(a)

Proof. Since f is continuous on an interval containing the values of g(x) for a ≤ x ≤ b,


it follows that f has an antiderivative F on that interval. If we let u = g(x), then the chain
rule implies that
d d dF du du
F (g(x)) = F (u) = = f (u) = f (g(x))g  (x)
dx dx du dx dx
for each x in [a, b]. Thus, F (g(x)) is an antiderivative of f (g(x))g  (x) on [a, b]. Therefore,
by Part 1 of the Fundamental Theorem of Calculus (Theorem 5.6.1)
b b g(b)

f (g(x))g (x) dx = F (g(x)) = F (g(b)) − F (g(a)) = f(u) du
a a g(a)

The choice of methods for evaluating definite integrals by substitution is generally a


matter of taste, but in the following examples we will use the second method, since the idea
is new.
January 18, 2001 14:01 g65-ch5 Sheet number 70 Page number 388 cyan magenta yellow black

388 Integration

Example 2 Evaluate
π/8 5
(a) sin5 2x cos 2x dx (b) (2x − 5)(x − 3)9 dx
0 2

Solution (a). Let


 
u = sin 2x so that du = 2 cos 2x dx or 1
2
du = cos 2x dx
With this substitution,
if x = 0, u = sin(0) = 0

if x = π/8, u = sin(π/4) = 1/ 2
so
√ 1/√2
π/8 1/ 2
1 1 u6
sin5 2x cos 2x dx = u5 du = ·
0 2 0 2 6

u=0
1 1 1
= √ −0 =
2 6( 2) 6 96

Solution (b). Let


u=x−3 so that du = dx
This leaves a factor of 2x + 5 unresolved in the integrand. However,
x = u + 3, so 2x − 5 = 2(u + 3) − 5 = 2u + 1
With this substitution,
if x = 2, u = 2 − 3 = −1
if x = 5, u=5−3=2
so
5 2 2
(2x − 5)(x − 3)9 dx = (2u + 1)u9 du = (2u10 + u9 ) du
2 −1 −1
11 10 2    
2u u 212 210 2 1
= + = + − − +
11 10 u=−1 11 10 11 10
52,233 93
= = 474 
110 110

Example 3 Find the average value of the function


cos(π/x)
f(x) =
x2
over the interval [1, 3].

Solution. From Definition 5.7.5 the average value of f over the interval [1, 3] is
3
1 cos(π/x) 1 3 cos(π/x)
fave = dx = dx
3−1 1 x2 2 1 x2
To evaluate this integral, we make the substitution
π π 1 1 1
u= so that du = − 2 dx = −π · 2 dx or − du = 2 dx
x x x π x
With this substitution,
if x = 1, u=π
if x = 3, u = π/3
January 18, 2001 14:01 g65-ch5 Sheet number 71 Page number 389 cyan magenta yellow black

5.8 Evaluating Definite Integrals by Substitution 389

Thus, the average value of f over the interval [1, 3] is


  π/3
1 3 cos(π/x) 1 1
fave = dx = · − cos u du
2 1 x2 2 π π
π/3 √
1 1 3
= − sin u = − (sin(π/3) − sin π) = − ≈ −0.1378 
2π u=π 2π 4π
• REMARK.
• Observe that the u-substitution in this example produced an integral in which








the upper u-limit of integration was smaller than the lower u-limit of integration. In our





computations we left the limits of integration in that order, but we could have reversed the





order to put the larger limit on top and compensated by reversing the sign of the integral





in accordance with Definition 5.5.3(b). The choice of procedures is a matter of taste; both



• produce the same result (verify).

EXERCISE SET 5.8 C CAS


••••••••••••••••••••••••••••••••••••••••••••••••••••••••••••••••••••••••••••••••••••••••••••••••••••••••••••••
π/2 π/6
In Exercises 1 and 2, express the integral in terms of the vari- 9. 4 sin(x /2) dx 10. 2 cos 3x dx
0 0
able u, but do not evaluate it. −1
x 1+π 1 
11. dx 12. sec2 x− 1
dx
2 −2 (x 2 + 2)3 1−π
4 4

1. (a) (x + 1) dx; u = x + 1
7
0 In Exercises 13–16, evaluate the definite integral by express-
2  ing it in terms of u and evaluating the resulting integral using
(b) x 8− x2 dx; u = 8 − x 2
−1 a formula from geometry.
1
(c) sin(πθ) dθ ; u = πθ 5/3 
−1
13. 25 − 9x 2 dx; u = 3x
3
0
(d) (x + 2)(x − 3) dx; u = x − 3
20
2 
0
14. x 16 − x 4 dx; u = x 2
4
0
2. (a) (5 − 2x)8 dx; u = 5 − 2x π/2 
−1
2π/3
15. sin θ 1 − 4 cos2 θ dθ ; u = 2 cos θ
sin x π/3
(b) √ dx; u = 2 + cos x 1 
−π/3 2 + cos x
π/4
16. 3 − 2x − x 2 dx; u = x + 1
−3
(c) tan x sec x dx; u = tan x
2 2
0 17. Find the area under the curve y = sin πx over the interval
1  [0, 1].
(d) x 3 x 2 + 3 dx; u = x 2 + 3
0
18. Find the area under the curve y = 3 cos 2x over the interval
[0, π/8].
In Exercises 3–12, evaluate the definite integral two ways: 19. Find the area under the curve y = 1/(x + 5)2 over the in-
first by a u-substitution in the definite integral and then by a terval [3, 7].
u-substitution in the corresponding indefinite integral.
20. Find the area under the curve y = 1/(3x + 1)2 over the in-
1 2 terval [0, 1].
3. (2x + 1)4 dx 4. (4x − 2)3 dx 21. Find the average value of
0 1
x
0 2 f(x) =
5. (1 − 2x)3 dx 6. (4 − 3x)8 dx (5x 2 + 1)2
−1 1
over the interval [0, 2].
8 √ 0 √
7. x 1 + x dx 8. x 4 − x dx 22. Find the average value of f(x) = sec2 πx over the interval
0 −5 − 14 , 14 .
January 18, 2001 14:01 g65-ch5 Sheet number 72 Page number 390 cyan magenta yellow black

390 Integration

43. Electricity is supplied to homes in the form of alternating


In Exercises 23–36, evaluate the integrals by any method.
current, which means that the voltage has a sinusoidal wave-
form described by an equation of the form
1
dx 2 √
23. √ 24. 5x − 1 dx V = Vp sin(2πft)
0 3x + 1 1
1 0 (see the accompanying figure). In this equation, Vp is called
x 2 dx
25.  26. 6t 2 (t 3 + 1)19 dt the peak voltage or amplitude of the current, f is called
−1 x3 + 9 −1 its frequency, and 1/f is called its period. The voltages V
3
x+2 2
dx and Vp are measured in volts (V), the time t is measured
27. √ dx 28. in seconds (s), and the frequency is measured in hertz (Hz)
1 x 2 + 4x + 7 1 x 2 − 6x + 9
or sometimes in cycles per second. (A cycle is the electri-
π/4 π/4 √
29. sin x cos x dx 30. tan x sec2 x dx cal term for one period of the waveform.) Most alternating-
−3π/4 0 current voltmeters read what is called the rms or root-mean-

π 4π2
1 √ square value of V . By definition, this is the square root of
31. 5x cos(x 2 ) dx 32. √ sin x dx the average value of V 2 over one period.
0 π2 x
(a) Show that
π/9 π/2
33. sec2 3θ dθ 34. sin2 3θ cos 3θ dθ Vp
Vrms = √
π/12 0
2
1 4
y 2 dy x dx
35.  36. √ [Hint: Compute the average over the cycle from t = 0
0 4 − 3y −1 5+x to t = 1/f , and use the identity sin2 θ = 12 (1 − cos 2θ)
C 37. (a) Use a CAS to find the exact value of the integral to help evaluate the integral.]
π/6 (b) In the United States, electrical outlets supply alternat-
sin4 x cos3 x dx ing current with an rms voltage of 120 V at a frequency
0
of 60 Hz. What is the peak voltage at such an outlet?
(b) Confirm the exact value by hand calculation.
[Hint: Use the identity cos2 x = 1 − sin2 x.] V
Vp
C 38. (a) Use a CAS to find the exact value of the integral Vrms
π/4 t
4
tan x dx
−π/4

(b) Confirm the exact value by hand calculation. –Vp


[Hint: Use the identity 1 + tan2 x = sec2 x.]
1 4 V = Vp sin (oft)
39. (a) Find f(3x + 1) dx if f(x) dx = 5. Figure Ex-43
0 1
3 9 44. Show that if f and g are continuous functions, then
(b) Find f(3x) dx if f(x) dx = 5. t t
0 0
f(t − x)g(x) dx = f(x)g(t − x) dx
0 4 0 0
(c) Find 2
xf(x ) dx if f(x) dx = 1. a
f(x)
−2 0 45. (a) Let I = dx. Show that I = a /2.
40. Given that m and n are positive integers, show that 0 f(x) + f(a − x)

1 1 [Hint: Let u = a − x, and then note the difference


x m (1 − x)n dx = x n (1 − x)m dx between the resulting integrand and 1.]
0 0 (b) Use the result of part (a) to find
by making a substitution. Do not attempt to evaluate the 3

x
integrals. √ √ dx
0 x+ 3−x
41. Given that n is a positive integer, show that
π/2 π/2 (c) Use the result of part (a) to find
sinn x dx = cosn x dx π/2
sin x
0 0 dx
0 sin x + cos x
by using a trigonometric identity and making a substitution. 1
Do not attempt to evaluate the integrals. 1
46. Let I = dx. Show that the substitution x = 1/u
42. Given that n is a positive integer, evaluate the integral −1 1 + x2
results in
1
1
x(1 − x)n dx 1
I =− du = −I
0
−1 1 + u2
January 18, 2001 14:01 g65-ch5 Sheet number 73 Page number 391 cyan magenta yellow black

Supplementary Exercises 391

so 2I = 0, which implies that I = 0. However, this is im- and give a geometric explanation of this result. [Hint:
possible since the integrand of the given integral is positive One way to prove that a quantity q is zero is to show
over the interval of integration. Where is the error? that q = −q.]
47. Find the limit (b) Prove that if f is an even function, then
n a a
sin(kπ/n) f(x) dx = 2 f(x) dx
lim
n → +⬁ n −a 0
k=1
by evaluating an appropriate definite integral over the inter- and give a geometric explanation of this result. [Hint:
val [0, 1]. Split the interval of integration from −a to a into two
parts at 0.]
C 48. Check your answer to Exercise 47 by evaluating the limit
directly with a CAS. 50. Evaluate
1  π
49. (a) Prove that if f is an odd function, then (a) x cos(x 2 ) dx (b) sin8 x cos5 x dx.
a −1 0
f(x) dx = 0
−a
[Hint: Use the substitution u = x − (π/2).]

SUPPLEMENTARY EXERCISES
C CAS

1. Write a paragraph that describes the rectangle method for


defining the area under a curve y = f(x) over an interval
[a, b].
2. What is an integral curve of a function f ? How are two inte-
gral curves of a function f related? Figure Ex-6
3. The definite integral of f over the interval [a, b] is defined
as the limit 7. Suppose that
b n
f(xk∗ ))xk
1 2
f(x) dx = lim
a max )xk → 0 f(x) dx = 21 , f(x) dx = 14 ,
k=1
0 1
Explain what the various symbols on the right side of this 3 1
equation mean. f(x) dx = −1, g(x) dx = 2
0 0
4. State the two parts of the Fundamental Theorem of Calcu-
lus, and explain what is meant by the phrase “differentiation In each part, use this information to evaluate the given inte-
and integration are inverse processes.” gral, if possible. If there is not enough information to eval-
5. Derive the formulas for the position and velocity functions uate the integral, then say so.
2 3 3
of a particle that moves with uniformly accelerated motion
(a) f(x) dx (b) f(x) dx (c) 5f(x) dx
along a coordinate line. 0 1 2
6. (a) Devise a procedure for finding upper and lower esti- 0 1 1
mates of the area of the region in the accompanying (d) g(x) dx (e) g(2x) dx (f ) [g(x)]2 dx
1 0 0
figure (in cm2 ).
(b) Use your procedure to find upper and lower estimates 8. In each part, use the information in Exercise 7 to evaluate
of the area. the given integral. If there is not enough information to
(c) Improve on the estimates you obtained in part (b). evaluate the integral, then say so.
January 18, 2001 14:01 g65-ch5 Sheet number 74 Page number 392 cyan magenta yellow black

392 Integration

1 1
(a) [f(x) + g(x)] dx (b) f(x)g(x) dx 14. The accompanying figure shows the direction field for a dif-
0 0 ferential equation dy/dx = f(x). Which of the following
1 1 functions is most likely to be f(x)?
f(x)
(c) dx (d) [4g(x) − 3f(x)] dx √
0 g(x) 0 x, sin x, x 4 , x
9. In each part, evaluate the integral. Where appropriate, you Explain your reasoning.
may use a geometric formula.
1  y
(a) (1 + 1 − x 2 ) dx
−1
3  
(b) (x x 2 + 1 − 9 − x 2 ) dx
0
1  x
(c) x 1 − x 4 dx
0
1
10. Evaluate the integral 0 |2x − 1| dx, and sketch the region
whose area it represents.
11. One of the numbers π, π/2, 35π/128, 1 − π is the correct
value of the integral
π
sin8 x dx Figure Ex-14
0
Use the accompanying graph of y = sin8 x and a logical 15. In each part, confirm the stated equality.
process of elimination to find the correct value. [Do not
(a) 1 · 2 + 2 · 3 + · · · + n(n + 1) = 31 n(n + 1)(n + 2)
attempt to evaluate the integral.]
n−1  
y 9 k 17
(b) lim − 2 =
n → +⬁ n n 2
1 k=1
 
3 2
(c) (i + j ) = 21
i=1 j =1
x
c 16. Express
Figure Ex-11 18
k(k − 3)
12. In each part, find the limit by interpreting it as a limit of k=4
Riemann sums in which the interval [0, 1] is divided into n
in sigma notation with
subintervals of equal length.
√ √ √ √ (a) k = 0 as the lower limit of summation
1 + 2 + 3 + ··· + n (b) k = 5 as the lower limit of summation.
(a) lim
n → +⬁ n3/2 17. The accompanying figure shows a square that is n units by
1 + 2 + 3 + · · · + n4
4 4 4
n units that has been subdivided into a one-unit square and
(b) lim
n → +⬁ n5 n − 1 “L-shaped” regions. Use this figure to show that the
13. The accompanying figure shows five points on the graph sum of the first n consecutive positive odd integers is n2 .
of an unknown function f . Devise a strategy for using the
known points to approximate the area A under the graph of 1 2 3 4 ... n
y = f(x) over the interval [1, 5]. Describe your strategy, 1
and use it to approximate A. 2
3
y
4
(2, 3) .. ...
.
n
(3, 2) (5, 2)
(1, 2) Figure Ex-17
(4, 1)
x
1 2 3 4 5 18. Derive the result of Exercise 17 by writing
n
Figure Ex-13 1 + 3 + 5 + · · · + 2n − 1 = (2k − 1)
k=1
January 18, 2001 14:01 g65-ch5 Sheet number 75 Page number 393 cyan magenta yellow black

Supplementary Exercises 393

C 28. In each part, make a conjecture about the limit by using a


When part of each term of a sum cancels part of the next
CAS to evaluate the sum for n = 10, 20, and 50; and then
term, leaving only portions of the first and last terms at the
check your conjecture by using the formula in Exercise 26
end, the sum is said to telescope. In Exercises 19–22, evaluate
to express the sum in closed form, and then finding the limit
the telescoping sum.
exactly.
  n n  k
17 50 1 3
1 1 (a) lim (b) lim
19. (3 − 3
k k−1
) 20. − n → +⬁ 2 k n → +⬁ 4
k=5 k=1
k k+1 k=0 k=1

20   100 29. (a) Show that the substitutions u = sec x and u = tan x
1 1
21. − 22. (2k+1 − 2k ) produce different values for the integral
k=2
k 2 (k − 1)2 k=1
23. (a) Show that sec2 x tan x dx
1 1 1 n
+ + ··· + = (b) Explain why both are correct.
1·3 3·5 (2n − 1)(2n + 1) 2n + 1
   30. Use the two substitutions in Exercise 29 to evaluate the def-
1 1 1 1 inite integral
Hint: = − .
(2n − 1)(2n + 1) 2 2n − 1 2n + 1 π/4
sec2 x tan x dx
(b) Use the result in part (a) to find 0

n and confirm that they produce the same result.


1
lim 31. Evaluate the integral
n → +⬁
k=1
(2k − 1)(2k + 1)

24. (a) Show that 1 + x −2/3 dx
1 1 1 1 n
+ + + ··· + = by making the substitution u = 1 + x 2/3 .
1·2 2·3 3·4 n(n + 1) n+1
 32. (a) Express the equation
1 1 1
Hint: = − . b
n(n + 1) n n+1 [f1 (x) + f2 (x) + · · · + fn (x)] dx
a
(b) Use the result in part (a) to find b b b
= f1 (x) dx + f2 (x) dx + · · · + fn (x)] dx
n
1 a a a
lim
n → +⬁
k=1
k(k + 1) in sigma notation.
(b) If c1 , c2 , . . . , cn are constants and f1 , f2 , . . . , fn are
25. Let x̄ denote the arithmetic average of the n numbers
integrable functions on [a, b], do you think it is always
x1 , x2 , . . . , xn . Use Theorem 5.4.1 to prove that
n
true that
  
(xi − x̄) = 0 b n n b
i=1 ck fk (x) dx = ck fk (x) dx ?
26. Let a k=1 k=1 a
n
S= ar k Explain your reasoning.
k=0 33. Find an integral formula for the antiderivative of 1/(1 + x 2 )
Show that S − rS = a − ar n+1 and hence that on the interval (−⬁, +⬁) whose value at x = 1 is (a) 0 and
n
a − ar n+1 (b) 2.
ar k = (r = 1) x
t −3
k=0
1−r C 34. Let F (x) = dt.
t 2+7
0
(A sum of this form is called a geometric sum.)
27. In each part, rewrite the sum, if necessary, so that the lower (a) Find the intervals on which F is increasing. Decreasing.
limit is 0, and then use the formula derived in Exercise 26 to (b) Find the open intervals on which F is concave up. Con-
evaluate the sum. Check your answers using the summation cave down.
feature of a calculating utility. (c) Find the x-values, if any, at which the function F has
20 30 100 absolute extrema.
1
(a) 3k (b) 2k (c) (−1)k+1 (d) Use a CAS to graph F , and confirm that the results in
k=1 k=5 k=0
2k parts (a), (b), and (c) are consistent with the graph.
January 18, 2001 14:01 g65-ch5 Sheet number 76 Page number 394 cyan magenta yellow black

394 Integration

35. Prove that the function 40. Suppose that a tumor grows at the rate of r(t) = t /7 grams
x 1/x per week. When, during the second 26 weeks of growth,
1 1
F (x) = dt + dt is the weight of the tumor the same as its average weight
0 1 + t 2
0 1 + t2
during that period?
is constant on the interval (0, +⬁).
36. What is the natural domain of the function In Exercises 41–46, evaluate the integrals by hand, and check
x
1 your answers with a CAS if you have one.
F (x) = dt?
1 t −9
2
 √
Explain your reasoning. cos 3x 3+ x
41. √ dx 42. √ dx
37. In each part, determine the values of x for which F (x) is 5 + 2 sin 3x x
positive, negative, or zero without performing the integra- x2
tion; explain your reasoning. 43. dx 44. x sec2 (ax 2 ) dx
(ax 3 + b)2
x   
x
t4 −1
(a) F (x) = dt (b) F (x) = 4 − t 2 dt 1
1 t +3
2
−1
45. u−4 + 3u−2 − du
−2 u5
38. Find a formula (defined piecewise) for the upper boundary 1
of the trapezoid shown in the accompanying figure, and then 46. sin2 (πx) cos(πx) dx
integrate that function to derive the formula for the area of 0

the trapezoid given on the inside front cover of this text. C 47. Use a CAS to approximate the area of the region in the first
quadrant that lies below the curve y = x + x 2 − x 3 and
y above the x-axis.
a C 48. In each part, use a CAS to solve the initial-value problem.
dy
(a) = x 2 cos 3x; y(π/2) = −1
h dx
dy x3
x (b) = ; y(0) = −2
b dx (4 + x 2 )3/2
Figure Ex-38
C 49. In each part, use a CAS, where needed, to solve for k.
k
39. The velocity of a particle moving along an s-axis is mea-
sured at 5-s intervals for 40 s, and the velocity function is (a) (x 3 − 2x − 1) dx = 0, k > 1
1
modeled by a smooth curve. The curve and the data points k
are shown in the accompanying figure. (b) (x 2 + sin 2x) dx = 3, k≥0
(a) Does the particle have constant acceleration? Explain 0
your reasoning. C 50. Use a CAS to approximate the largest and smallest values
(b) Is there any 15-s time interval during which the accel- of the integral
eration is constant? Explain your reasoning. x
(c) Estimate the average velocity of the particle over the t
 dt
40-s time period. −1 2 + t3
(d) Estimate the distance traveled by the particle from time
t = 0 to time t = 40. for 1 ≤ x ≤ 3.
(e) Is the particle ever slowing down during the 40-s time C 51. The function J0 defined by
period? Explain your reasoning. 1 π
(f ) Is there sufficient information for you to determine the J0 (x) = cos(x sin t) dt
s-coordinate of the particle at time t = 10? If so, find π 0

it. If not, explain what additional information you need. is called the Bessel function of order zero.
(a) Use a CAS to graph the equation y = J0 (x) over the
v (ft/s) interval 0 ≤ x ≤ 8.
8 (b) Estimate J0 (1).
(c) Estimate the smallest positive zero of J0 (x).
6
52. Find the area under the graph of f(x) = 5x − x 2 over the
interval [0, 5] using Definition 5.4.3 with xk∗ as the left end-
4
point of each subinterval.

t (s)
0
0 10 20 30 40 Figure Ex-39
January 18, 2001 14:01 g65-ch5 Sheet number 77 Page number 395 cyan magenta yellow black

Expanding the Calculus Horizon 395

EXPANDING THE CALCULUS HORIZON

Blammo the Human Cannonball


Blammo the Human Cannonball will be fired from a cannon and hopes to land in
a small net at the opposite end of the circus arena. Your job as Blammo’s manager
is to do the mathematical calculations that will allow Blammo to perform his death-
defying act safely. The methods that you will use are from the field of ballistics (the
study of projectile motion).

The Problem
Blammo’s cannon has a muzzle velocity of 35 m/s, which means that Blammo will leave the
muzzle with that velocity. The muzzle opening will be 5 m above the ground, and Blammo’s
objective is to land in a net that is also 5 m above the ground and that extends a distance of 10
m between 90 m and 100 m from the cannon opening (Figure 1). Your mathematical problem is
to determine the elevation angle α of the cannon (the angle from the horizontal to the cannon
barrel) that will make Blammo land in the net.

a Net x
5m
Ground 90 m 100 m
Figure 1

Modeling Assumptions
Blammo’s trajectory will be determined by his initial velocity, the elevation angle of the cannon,
and the forces that act on him after he leaves the muzzle. We will assume that the only force acting
on Blammo after he leaves the muzzle is the downward force of the Earth’s gravity. In particular,
we will ignore the effect of air resistance. It will be convenient to introduce the xy-coordinate
system shown in Figure 1 and to assume that Blammo is at the origin at time t = 0. We will also
assume that Blammo’s motion can be decomposed into two independent components, a horizontal
component parallel to the x-axis and a vertical component parallel to the y-axis. We will analyze
the horizontal and vertical components of Blammo’s motion separately, and then we will combine
the information to obtain a complete picture of his trajectory.

Blammo’s Equations of Motion


We will denote the position and velocity functions for Blammo’s horizontal component of motion
by x(t) and vx (t), and we will denote the position and velocity functions for his vertical component
of motion by y(t) and vy (t).
Since the only force acting on Blammo after he leaves the muzzle is the downward force
of the Earth’s gravity, there are no horizontal forces to alter his initial horizontal velocity vx (0).
Thus, Blammo will have a constant velocity of vx (0) in the x-direction; this implies that
x(t) = vx (0)t (1)
In the y-direction Blammo is acted on only by the downward force of the Earth’s gravity. Thus,
his motion in this direction is governed by the free-fall model; hence, from (12) in Section 5.7 his
January 18, 2001 14:01 g65-ch5 Sheet number 78 Page number 396 cyan magenta yellow black

396 Integration

vertical position function is


y(t) = y(0) + vy (0)t − 12 gt 2
Taking g = 9.8 m/s2 , and using the fact that y(0) = 0, this equation can be written as
y(t) = vy (0)t − 4.9t 2 (2)
•••••••••••
Exercise 1 At time t = 0 Blammo’s velocity is 35 m/s, and this velocity is directed at an
angle α with the horizontal. It is a fact of physics that the initial velocity components vx (0) and
vy (0) can be obtained geometrically from the muzzle velocity and the angle of elevation using
the triangle shown in Figure 2. We will justify this later in the text, but for now use this fact to
show that Equations (1) and (2) can be expressed as
x(t) = (35 cos α)t
y(t) = (35 sin α)t − 4.9t 2

35
vy(0)

a
vx (0)
Figure 2

•••••••••••
Exercise 2
(a) Use the result in Exercise 1 to find the velocity functions vx (t) and vy (t) in terms of the ele-
vation angle α.
(b) Find the time t at which Blammo is at his maximum height above the x-axis, and show that
this maximum height (in meters) is
ymax = 62.5 sin2 α

•••••••••••
Exercise 3 The equations obtained in Exercise 1 can be viewed as parametric equations for
Blammo’s trajectory. Show, by eliminating the parameter t, that if 0 < α < π/2, then Blammo’s
trajectory is given by the equation
0.004 2
y = (tan α)x − x
cos2 α
Explain why Blammo’s trajectory is a parabola.

Finding the Elevation Angle


Define Blammo’s horizontal range R to be the horizontal distance he travels until he returns to
the height of the muzzle opening (y = 0). Your objective is to find elevation angles that will make
the horizontal range fall between 90 m and 100 m, thereby ensuring that Blammo lands in the net
(Figure 3).

R Net x

Ground 90 m 100 m
Figure 3
January 18, 2001 14:01 g65-ch5 Sheet number 79 Page number 397 cyan magenta yellow black

Expanding the Calculus Horizon 397

•••••••••••
Exercise 4 Use a graphing utility and either the parametric equations obtained in Exercise 1
or the single equation obtained in Exercise 3 to generate Blammo’s trajectories, taking elevation
angles at increments of 10 ◦ from 15 ◦ to 85 ◦ . In each case, determine visually whether Blammo
lands in the net.

•••••••••••
Exercise 5 Find the time required for Blammo to return to his starting height (y = 0), and use
that result to show that Blammo’s range R is given by the formula
R = 125 sin 2α

•••••••••••
Exercise 6
(a) Use the result in Exercise 5 to find two elevation angles that will allow Blammo to hit the
midpoint of the net 95 m away.
(b) The tent is 55 m high. Explain why the larger elevation angle cannot be used.
•••••••••••
Exercise 7 How much can the smaller elevation angle in Exercise 6 vary and still have Blammo
hit the net between 90 m and 100 m?

Blammo’s Shark Trick


Blammo is to be fired from 5 m above ground level with a muzzle velocity of 35 m/s over a flaming
wall that is 20 m high and past a 5-m-high shark pool (Figure 4). To make the feat impressive, the
pool will be made as long as possible. Your job as Blammo’s manager is to determine the length
of the pool, how far to place the cannon from the wall, and what elevation angle to use to ensure
that Blammo clears the pool.

20 m
5m
Flaming Shark pool Net
wall
Figure 4

•••••••••••
Exercise 8 Prepare a written presentation of the problem and your solution of it that is at an
appropriate level for an engineer, physicist, or mathematician to read. Your presentation should
contain the following elements: an explanation of all notation, a list and description of all formulas
that will be used, a diagram that shows the orientation of any coordinate systems that will be used,
a description of any assumptions you make to solve the problem, graphs that you think will
enhance the presentation, and a clear step-by-step explanation of your solution.
..................................................................................................................................
Module by: John Rickert, Rose-Hulman Institute of Technology
Howard Anton, Drexel University

You might also like